Sie sind auf Seite 1von 141

Public Forum | December 2011

Resolved: In the United States, current income disparities threaten democratic ideals.

Victory Briefs Topic Analysis Book: Public Forum December 2011 11PF4-Income Disparities 2011 Victory Briefs, LLC Victory Briefs Topic Analysis Books are published by: Victory Briefs, LLC 925 North Norman Place Los Angeles, California 90049 Publisher: Victor Jih | Managing Editor: Adam Torson | Editor: Adam Torson | Topic Analysis Writers: Jessica Bailey, Ben Lewis, Bill Neesen, Adam Torson | Evidence: Adam Torson For customer support, please email help@victorybriefs.com or call 310.472.6364.

11PF4-Income Disparities www.victorybriefs.com

Page 2 of 141

TABLE OF CONTENTS
TABLE OF CONTENTS TOPIC ANALYSIS BY JESSICA BAILEY TOPIC ANALYSIS BY BEN LEWIS TOPIC ANALYSIS BY BILL NEESEN TOPIC ANALYSIS BY ADAM TORSON FRAMEWORK EVIDENCE 2 6 15 24 32 43

INCOME INEQUALITY IS INCREASING BOTH QUANTITATIVELY AND QUALITATIVELY .......................43 ECONOMIC INEQUALITY IS PARTICULARLY PRONOUNCED BETWEEN RACIAL AND ETHNIC GROUPS .......................................................................................................................................................44 INCOME INEQUALITY IS NOT THE SAME AS GENERAL ECONOMIC INEQUALITY, AND THE FORMER HAS BECOME MARKEDLY MORE STRATIFIED .........................................................................................45 THIRD WAVE DEMOCRATIC MOVEMENTS COINCIDED WITH THE ASCENDANCE OF NEOLIBERAL ECONOMICS AND SO FOCUSED LESS ON ECONOMIC INEQUALITY ....................................................46 NEOLIBERAL ECONOMICS HAS A GREATER TOLERANCE FOR INEQUALITY ......................................47 FEW PEOPLE BELIEVE THAT EITHER TRUE LAISSEZ FAIRE OR THE PURSUIT OF TOTAL EQUALITY ARE DESIRABLE POLICY OBJECTIVES .....................................................................................................48 THERE ARE A WIDE VARIETY OF ECONOMIC DISTRIBUTION PATTERNS AMONG CAPITALIST NATIONS .......................................................................................................................................................49

AFFIRMATIVE EVIDENCE

50

INCOME INEQUALITY GIVES A DISPROPORTIONATE POLITICAL VOICE TO THE RICH ......................50 THE WEALTHY HAVE DISPROPORTIONATE POLITICAL POWER BECAUSE THEY VOTE AND DONATE SUBSTANTIAL SUMS OF MONEY, AND BECAUSE MANY ELECTED LEADERS ARE WEALTHY......................................................................................................................................................51 WITH THE DECLINE OF UNIONS AND THE INCREASINGLY EFFECTIVE INFLUENCE OF BUSINESS INTERESTS, POLITICAL POWER IS LIKELY TO CONCENTRATE IN THE HANDS OF THE RICH ..........52 INCOME INEQUALITY UNDERMINES FORMAL POLITICAL EQUALITY ...................................................53 THE WEALTHY HAVE DISPROPORTIONATE POLITICAL POWER ...........................................................54 LESS AFFLUENT AMERICANS TEND TO ENGAGE IN LESS POLITICAL PARTICIPATION .....................55 DONATIONS TO POLITICAL CANDIDATES AND PARTIES IS A FORM OF PARTICIPATION ALMOST EXCLUSIVELY PRACTICED BY THE WEALTHY.........................................................................................56 PROGRAMS DESIGNED TO REDRESS ECONOMIC INEQUALITY HAVE HISTORICALLY IMPROVED POLITICAL PARTICIPATION ........................................................................................................................57 MANY CONTEMPORARY LIBERALS DEFEND SOCIAL INSTITUTIONS DESIGNED TO EMPOWER THE ECONOMICALLY DISADVANTAGED ...........................................................................................................58 IT IS MORALLY ARBITRARY TO ALLOW SOCIAL GOODS TO BE DISTRIBUTED BY THOSE ARRANGEMENTS ARISING SPONTANEOUSLY THROUGH THE EXERCISE OF OUR NATURAL LIBERTIES.....................................................................................................................................................59 LIBERAL ATTEMPTS TO SECURE EQUALITY OF OPPORTUNITY AND SO GIVE PEOPLE WHAT THEY DESERVE STILL FALL VICTIM TO MORAL ARBITRARINESS..................................................................60

11PF4-Income Disparities www.victorybriefs.com

Page 3 of 141

JUSTICE REQUIRES REDRESSING ARBITRARY INEQUALITY ................................................................61 THE NATURAL DISTRIBUTION OF TALENT IS NEITHER JUST NOR UNJUST; HOW INSTITUTIONS DEAL WITH IT IS ...........................................................................................................................................62 POSSESSING CHARACTER IS MORALLY ARBITRARY; IT CANNOT JUSTIFY SUPERIOR ENTITLEMENTS ............................................................................................................................................63 THE EXERCISE OF VIRTUES LIKE EFFORT IS A MORALLY ARBITRARY ENDOWMENT AND SO CANNOT CREATE A JUST ENTITLMENT....................................................................................................64 THE ABSENSE OF INDIVIDUAL DESERT CREATES A PRESUMPTION IN FAVOR OF REGARDING TALENTS AS A COMMON ASSET ...............................................................................................................65 INCOME INEQUALITY UNDERMINES THE BASIC POLITICAL VALUE OF FAIRNESS .............................66 ECONOMIC INEQUALITY UNDERMINES COMMUNITY SOLIDARITY .......................................................67 ECONOMIC STRATIFICATION CAN UNDERMINE SOCIAL COHESION....................................................68 PUBLIC PROGRAMS TO HELP ALLEVIATE INEQUALITY SOLIDIFY COMMUNITY SOLIDARITY ...........69 GOVERNMENT ACTIONS SYMBOLICALLY AFFIRM THE VALUES OF SOLIDARITY AND HUMAN DIGNITY ........................................................................................................................................................70 VOLUNTARY CONTRIBUTIONS TO HELP ALLEVIATE INEQUALITY ARE INSUFFICIENT TO SYMBOLICALLY EXPRESS THE VALUE OF COMMUNITY SOLIDARITY ..................................................71 THE SUPER-RICH ARE UNLIKELY TO SUPPORT SPENDING FOR THE COMMON GOOD ....................72 CONCENTRATION OF WEALTH UNDERMINES COLLECTIVE ACTION FOR THE COMMON GOOD .....73 ECONOMIC INEQUALITY HAS SUBSTANTIAL SOCIAL COSTS ................................................................74 ECONOMIC INEQUALITY PREVENTS SOCIAL COOPERATION TO PROTECT THE ENVIRONMENT ....75 THE CHOICES PEOPLE MAKE IN THE FREE MARKET ARE NOT ALWAYS FREE..................................76 THE BELIEF THAT THE WEALTHY ARE MORE PRODUCTIVE FOR SOCIETY IS UNTRUE ....................77 IF ALL WE CARE ABOUT IS THE SIZE OF THE ECONOMY RATHER THAN INCOME DISTRIBUTION, WE CREATE ECONOMIC AND POLITICAL DISTORTIONS THAT ARE DAMAGING IN THE LONG RUN 78 CONCENTRATION OF WEALTH AMONG THE TOP 1% IS LARGELY DUE TO GOVERNMENT MANIPULATION ............................................................................................................................................79 INEQUALITY IS BAD FOR ECONOMIC EFFICIENCY BECAUSE IT UNDERMINES COOPERATION BETWEEN CAPITAL AND LABOR ................................................................................................................80 AMERICANS FAIL TO ADVOCATE FOR MORE ECONOMIC EQUALITY BECAUSE THEY MISPERCEIVE EXISTING WEALTH DISPARITIES AND OVERESTIMATE SOCIAL MOBILITY ..........................................81 SOCIAL MOBILITY WOULD MITIGATE CONCERNS ABOUT INCOME INEQUALITY, BUT WE FAIL TO TAKE ADEQUATE MEASURES TO ASSURE SUCH MOBILITY .................................................................82 SOCIAL MOBILITY IS HAMPERED BY THE CONCENTRATION OF WEALTH AND POWER, FOR EXAMPLE BY THE REDUCTION OR ELIMINATION OF THE ESTATE TAX ...............................................83 A VARIETY OF INCOME DISTRIBUTION PROGRAMS HELP TO BOLSTER THE MIDDLE CLASS AND THUS IMPROVE SOCIAL MOBILITY ............................................................................................................84 LACK OF SOCIAL MOBILITY DRIVES THE PUSH FOR SOCIAL PROGRAMS TO ENSURE EQUALITY OF OPPORTUNITY .............................................................................................................................................85 THERE ARE A VARIETY OF POLICY OPTIONS AVAILABLE TO REDRESS INCOME INEQUAILTY........86 ECONOMIC INEQUALITY IS AT AN ALL TIME HIGH, THOUGH PEOPLE DISAGREE ON WHAT THE IDEAL WEALTH DISTRIBUTION IS ..............................................................................................................87 WITHOUT KNOWING THEIR PLACE IN SOCIETY, PEOPLE TEND TO PREFER A WEALTH DISTRIBUTION THAT IS MUCH MORE EGALITARIAN THAN THE UNITED STATES ...............................88 PEOPLE UNDERESTIMATE THE WEALTH GAP IN THE UNITED STATES AND WOULD DESIGN A MORE EGALITARIAN SOCIETY ...................................................................................................................89

11PF4-Income Disparities www.victorybriefs.com

Page 4 of 141

DESPITE SOME DIFFERENCES IN DEMOGRAPHIC GROUPS, THERE IS WIDE CONSENSUS THAT SOCIETY SHOULD BE MORE EGALITARIAN BUT THAT SOME INEQUALITY IS APPROPRIATE ..........90 BECAUSE WEALTH HAS DECREASING MARGINAL UTILITY, MORE EGALITARIAN DISTRIBUTIONS WILL INCREASE THE AMOUNT OF AGGREGATE HAPPINESS................................................................91 THERE IS DRAMATIC AND PROGRESSIVE INCOME INEQUALITY IN THE UNITED STATES ................92

NEGATIVE EVIDENCE

93

WE NATURALLY BENEFIT FROM THE UNEQUAL ENDOWMENTS OF NATURE; IT IS NOT GOVERNMENTS JOB TO TRY TO FIX THEM ...........................................................................................93 A FREE MARKET SYSTEM RESPECTS THE FACT THAT PEOPLE ARE RESPONSIBLE FOR THEIR CHOICES.......................................................................................................................................................94 EXTENDED SOCIETY CANNOT BE CENTRALLY PLANNED .....................................................................95 PRIVATE PROPERTY FACILITATES COORDINATION IN A SOCIETY WHERE EACH OF US ONLY HAS PARTIAL KNOWLEDGE, AND THUS FACILITATES SERVING THE NEEDS OF DISTANT OTHERS .......96 OUR INSTINCTUAL ALTRUISM ACTUALLY HARMS THOSE IN NEED .....................................................97 EFFICIENT MARKETS MAKE EVERYONE BETTER OFF ...........................................................................98 CENTRAL PLANNING IS BOUND TO FAIL ..................................................................................................99 SOME UTILITARIANS OBJECT TO REDISTRIBUTION OF WEALTH ON THE GROUNDS THAT IT DISINCENTIVES PRODUCTIVITY ..............................................................................................................100 DEVELOPED DEMOCRACIES ARE FACING A CRISIS IN THE LONG-TERM FISCAL SUSTAINABILITY OF SOCIAL PROGRAMS ............................................................................................................................101 INCREASING ECONOMIC EFFICIENCY IN CONSUMER GOODS MARKETS HAS BENEFITED LESS AFFLUENT AMERICANS MORE THAN THE RICH ....................................................................................102 ECONOMIC INEQUALITY HAS BEEN DRIVEN BY GROWTH IN THE FINANCIAL SECTOR, NOT MARKET FAILURE ......................................................................................................................................103 TRANSFERING WEALTH HAS SUBSTANTIAL TRANSACTION COSTS ..................................................104 A PROGRESSIVE INCOME TAX CREATES COMPLEXITY BECAUSE OF PRIVATE TAX EVASION STRATEGIES ..............................................................................................................................................105 FLAT TAXES REMOVE THE IMPRECISION IN DECIDING HOW STEEPLY PROGRESSIVE THE INCOME TAX SHOULD BE ........................................................................................................................................106 HIGH TAX RATES KILL ECONOMIC GROWTH .........................................................................................107 IN A COMPLICATED TAX SYSTEM DEISGNED TO REDISTRIBUTE WEALTH, THE AFFLUENT ARE ABLE TO EMPLOY SOPHISTICATED TAX AVOIDANCE STRATEGIES...................................................108 REDISTRIBUTIVE TAX SYSTEMS CAN BE UNDERMINED WHEN SYSTEMIC INEFFICIENCIES COMPROMISE THE TAX BASE ON WHICH THE SYSTEM DEPENDS ....................................................109 THE THEORETICAL BASIS OF LIBERAL DEMOCRACY REQUIRES PROTECTION OF THE RIGHT TO PROPERTY .................................................................................................................................................110 LIBERTARIANS OBJECT TO REDISTRIBUTION OF WEALTH ON THE GROUNDS THAT DOING SO VIOLATES A FUNDAMENTAL RIGHT TO LIBERTY ..................................................................................111 LIBERTARIANS OPPOSE REDISTRIBUTION OF WEALTH OR INCOME IN ANY FORM ........................112 LIBERTARIANS ARGUE THAT THERE IS NOTHING INHERENTLY WRONG WITH ECONOMIC INEQUALITY SO LONG AS WEALTH IS PROCURED LEGITIMATELY ....................................................113 ONE IS ENTITLED TO PROPERTY IF IT WAS JUSTLY OBTAINED AND JUSTLY TRANSFERRED ......114 TRYING TO CONSTRUCT A PARTICULAR PATTERN OF THE DISTRIBUTION OF GOODS FAILES TO REALIZE THE HISTORICAL CHARACTER OF ENTITLEMENTS ..............................................................115 OUR INTUITIONS TELL US THAT PEOPLE ARE ENTITLED TO WHAT THEY EARN THROUGH FAIR TRANSFERS ...............................................................................................................................................116

11PF4-Income Disparities www.victorybriefs.com

Page 5 of 141

DISTRIBUTING RESOURCES TO ATTEMPT TO ACHIEVE SOME DESIRABLE PATTERN REQUIRES CONSTANT REGULATION OF PEOPLES CHOICES ...............................................................................117 TAXATION IS ON PAR WITH FORCED LABOR .........................................................................................118 IT IS ILLEGITIMATE TO SEIZE A PERSONS PROPERTY TO HELP THE NEEDY ..................................119 REDISTRIBUTION OF WEALTH IS THE EQUIVALENT OF GIVING PEOPLE A PROPERTY RIGHT IN EACH OTHERS LABOR .............................................................................................................................120 TO CLAIM THAT ALL ENTITLEMENTS ARE MORALLY ARBITRARY UNDERMINES HUMAN WORTH .121 REDISTRIBUTION OF WEALTH THINKS OF PEOPLES NATURAL ABILITIES AS THE COMMON ASSETS OF THE COMMUNITY..................................................................................................................122 THE ARBITRARINESS OF NATURAL ENDOWMENTS DOES NOT LOGICALLY IMPLY THAT THESE ENDOWMENTS ARE COMMUNAL PROPERTY ........................................................................................123 SOCIAL PROGRAMES THAT REDISTRIBUTE WEALTH ARE OFTEN OPPOSED ON GROUNDS THAT THEY FOSTER DEPENDENCE ON THE STATE .......................................................................................124 ENTITLEMENT PROGRAMS CAUSE UNSUSTAINABLE POLITICAL PATRONAGE ...............................125 PICKETT AND WILKINSONS STUDY OF ECONOMIC INEQUALITY HAS SERIOUS METHODOLOGICAL PROBLEMS .................................................................................................................................................126 SINGLE TIME-POINT INDICATORS OF INEQUALITY FAIL TO CAPTURE SOCIAL MOBILITY ACROSS GENERATIONS ...........................................................................................................................................127 INCOME GAINS HAVE ACTUALLY BEEN SHARED MUCH MORE EQUALLY THAN MANY BELIEVE ...128 AFTER APPROPRIATE STATISTICAL ADJUSTMENT, STUDIES SHOW MEDIAN HOUSEHOLD INCOME HAS GONE UP SUBSTANTIALLY ..............................................................................................................129 MEASURES OF INCOME INEQUALITY FAIL TO CAPTURE THE INCREASING VALUE OF GOVERNMENT SERVICES AND BENEFICIAL TAX TREATMENT ...........................................................130 IN THE LAST FEW DECADES QUALITY OF LIFE HAS IMPROVED SUBSTANTIALLY FOR ALL AMERICANS................................................................................................................................................131 INEQUALITY IN WELLBEING IS SHARPLY DOWN IN THE LAST CENTURY, EVEN IF ECONOMIC INEQUALITY IS GOING UP ........................................................................................................................132 DEMOGRAPHIC CHANGE ACCOUNTS FOR MORE ECONOMIC INEQUALITY .....................................133 INCOME INEQUALITY OUTSIDE THE TOP 1% HAS NOT RISEN DRAMATICALLY, AND GROWTH IN THE TOP 1% IS MOSTLY TIED TO PERFORMANCE PAY .......................................................................134 INCOME INEQUALITY IS DRAMATICALLY INFLATED BY THE EVOLVING TAX TREATMENT OF CERTAIN KINDS OF WEALTH ...................................................................................................................135 STATISTICS SHOWING INCREASED INCOME INEQUALITY ARE DISTORTED BY THE FACT THAT DECREASED MARGINAL TAX RATES IN RECENT YEARS HAVE CAUSED THE WEALTHY TO REPORT MORE INCOME THAN THEY OTHERWISE WOULD.................................................................................136 EVIDENCE SUGGESTING INCREASING INCOME INEQUALITY IS INCONSISTENT WITH MOST DATA .....................................................................................................................................................................137 EVIDENCE FOR INCREASING INCOME INEQUALITY HAS BEEN SUBSTANTIALLY DISTORTED .......138 NEW POVERTY METRICS ARE HUGELY DISTORTED ............................................................................139 WE ARE LESS CONCERNED WITH ECONOMIC INEQALITY AS SUCH THAN WE ARE THAT THE SYSTEM ALLOWS SOME PEOPLE TO GET RICH UNDESERVEDLY .....................................................140 EMBRACING FAMILY VALUES CAUSE ECONOMIC CHOICES WHICH EXACERBATE INEQUALITY BUT WHICH ARE NONETHELESS MORALLY PRAISEWORTHY .....................................................................141

11PF4-Income Disparities www.victorybriefs.com

Page 6 of 141

Topic Analysis by Jessica Bailey


In the early days of the Occupy Wall Street movement, most political analysts dismissed the protesters as disorganized and without a central objective. As days turned into weeks, however, and the movement spread to other cities and even other countries, one could not deny that there was something about the protesters message that was gaining popularity. The movements we are the 99% slogan is a reference to the extraordinary wealth gap between the richest 1% and the rest of us. This message and the reaction to it encapsulate the issues at the heart of the December resolution. Although the gap between the very rich and everyone else is nothing new, it has grown dramatically since the 1990s, and is the highest it has been in over a hundred years.
1

The resolution couldnt be more timely. In early November, the U.S. government unveiled a new poverty measure meant to correct inadequacies of the official poverty measure. Under the new measure, based on 2010 Census data an estimated 16% of Americans live in poverty. The nation faces high unemployment, disappearing manufacturing jobs, a widening education achievement gap, a depressed housing market, shrinking wages, and a host of other woes. These are real, measurable issues that affect real people. They have had an impact on the cultural fabric of the country. We face: concerns about economic polarization, the widening divide between the haves and the have-nots, and the increasingly unequal distribution at the high end, with the richest one tenth of one percent of Americans making seven percent of the income. Red America versus Blue America, a clash of values so strong that liberals can no longer talk to conservatives.
4 3 2

I bring up all of this because it is important to understand that even the most objective judges will have a hard time separating their personal convictions from the debate. Moreover, most
Gelman, Andrew. Economic Divisions and Political Polarization in Red and Blue America. Pathways, Summer 2011, p. 3-6. (This publication is produced by the Stanford Center for the Study of Poverty and Inequality, and is available for free at the Centers website, http://stanford.edu/group/scsp 2 An excellent summary of the poverty measure can be found in Pam Fesslers story New Measure Shows Higher Poverty Rate in U.S. on All Things Considered, 11/7/2011. To listen to the story, go to www.npr.org and search poverty measure, where you can also read the transcript online. 3 In practical terms, this has an important implication: you will need to have very current research. The last five or so years of recession combined with the release of 2010 Census data means that any empirical evidence older than 2007 will not represent current income disparities. 4 Gelman, previously cited, p. 3
1

11PF4-Income Disparities www.victorybriefs.com


5

Page 7 of 141

Americans think that income is much more equally dispersed than it is, meaning that the debaters have a lot of work to do in terms of clarifying what current income disparities actually are. The wording of the resolution further complicates it, because the essential terms will be viewed differently by the two sides (i.e., the Pro and Con will identify different democratic ideals). Thus it is very important to conduct substantive research and use it in round. Given the complexity of the resolution, it makes sense to spend a little time now discussing how the terms may be interpreted by each side.

1. Unpacking the Resolution There are four terms in the resolution that modify the essential question it poses in a significant way: A. In the United States B. Current income disparities C. Threaten D. Democratic ideals The phrase in the United States is significant for two reasons: first, it defines the current income disparities we are looking at, and second, it determines the nature of democratic ideals. To the first point, the impoverished in the United States are still the richest poor in the world. Although there is abject poverty in the U.S., we are not dealing with the kind of gaps that are found in many parts of the developing world. To the second point, limiting the resolution to the U.S. has a huge impact on what we mean by democratic ideals, which well get to momentarily. Limiting the debate to current income disparities seems to imply that some level of inequality may be tolerable. Disparities in themselves are not in violation of democratic ideals, meaning that the presence of an income gap is not in itself undemocratic. The Pro then has a complicated task it is no doubt easier to advocate the Pro when you accept some level of inequality. On the other hand, this requires the Pro to be able to specify what the cutoff point is when the level of income disparity moves from acceptable to unacceptable. To be even more specific, income is different from wealth.

Gilson, David & Carolyn Perot. Its the Inequality, Stupid. Mother Jones, March/April 2011. Available online at http://motherjones.com/print/99036

11PF4-Income Disparities www.victorybriefs.com We also need to distinguish wealth from income. Income is what people earn from work, but also from dividends, interest, and any rents or royalties that are paid to them on properties they own. In theory, those who own a great deal of wealth may or may not have high incomes, depending on the returns they receive from their wealth, but in reality those at the very top of the wealth distribution usually have the most income.
6

Page 8 of 141

Disparities further implies that it is not poverty that is intolerable, but the inequality arising from it. I suspect that many Pro teams may try to blur the distinction, as it is significantly easier to prove poverty is a threat to democracy than income disparity. Disparity is also a more neutral term than inequality, which (particularly in debate) has a negative connotation. The fact that the gap has grown: doesnt mean that the average family is worse off than a generation ago; more people own homes, go to college, drive reliable cars, and have access to sophisticated health care than ever before. But while the average family has done well, the very rich have done much, much better.
7

We first need to discuss democratic ideals before addressing what it means to threaten them. Because the resolution specifies the United States, were looking at a very particular set of democratic ideals, which may or may not be shared by other democratic countries. An obvious ideal for the Pro is equality. Equality, however, is such a general term that it wont provide much focus for the debate. Specifying a kind of equality, such as equality of people before the law, will be much more helpful. Other ideals (discussed in detail below) include the right of individuals to equal representation in the government, opportunity for personal advancement (pursuing the American dream), equal protection of rights of expression (I didnt intend that to rhyme), like freedom of speech, domestic tranquility, and the right to assemble in groups to petition the government for change.

6 Domhoff, William G. (Professor of Sociology) Wealth, Income and Power Who Rules America? November, 2011. (online) http://sociology.ucsc.edu/whorulesamerica/power/wealth.html 7 Berliner, Uri. Haves and Have-Nots: Income Inequality in America. National Public Radio, Feb. 5, 2007. Online: http://www.npr.org/templates/story/story.php?storyId=7180618 (Note: I realize that I said above not to use any empirical data older than 2007, but am now citing an 07 article. The claim that the quotation makes remains true given the current evidence

11PF4-Income Disparities www.victorybriefs.com

Page 9 of 141

I think the U.S. specific nature of democratic ideals is most apparent on the Con. Watch a cable news channel any of them and it immediately becomes apparent that capitalism is a deeply held value for many Americans. This goes farther than simply emphasizing individual freedom to do what one wants with ones property; it places value on an economic system in addition to the people who act within it. It also places value on businesses, entrepreneurship, and innovation, which the Con can argue are threatened by the Pro. The ultimate justification for capitalism is the ideal of individual liberty, including rights of property and ownership. Behind the ideal of a right to property is a notion of merit the American ideal that anyone can pull themselves up by their boot straps if they work hard enough. Whichever side youre on, I urge you to be as specific as possible when describing the kind of democratic ideals youre focusing on, much more specific than I have been. Some debaters may be tempted to boil the debate down to one of equality vs. liberty, which is clearly understandable. I fear that doing so, however, will result in dueling rhetoric about this great country, which already has its place in Presidential debates. Importantly, in addition to arguing that income disparities do or do not threaten democratic ideals, you will need to be justifying the democratic ideals you appeal to - i.e. explain why they are democratic ideals. We turn to the Pro to talk about what it means to threaten democratic ideals.

2. Pro: Current Income Inequalities Threaten Democratic Ideals The Pro may argue that current income disparities threaten peoples opportunity for personal advancement. Earlier I referred to this as the American dream, but what this really encompasses are things like home ownership, access to quality education, and well paying jobs. Income disparities have arisen because jobs held by those on the lower end of the economic spectrum have been disappearing. Technology and outsourcing have eliminated many manufacturing jobs. Many of these jobs were well paying, as in the auto industry (especially for union-represented employees). The elimination of these jobs was especially problematic because the few jobs that are being created in our economy demand a skill set or type of training beyond the reach of the people laid off: The widening gulf between the haves and have-nots has been a consistent trend for a generation or more. Economists largely agree about the primary underlying reasons. New technology has made many jobs obsolete, while creating dramatic opportunities for wealth in computers, finance and media and

11PF4-Income Disparities www.victorybriefs.com entertainment. Global competition has done the same. As middle-class, assembly-line jobs vanish, and routine white-collar work gets outsourced overseas, the value of education and special skills rises. The power of unions continues to decline.
8

Page 10 of 141

This explains why the disparities exist. Remember that the debate isnt about whether poverty threatens democratic ideals; its about inequality of income, so its important to identify what the consequences of these changes in our economy are. One way to do this is to discuss the impact of disparities on citizens ability to participate in their democracy. Campaign finance reforms havent succeeded in reducing the weathys greater influence in politics. Combine with this the declining influence of labor unions (brought on by both political changes and the simple elimination of jobs), and the ability of those on the lower end of the economic spectrum to influence the government diminishes even further. The Pro could argue that the diminished access to participation in democracy is a rights violation itself in a society based on the principle of equality before the law. Even if the forms of democracy remain, they may become meaningless. Its all too easy to see how we may become a country in which the big rewards are reserved for people with the right connections; in which ordinary people see little hope of advancement; in which political involvement seems pointless, because in the end the interests of the elite always get served.
9

Income inequalities have deepened the polarization in American politics. Those on the right accuse the left of engaging in class warfare when they discuss the gap between the rich and poor, but the Pro may argue that if we entrench the economic divide, the wealthy will be able to insulate themselves from the problems of society. Money doesnt just buy political influence; it can buy green space for a neighborhood, attract high-end commerce to the community, fund good schools and community organizations, and so forth. Meanwhile, as incomes decrease in the middle and lower classes, voters become less willing to fund local initiatives, government service programs, and, ironically, anti-poverty measures.
10

8 9

Berliner, previously cited Paul Krugman, For Richer, New York Times Magazine, October 20, 2002, http://query.nytimes.com/gst/fullpage.html?res=9505EFD9113AF933A15753C1A9649C8B63. 10 Domhoff, previously cited

11PF4-Income Disparities www.victorybriefs.com

Page 11 of 141

This brings us to an important question: do we do anything to stop the disparities from threatening democracy, and if so, does the Pro have to specify what it is? I think the answer to both questions is a qualified yes, otherwise we wouldnt regard them as threats. It isnt necessary or possible for the Pro to provide a plan to decrease income disparities, but I think the Pro has to acknowledge that voting for them will require the government to do something about the disparities. Whether this means creating initiatives to give education to people who are laid off so they can get another job, creating government jobs, or providing greater funding for social programs, the Pro cant avoid the argument that this requires some form of wealth redistribution, even if its simply taking the tax dollars from wealthy people to fund programs for the poor. This redistribution of wealth is the jumping off point for the Con: without it there isnt much clash to the debate

3. Con: Current Income Disparities Do Not Threaten Democracy The most straightforward advocacy for the Con is to challenge the idea that income inequalities are as bad as the Pro would have us believe. The Cato Institute, a think tank devoted to promoting free market economics, argues that income inequality is a false measure of the gap between the rich and the poor because of how it accounts for income: However, there have been large changes in U.S. tax rules over time that have made a dramatic difference on what is reported as income on individual tax returns. Tax changes induced thousands of businesses to switch from filing under the corporate tax system to filing under the individual tax system. Corporate executives switched from accepting stock options taxed as capital gains to nonqualified stock options taxed as salaries. The huge growth in tax-favored savings plans, such as 401(k)s, has resulted in billions of dollars of investment income disappearing from tax returns. Meanwhile, studies of inequality that are based on tax return data usually exclude transfer payments, which results in exaggerating the shares of income received by those at the top by ignoring growing amounts of income at the bottom.
11

11 Reynolds, Alan. Has U.S. Income Inequality Really Increased? Cato Institute Policy Analysis, no. 586 Jan. 8, 2007. Online: http://www.cato.org/pub_display.php?pub_id=6880

11PF4-Income Disparities www.victorybriefs.com

Page 12 of 141

The Con could also argue that even if the gap between those with very high incomes and those with very low incomes is large, the standard of living of those on the low end of the economic spectrum does not impose great suffering: more than 70 percent of Americans under the official poverty line own at least one car. Despite a vast difference in price, the difference between driving a used Hyundai Elantra and a new Jaguar XJ is practically undetectable compared with the difference between motoring and hoofing it. A similar compression has occurred for food, clothing, and shelter.
12

This challenges the notion that income inequalities threaten democratic ideals. Another way to debate the Con would be to argue that the Pro has it wrong about what democratic ideals are at stake. The presence of income inequalities obviously violates the ideal of equality. So the Con can either choose to accept the ideals that the Pro identifies and then fight somewhat of an uphill battle of explaining that current income inequalities are not enough of a violation to constitute a threat, or they can argue that voting Pro means violating other, more important ideals. If the Con argues that the Pro entails some form of wealth redistribution to correct the income inequalities, they can argue that a host of democratic ideals are violated. If we redistribute wealth, were violating the property rights of people who own it. The right to do what one wishes with ones stuff is one of the founding ideals of our government and is an extension of the individual right to liberty: Prosperity and property rights are inextricably linked. The importance of having well-defined and strongly protected property rights is now widely recognized among economists and policymakers. A private property system gives individuals the exclusive right to use their resources as they see fit. That dominion over what is theirs leads property users to take full account of all the benefits and costs of employing those resources in a particular manner. The process of weighing costs and benefits produces what economists call efficient outcomes. That translates into higher standards of living for all.
13

Wilkinson, Will. Thinking Clearly about Economic Inequality. Cato Institute Policy Analysis, no. 640, July 14, 2009. http://www.cato.org/pubs/pas/pa640.pdf 13 ODriscoll, Gerald Jr. & Hoskins, W. Lee. Property Rights: The Key to Economic Development. Cato Institute Policy Analysis. Aug. 7, 2003, no. 482. Online: http://www.cato.org/pub_display.php?pub_id=1341

12

11PF4-Income Disparities www.victorybriefs.com

Page 13 of 141

The Con may argue that in violating individual property rights, efforts to correct income inequalities undermine ideals of innovation and entrepreneurship. If you do, you will need to justify these claims with data that shows people stop innovating or starting businesses when their property rights are violated or when wealth is redistributed. Capitalism is a particularly American ideal that trying to correct income inequalities would. As I said above, this ideal places value on the system of a free market economy itself. Connected to this is the ideal of limited government, both of which the Con may argue are threatened if we vote Pro. Underlying all of these ideals is the notion of merit, that people earn and therefore deserve their income. This is also a point where the distinction between income and wealth becomes important: things like inheritances can be said to be part of an individuals overall wealth, where income by its definition refers to what people earn for their labor. Professor of Economics John V.C. Nye explains how our current income disparities are consistent with our idea of merit: Consider the recent housing crisis. Say that two families start out in the early 90s with similar incomes and jobs. But family A saves and invests prudently and conservatively, limiting family spending, sometimes in unfashionable ways (e.g. children are subject to the mockery of school fashionistas or colleagues deride parents for not eating out enough), and chooses to rent accommodations till 2009 when the crisis makes it possible to buy a home cheaply and with low interest. They now have reasonably high net worth and are quite well off. Family B does not save much, borrows and spends on disposable consumer goods, and buys a house in 2005 with no money down and a subprime variable rate. The market crashes. They have difficulty making the monthly payments which have been adjusted upwards and they cannot refinance. In 2009 our measures tell us that family A is worth 10 times more than family B and the latter is in danger of losing their house to foreclosure. The raw fact of income or (in this case) wealth inequality will tell us nothing of how A and B got to the current situation nor guide

11PF4-Income Disparities www.victorybriefs.com us in deciding whether it is ethical to tax A to fund housing assistance for B.
14

Page 14 of 141

Although there are families out there like family B, Nye grossly oversimplifies the income disparities in the U.S. It assumes that family A and family B both start out with the same resources and access to things like, say, financial management education. In essence, the Pro will want to point out that this presupposes that these labor transactions are fair. Consider the disparity in income between a janitor and, say, Kobe Bryant. Now imagine a world with no janitors in it, and a world with no Kobe Bryants in it. Which would you rather live in? If you can give up Kobe Bryant in exchange for clean and safe facilities, then you have to acknowledge that there is a difference between the monetary value of a persons labor and the value of their labor to society.

4. Final Thoughts Because the issue of income inequality is so polarizing, exercise caution when conducting research and be aware of the agendas or leanings of your sources. For example, the Stanford Center for the Study of Poverty and Inequality has great research and makes some wellsubstantiated empirical claims. It is, however, an organization dedicated to ending economic inequality. Also be aware of what the data actually says. You can access the 2010 Census data specifically on income for free online at http://www.census.gov/hhes/www/income/income.html Good luck with your research and position development. This resolution should challenge you to back up ethical, philosophical claims with data, combining in my opinion the most interesting of all possible debate worlds.

14

Nye, John V.C. What Does Income Inequality Tell Us? The Conversation, Oct. 22, 2009. http://www.catounbound.org/2009/10/22/john-v-c-nye/what-does-income-inequality-tell-us/

11PF4-Income Disparities www.victorybriefs.com

Page 15 of 141

Topic Analysis by Ben Lewis


Is the level of inequality unacceptable for a society committed to democracy? Does the government need to take action, even redistribute? With the public backlash to the earnings of those on Wall Street following financial collapse in 2008, and now with the more recent global Occupy movement, the level of income inequality in the United States has generated intense outcry, but little sustained action. This topic is both timely and controversial, both heated and politically polarizing. Many judges will come into the round with strong opinions already, so debaters cannot take anything for granted. Instead, they will need to be sensitive, tactful, and honest to ensure they do not alienate judges, and their need for deep, credible warrants will be even higher than usual. With this topic, debaters have an opportunity to learn about one of the most contentious issues in American politics today. Prepare to step into the fray.

Background Its no secret that income inequality in the United States is high and rising. The Gini index, which measures inequality on a scale from 0 to 1 multiplied by 100, with 0 representing exactly equal income and 1 representing all income in the hands of one person, indicates that the United States has one of the largest inequalities for post-industrial societies. The American Gini coefficient is between 40-45. More significantly, this rise in inequality has not been distributed equally. The incomes of different parts of society have not simply stretched out so that the highest rung simply receives a higher income while the overall percentage of new income for each rung remains the same. Instead, American inequality is uniquely winner-take-all, with a very small slice of the population becoming dramatically richer and the rest largely holding steady.15 For example, the share of pretax income earned by the richest 1 percent of the U.S. population
16

has increased

from around 8 percent in 1974 to more than 18 percent in 2007. Including gains from financial institutions, the share of the top 1 percent rises to 24 percent.17 Greater controversy surrounds the primary drivers behind this dramatic shift in income inequality. For some, economic factors best explain the rise. At the top, changes in information technology have opened up opportunities for the rich to mobilize their resources because success in areas such as entertainment, media, and sports now mainly depends on the ability to reach a wide audience. And in the information age, specialized skills and sophisticated education were valued
Hacker, Jacob S. and Pierson, Paul. 2010. Winner-Take-All Politics: Public Policy, Political Organization, and the Precipitous Rise of Top Incomes in the United States. Politics & Society. 38: 152. 16 Ibid 17 Saez, Emmanuel. August 2009. Striking it Richer: The Evolution of Top Incomes in the United States (Update with 2007 estimates).
15

11PF4-Income Disparities www.victorybriefs.com

Page 16 of 141

more, raising the bar for entry into the fields that return the highest incomes. Moreover, the combination of computers, globalization, and new financial instruments enabled the rich to make millions instantly. For the rest, the pressure from machines, immigration, and outsourcing depressed the value of low-skill jobs, while the collapse of traditional unions lowered the value even further. The income gap grew because advanced markets simply work in favor of the rich. However, for others, the rise in inequality is best explained by political factors, particularly the absence of government intervention. For these writers, markets will always reinforce the strength of the rich because the rich have the resources to best take advantage of the markets. Thus, to contain inequality, governments must set policies that either shift the way the market operates, like subsidies, financial regulation, and antitrust legislation, or mitigate unequal market gains, like progressive taxation. In this view, the expanding income gap and concentration of wealth is a result of an ideological shift against governmental activism and a greasing of the wheels in favor of policies that favor the rich. While both economic and political factors likely drive increased income inequality, a proper understanding of the primary driving forces of inequality is important to understand how inequality operates and to predict what actions would be necessary to close the income gap. The response to income inequality caused by financial globalization looks very different than income inequality caused by a shift in political ideologies, so debaters must be aware of the causal assumption behind each argument for and against inequality. At the same time, the American democracy has responded to changes in technology, media, and means of organization. Political campaigns increasingly focus on advertising and organizing a sustained grassroots advocacy for several years at a time. With the Internet, directing social media is just as important as receiving a newspaper endorsement. Meanwhile, particularly after Citizens United vs. the Federal Election Commission, corporations have increased power to support particular candidates. And once in office, lobbyists attack policymakers at every turn in attempts to shape government attention away from issues that threaten their interest group. While the Constitution has not changed much, the way American democracy operates reflects modern realities about information exchange, corporate connections, and media. Two takeaways. First, for a topic that examines the role of concentrated economic power in American government, debaters must understand where the two fields intersect and where they branch off. Not all governmental policies are influenced by inequality, and not all features of income inequality have political implications. Good cases will focus on the middle ground, where income inequality consistently influences the impressionable parts of democratic governance. Second, the economic and political drivers of income inequality today are time-sensitive: the income inequality today looks different than the income inequality of even thirty years ago

11PF4-Income Disparities www.victorybriefs.com

Page 17 of 141

because some of the main modern causes are things that had not even been invented in 1980. As such, debaters must be sensitive to the way inequality and democracy operate in reality. If debaters tie arguments to the particular processes by which inequality expands or democracy contracts, those arguments will be much stronger.

In the United States Debaters must keep in mind the unique features of American inequality and democracy, like the rare concentration of wealth, information-based economy, and representative system. The literature on economic inequality and democracy spans many years and many nations, but not all arguments will make sense if applied to the unique American context. For example, while inequality may yield widespread violence and social distrust in underdeveloped countries with weak institutions and preexisting tensions, inequality will likely not energize widespread civil strife in the US. At a more basic level, this wording restricts the topic to an existing, established representative democracy that has survived for more than two hundred years. While there is a robust literature on how inequality influences democratization and shapes the institutions that follow, that literature is not very applicable to the question of whether an existing democracy with large income inequalities survives or thrives. Similarly, it is difficult to imagine a world where American democracy collapses, regardless of the level of income inequality. The real debate will instead most likely center on the quality of representation provided or the overall level of political freedom.

Current Income Inequalities The only thing to note here is that income inequality is not the same thing as wealth inequality or economic inequality. While many authors will use the terms interchangeably, debaters should not. The difference between wealth inequality and income inequality is that wealth includes all of the things a person owns, their assets, in addition to their income.18 Thus, income inequality does not include things like homes, small business, savings accounts, or outstanding debt. As two people with the same income may have very different qualities of life if one of them owns a house while the other does not, debaters should be wary about equating income with economic wellbeing. Be careful to make arguments about how income inequality has political implications rather than wealth inequality or general economic inequality.

18

Edward Wolff. May 2003. The Wealth Divide: The Growing Gap in the United States Between the Rich and the Rest. The Multinational Monitor. Volume 24, Number 5.

11PF4-Income Disparities www.victorybriefs.com Threatens

Page 18 of 141

While debaters can probably substitute threatens for the word harms in most rounds, asking whether current inequality is good or bad for democracy, the term threatens does seem to lower the burden of proof for the pro. Rather than having to prove that current economic inequalities actively undermine or contradict democratic ideals, the pro simply has to prove that current income inequalities pose a significant risk to the fulfillment of democratic principles.

Democratic Ideals Many rounds will come down to who wins the debate about the relationship between income inequality and American government, but just as many will come down to the question of what constitute democratic ideals. The question of what democratic ideals entail is critical to the question of whether a certain economic makeup violates those ideals. Is democracy processoriented or substantive? If substantive, what are the outcomes that democracies should promote? Can an outcome or environment threaten democratic ideals, or are democratic institutions committed to accepting any outcome that their constituents vote for? Are democratic ideals dependent on the system of government, or are they independent of any institution? At one end of the spectrum, authors argue that democratic ideals are purely about representing the views of constituents. Democracy is just a machine that takes the desired inputs from voters and translates those views into outputs, usually policies. Leaders have no place to disregard voters, no matter what they vote for, even if that means the transition away from democracy in the future. At the other end of the spectrum, authors argue that democratic ideals are about broad social goals. Democracies should promote equality, freedom, accountability, and the provision of the basic needs necessary to access political society. If constituents vote for policies that increase inequality or reduce liberty, governments have an obligation to disregard those votes in order to protect democratic values. Somewhere in the middle, authors advocate that while democracies have no obligation to promote widespread social emancipation, they have to promote the values that ensure continued representation. For these authors, while democracies do not have to promote public health or social equality, they must ensure political equality, public accountability, and universal suffrage. Finally, there is the question of whether democratic ideals differ based on location. Do democratic ideals applied to the United States differ from democratic ideals applied to other nations? Does the nature of democratic ideals themselves change based on the surrounding political institution? Does it matter that the Declaration of Independence affirms, All men are created equal? Does it

11PF4-Income Disparities www.victorybriefs.com


th

Page 19 of 141

matter that the 14 Amendment of the Constitution affirms that states cannot abridge the privileges or immunities of citizens of the United States? Does American democracy have any obligations outside of their founding, binding documents? Or are democratic ideals in the United States more closely connected to the modern beliefs of the US citizenry rather than a document written in 1787? Think carefully about the conception of democratic ideals you choose to defend. The broad substantive view will make many more aspects of income inequality relevant to the debate, while the strict process view may shift the focus of the debate entirely. In many cases, the debate over the nature of democratic ideals may be the difference between who wins and who loses the round.

Key Question of the Debate With all of this in mind, debaters should reframe the question of the debate, isolating the relevant features of income inequality and defending a conception of democratic ideals. Thus, for some, the question of the debate is whether income concentration in the hands of a few strengthens or weakens representation. For others, the question is simply whether a majority of citizens in the United States endorse or oppose the current economic makeup in the United States. Or maybe the relevant question is whether current income differentials are constitutionally permissible. Or maybe the topic questions whether the ideological shift against governmental intervention in the market promotes or undermines the provision of basic needs. There are many possibilities. Debaters should choose the framing that makes most sense to them, and then prepare against all of the other frames. Regardless of the broad frame they choose, debaters must still deal with two unresolved issues. First, does the Pro commit society or the government to taking action designed to reduce income inequality? If they are, then the Con may be able to prove that current income inequalities do not threaten democratic ideals because a world with income inequality is preferable to a world with, for example, progressive taxation. However, if the Pro simply has to defend that a hypothetical, ideal democracy could not tolerate current levels of income inequality, then the Con could not win by attacking progressive taxation. Under that view, both current income inequalities and progressive taxation may threaten democratic ideals. Second, does the topic question whether higher inequality or lower inequality is preferable for democracy, or does the Pro have to prove that the inequality has passed a certain threshold of danger so that current income inequality now threatens American democracy? In other words,

11PF4-Income Disparities www.victorybriefs.com

Page 20 of 141

can the Con win if the Pro proves that current income inequality has negative effects on American democracy, but the system still functions pretty well, or even just okay? Do the harms have to pass a certain threshold to threaten democratic ideals? The answers on these two issues will color the burdens for both sides.

Positions Available to Both Sides Both sides can take the position that current income inequality threatens or does not threaten democratic ideals because the American public opposes or supports the current levels of income inequality. Taking the view that the American public shapes American democratic ideals, debaters can attempt to prove public support or opposition using survey data, voting patterns, and local policies. The critical, most difficult part of the case will be proving that the data sufficiently reflects the views of the American public. However, strategic debaters who run this type of position, particularly on the Con side, will leverage arguments about the corruption in modern American government to support the proposition that survey data and local voting patterns best reflect public beliefs.

Affirmative Positions There are four main types of positions for the Pro on this topic. The first is the argument that large income inequalities beget political inequalities. Defending a conception of democratic ideals that focuses on popular sovereignty, political equality, and representation, this position argues that the concentration of income gives the rich undue political influence during campaigns and policymaking. For campaigns that must spend millions on advertising and organization, funding is crucial. The more that political actors depend on a select few to get elected, the more leverage that those select few have to determine who wins. Just as importantly, subsequent policies are more likely to reflect the interests of the rich because the rich can mobilize interest groups to set the agenda and direct congressional debate. As Paul Pierson and Jacob Hacker put it, political struggles involve drawn-out conflicts in multiple arenas, extremely complicated issues where only full-time, well-trained participants are likely to be effective, and stakes that can easily reach hundreds of billions of dollars. Inevitably, organized groups are crucial actors, and usually the crucial actors, in these struggles.19 Thus, the very rich have undue influence over the political process. Current income inequality threatens democratic ideals because a very small minority that does not need protection has significant control over political outcomes.

19

Ibid 1

11PF4-Income Disparities www.victorybriefs.com

Page 21 of 141

The stronger version of this position will not simply claim that the very rich control the political process, but that the views of the very rich substantially differ from the views of other Americans. While some authors claim that the very rich are heterogeneous enough that their views reflect the views of other Americans,20 most concede that the rich have different views about certain policies, particularly those that affect their economic strength.21 On topics like executive compensation, union power, and taxes, policies are more likely to reflect the views of the very rich rather than the general public. The second is the position that large income inequalities depress general political participation. In conjunction with the claim that the very rich have undue influence, some authors claim that the very economic makeup of large income inequality depresses general political activity.22 Due to the influence of the very rich, issues that matter to the poor get excluded from political channels. In response, more people view their participation as futile, and fewer people participate. Or maybe aside from the political influence of the very rich, the rich use their influence directly to depress general participation. In order to increase their relative influence on policymaking, maybe the rich mobilize their resources to close off normal political channels and increase political apathy. To some extent, very rich are more politically active, but is that due to increase education and political awareness, or are their motives more malicious? One issue that this position will have to deal with is the voluntary nature of democratic participation. Is it against democratic ideals for people to choose not to vote or to entrust their views to others, or is the choice not to vote itself a political expression? Since these positions conclude there is lower political participation, they must be careful to also prove that lower overall political participation is undemocratic. The third position for the Pro focuses on the democratic deficit of certain policies enacted because of rich influence. These positions take the literature on executive compensation, taxes, and union deterioration, which is well connected to the influence of the very rich, and they argue that changes in these policy areas have particular undemocratic effects. For example, some people will argue, taking a broad substantive view of democratic ideals, that the influence of the very rich led to the financial deregulation that played a part in global economic collapse. Regardless of your position on democratic ideals, global economic collapse is probably undemocratic.
Stuart N. Soroka and Christopher Wlezien. Spring 2008. [McGill, Temple]. On the Limits to Inequality in Representation. Political Science and Politics 21 Larry M. Bartels. August 2005. [Department of Politics and Woodrow Wilson School of Public and International Affairs, Princeton University] Economic Inequality and Political Representation. Chapter 7. The Unsustainable American State, ed. Lawrence R. Jacobs. 22 Frederick Solt. January 21, 2010. Does Economic Inequality Depress Electoral Participation? Testing the Schattschneider Hypothesis. Political Behavior, 32: 285-301.
20

11PF4-Income Disparities www.victorybriefs.com

Page 22 of 141

Stronger versions of these positions will prove that with greater income equality, there would have been greater financial regulations in place. But the difficulty is proving the counterfactual. Since economic and political situations change daily, there are a ton of variables involved in any broad policy area. The difficulty is isolating that damaging policy to the income gap.

Negative Positions For the Con, there are three main strategies. First, the Con can defend a concept of democratic ideals that cannot be threatened by current income inequality. For example, if the Con proves that democratic ideals in the United States are simply the words outlined in the Constitution, it will be very difficult to prove that permitting the income gap violates some part of the Constitution. This position will give the Con the opportunity to layer the debate so that they can win by proving their conception of democratic ideals or by proving that higher income inequality strengthens the Pros conception of democratic ideals. Second, the Con can affirm the importance of economic freedom as a political bulwark. Authors like Friedrich Hayek, Milton Friedman, and Ludwig Von Mises have all written about how economic freedom strengthens political freedom. Hayek, for example, worried that governments who had control over the economic well being of their constituents would eventually continue to expand their economic control because governments are not very good at predicting the future, and they would eventually use that economic leverage to affirm political ideologies. On the other side, the rich themselves may be a direct check on governmental oppression. Since support of the very rich is so important to get elected and enact new policies, the very rich can use their economic leverage to check the government when leaders overstep their authority. With these largely theoretical authors, be careful to apply them specifically to the modern political context. Third, the Con can fight fire with fire; they can defend that large income inequality strengthens democracy by increasing certain public services. For example, many authors argue that while federal policies may reflect the interests of the rich, greater income inequality produces local and state policies that are beneficial to the poor and the middle class. Sean Corcoran and William Evans studied the fiscal support for public education between 1970 and 2000 and came to the conclusion that higher income inequality induces higher local spending on public education.23 Be careful to weigh the importance of things like increased local expenditures against the Pro arguments about undue representation.

23

Sean P. Corcoran and William N. Evans. August 2008. Income Inequality, the Median Voter, and the Support for Public Education.

11PF4-Income Disparities www.victorybriefs.com Concluding Remarks

Page 23 of 141

The debate over the income gap lies at the intersection of modern economics and the American political theory. Its a debate that comes up again and again throughout world history, and debaters have a great opportunity with this topic to learn about how the grand debate applies to America today. Right now, hundreds of thousands of people are willing to give up parts of their lives to occupy one side of the debate, and hundreds of thousands of people are willing to express their public disapproval of the movement against income inequality. You are doing yourself a disservice if you do not take advantage of this opportunity to find out why, and to formulate some opinions of your own. Good luck and happy hunting!

11PF4-Income Disparities www.victorybriefs.com

Page 24 of 141

Topic Analysis by Bill Neesen


Introduction: The United States is once again seeing huge increases in the gap between the haves and the have-nots. Nowhere is this clearer than in the different incomes that people are making. Given this fact the topic forces us to look at whether or not we think this is okay to let happen in a democracy. While this topic looks simple at first glance there are many terms that will need to be defined because there is not a common definition that most people would agree on or there are ways that people could define a word that would favor one team. Once you have defined the problematic terms, the debate becomes pretty straightforward. At first glance I thought that the Affirmative side was going to be easy and that the negative team was in for a long month. While I still feel that the affirmative has some easy proofs about the harm the income disparity does to people in the United States, I think that the negative will have strong ground to defend that it is not a threat. This paper is meant to tell you my opinion of what is possible with this topic and to provide you with an overview of what everyone can do with this topic so that you can choose what you want to do and will not be shocked by the other team. So I am going to go through the definitions or framing of the resolution and show you what that will allow teams to do this year that might be a problem, then go over some affirmative ways to prove it true, and finally look at all the wonderful creative ways that the negative can approach this debate.

Framing of the resolution: To start framing this resolution first you must realize that it is a fact debate which always scares many people in the debate community because they are not sure how to evaluate the facts to figure out who is right. I mean very few debates come down to one side winning all the facts. So this debate is going to be about how you evaluate the facts to determine who is winning the debate. There are three ways that I have seen people try to prioritize or evaluate the facts that each team offers. The first is to look at who has more fact on their side. Most people like this because the judge can just count the facts and see who wins the debate, so it is easy for them and everyone in the debate round does not have to work very hard to see who wins. Sadly this rarely works out that way and one team is always on the side of less facts. Also it can lead to a dumb debate were all facts are equal and we just must have more, no matter how dumb they are. The second way is to look at the quality of the fact, but this means that you have to try to decide whose facts are better which is very difficult. If you have ever watched a presidential debate you know what I am talking about and if not go watch one and see how they try to decide whose facts

11PF4-Income Disparities www.victorybriefs.com

Page 25 of 141

are more important. You will see a lot of security versus liberty discussions and of course both sides think their facts are more important or right. The last way to look at it is to see whose facts are more impactful to us as a nation and a people. This is a little like policy debate but hopefully with more realistic impacts. Overall every debate team, before going to a tournament, should have a plan how they are going to win the debate by how they are going to tell the judge to evaluate the facts. The first word that is a problem is the word threaten. This word is going to be a problem because the big question will be to threaten someone do you really have to be a threat. There are theories of law that say to be a threat it must be realistic. So could I threaten you if I pointed a toy gun at you? While this is an exaggerated example wither the disparity can raise to a level of real threat not just an infringement on the ideals. So what I am saying is that some teams will define threat in a way that means the affirmative will have to prove that the disparity can destroy the ideals as a whole. On the other side of this debate is the idea that you just have to express a threat even if you do not have the means to really carry it out. A good example of this is Iran and the constant threats that are made against us by them. They cannot destroy the United States but they make the statements anyway. Sadly this is technical debate that both sides can be right about so I would suggest to be ready for it. The term Democratic ideal in the resolution is going to be a difficult one because what people think are the democratic ideals varies widely by who you read and what they think about current politics. This is made more difficult because a lot of the people who thought up democracy and its ideals were very old and did not have the same ideals we have in the United States today. So the question is how do we define it because it could be very different depending upon the culture of the person that you are defining it from. Currently in America both political parties will cry that something is violating our democratic ideals. The one place that most Americans would agree that democratic ideals are present is the Declaration of Independence and The Bills of Rights. These are supposed to be a clear application of the ideals and therefore a good place to see how they get defined in practice. There are a few other ideas about what make up democratic ideals that appear in a few different places. The first and most important is the basic humans rights are seen as a democratic idea (though a lot of what we would call a human right is in the Declaration of Independence and the Bill of Rights). There are also people in America who think that the ability to succeed and make it on their own is part of our ideals. In the end there are lots more people who define it in very specific ways depending upon what they are writing about. I would suggest that if you do not like this type of debate to stick to the one clear area (Bill of Rights and Declaration of independence) and try to avoid the mess.

11PF4-Income Disparities www.victorybriefs.com Affirmative

Page 26 of 141

As I have said this looks like the easy side of the debate because all you have to do is prove that the disparity is a threat to democratic ideals, but it will take more work than you might think. That said, this discussion has very few surprises in it because the affirmative team will be fairly predictable. To counter that predictability the affirmative has the choice of either defending one strong reason or having a case of a couple of different reasons. I would go with a few different reasons that individually prove the resolution true. To do this one would just have to have different observations about why the resolution is true and each one would independently prove the resolution true. Then if you lose any one of them it is not that big of a problem because it makes you more flexible later in the debate. Voter apathy is one of the strongest reasons that it threatens democracy. There are many people who link the recent increase in the income disparity to an increase in voter apathy. It is no stretch to say that if people are not voting then it threatens the democratic ideals. Most negative teams will say that if you choose not to vote that is a right in a democratic society. While this is true I think the affirmative can get out of this by pointing out the ideals say everyone votes not that people get to choose not to, so even if it does not threaten democracy it does threaten the ideals it is based upon. The other response that the affirmative can make to this argument is that people do not have a choice in our system because of a few different reasons: 1. Voting occurs when people are working and the poor have a harder time taking off of work or even getting to the local polls. 2. When people know that their vote will not make a difference for any reason then them not voting is not a really a choice because even if they do it does nothing. Equal representation is the second good argument that I feel an affirmative team can make. It is simple because most people who believe in democracy (another way to define ideals is what people believe about democracy) think all people should get an equal say or vote. If one person because of an economic status has a more impactful vote then that is a problem because it leaves some people out. Cleary if you are poor you cannot give a bunch of money to a candidate and get what you want, so it threatens those ideals in practice because it makes many peoples vote not count and that is a threat to true democracy. Also the fact that money spent is not heavily correlated to who wins means the rich get more say because their money is what wins the election. Another good argument is about equal opportunity in the United States. While we all like to think that anyone can make it big in the United States that is not always the case. This is even more true as the disparity in wealth gets bigger because money buys a better education, better tutors

11PF4-Income Disparities www.victorybriefs.com

Page 27 of 141

and many other things that set some people up for success. So if currently that is true then the democratic ideals that we founded this country on are not happening and have been threatened if not outright killed. If it really is a democratic ideal for all of us to have an equal shot at success then it seems like a very easy way to win is to prove that this is currently not true for most of us. Also if you run this argument you should prepare for them to talk about the Bill Gates of the world, but remember he started Microsoft in his familys garage. Not everyone has that and most of the people who get held up as a success story did not start in abject poverty and even if they did someone helped them and it was most likely not the system. Since some people believe that human rights are key to democratic ideals all you have to do is to point out were human rights are being violated because of the income disparity. I think one good place that you can go would be the medical issues and poverty in the United States. It is pretty clear that people in the middle and bottom of the economic ladder do not get the same medical coverage that the well-off people get. There are some authors who feel medical coverage is a basic human right. There are others that are less clear but could also be good arguments. If you wanted to make one of these arguments you can just go to places like Amnesty International and see where they say we are violating human rights in the United States and then link that to income disparity from the work there. The one crazy place I have found that would be fun to talk about is how income disparity affects crime. There are some studies that link the increase in income disparity to the increase in crime. To link this to a threatening of democratic values is not much of a leap. Here are a few reasons why: 1. When people are given no choice to support themselves it shows how unequal the society is and that we all do not have the same chances. 2. When you are a felon you cannot vote. Taking away the right to vote disenfranchises people and takes away a basic right and if they are doing this to support themselves how did they have a choice. The negative will focus on how crime is a choice and you are never forced to do it. One last idea that an affirmative could add to their case is the idea that the economic problems are causing people not to believe in democracy. This would seem to be a simple argument because as peoples situation gets worse in America people are starting to look at other solutions that most Americans would not call democratic. The reason this argument is pretty good is that you do not have to defend what the actual effect on the practice of democracy but rather you can just say that people will stop believing in it, like we currently do in America, and that will be the death of them because if no one believes in them then they do not exist. This could also be included with a more traditional affirmative as its own contention.

11PF4-Income Disparities www.victorybriefs.com

Page 28 of 141

Overall the affirmative can find many reasons why it is bad and wrong but remember to focus on how that threatens democratic values and not just why it is bad in itself.

Negative The negative gets a lot of very diverse strategies for this topic from denying that it is a threat to saying that they have already been dead for many years so that current disparities cannot affect it. This is why I like the negative in this debate better because I like to have a lot of choices about how to approach the debate and the affirmative on this topic does not have it. I am going to start with the most normal and easy to do and move to the more difficult ones at the end. Every negative team should attack the affirmatives case as soon as they can. I feel that the affirmative will have a hard time proving the threat to democracy part and also they will most likely try to define democratic values in a way that is very favorable to them. Another way to attack their arguments on case is to make an argument that says they are wrong and that the problems that they are blaming on the disparity are really just caused by other things so they affirmative did not prove the resolution true. A simple way to look at this argument is through an example of voting. They are going to say that the disparities mean that less people vote. If you can prove that dirty politics is the reason for it instead of the disparity then they have nothing to go on. Most of the arguments that most affirmative teams are going to make will have alternative causalities that apply to them. The last argument that you can make on case is a little more fun for me as a teacher of logic and critical thinking. You can run that the burden that they have to meet to win the debate is to prove the threat. I would argue that the threat is a causal argument because to prove it is a real threat you would have to be able to prove that it can hurt or kill the democratic ideals we all hold. To prove a causal argument, in formal logic, you need to prove that it is necessary and sufficient cause. A necessary cause is one that has to be present for the cause to happen. A sufficient cause is one that can by itself cause the problem to happen. A true causal argument in formal logic would need both of them and that is not an easy thing to prove and I think that for a lot of judges explaining something like logic would go over well. No matter what arguments you choose to make a few good arguments here will make the debate much harder for them to win because they want you to focus on why you do not think it is a threat and not poking holes in their ideas. At the end of the day they want to weigh their ideas untouched against yours, but when you do not let them have the full weight of their argument they are going to have a harder time. Once you have attacked the case then you can do any of the strategies that I talk about below.

11PF4-Income Disparities www.victorybriefs.com

Page 29 of 141

I think the first strategy you could go with is to simply say that while the income disparity is unfair, it actually makes people protest and do things that are at the heart of democratic ideals. So all you would have to show is the economic problems that are happening now are causing things like the wall street group (and the others that have sprung off it) to go out and become more political and therefore increasing democracy. Speaking your mind is at the heart of it and protests like the original tea party were not allowed. I like this strategy because all you have to do is prove that your increase in participation is more important to democracy than the other teams proof of how it hurts it. The hard part of this strategy is the other team is going to say it can both threaten and help and all they have to do is to prove it threatens it. I think you can argue that this is not the case and that you look at the effect on the whole and not just one side. The second strategy that is good is to say that harms that the case is talking about are not part of democratic ideals because they have always existed in the United States. This argument is interesting because you can say that the United States is the embodiment of the ideals of democracy so if it was not included in the United States it was not part of our ideals. Simply if the founders thought that it was okay to let the problems be there, then they were not an idea that is central to democratic ideals. The first place that you can point out is within the area of income disparity. There has always been a disparity that has existed in the United States; there were huge income disparities, and many people had no income because as slaves they were considered property and therefore did not need to be paid. Most of the leaders of that early time were wealthy land owners and everyone else was simply trying to survive. Another area that is great is voting. I think a lot of affirmative teams will make arguments about it decreasing voting or making one persons vote not count. Since not everyone had a right to vote in the United States, they felt that in a democracy not everyone should have a right to vote. Also the founders did not leave the results of the election up to the people. Instead they had the Electoral College because they felt that most people would not understand who to vote for and that decision was better made by the elite, who were like them. These two arguments will take out most of the good affirmative arguments that they have to make. Also, it goes nicely with the argument that since democratic ideals are hard to define and no one agrees we should just look at what is the case in the United States. I think that when the resolution says in the United States it gives you more ground to say that this is the form of democracy we should look at. I also think you could give ground later in the debate and argue that even if what the affirmative says is true your history lesson proves that things are getting better not worse, so they do not prove the word current in the resolution. Another solid approach is to use the definition of threat to your favor. If you take a legal definition of the word threat that says there has to be a reality behind the supposed threat, i.e. you have to

11PF4-Income Disparities www.victorybriefs.com

Page 30 of 141

believe that it can happen, so you can say that for a bunch of different reasons there is no way the current problem could raise to the level of a realistic threat to the ideals even if it hurts the idea a little. Combine this with the idea that a threat is something to do great harm and you have a winning argument. I mean for it to be a threat it would have to destroy or really hurt it and how can just economic disparities do that by themselves. From here the ideas for what the negative could do get a little more outlandish and may not work all the time depending upon your judge and what you think they could handle. I think you could argue that since the resolution says ideals and not something more concrete like democratic practices that it is impossible to threaten a democratic ideal in any way for a few reasons. 1. An idea is not reality so what happens in reality is very different than an ideal we hold to. 2. Ideals change over time depending on many different things. I think saying that democratic ideals will change over time means that they cannot be threatened as a whole. 3. Even in practice when ideals fail people still believe in them, I think that Marxism is a good example something with good ideals that has never resulted in anything productive as a government. Another strategy that you could take is to just say that they are not really threatened currently by the disparity but that it will become a threat in the future. So this argument would say that people in America for the most part still have all the things that the democratic ideals guarantee and they therefore believe in those ideals. In the future if the disparity gets too big between the rich and poor it will threaten them for a bunch of reasons. If people are not getting more of their health care in the future or if it will cause enough people to not vote then our democratic values will be gone. One fun way to really make it hard on the affirmative is to say that democratic ideals are already dead. Yes folks you can look at all the bad things that happen in the United States that have hurt the idea of democratic ideas and say that this mean they are dead or have been corrupted so much that they are gone. You can focus on the power of the companies and how the political action committees effect elections and laws or you could also look at how democracy was meant to be for smaller governments and not the huge thing we have - when people were voting for people they knew and being a politician was not a profession. One of the last ones that I am going to mention is that these ideals never existed and if you think about it that is true. If we have had slaves and economic inequity the whole existence of the United States then they were already killed or more likely never real existed at all. To win this argument all you have to do is look at all the things that the United States has done for years that goes against the perfect ideals of democracy. This means that we never followed them and I think

11PF4-Income Disparities www.victorybriefs.com

Page 31 of 141

you could argue that they never existed. The benefit to this is that all the harms of the affirmative can then be used against them. All you have to do is prove they have always been there. On the negative you can focus on one of them or you can run a few of them but make sure they are complementary and do not contradict. Also remember you should attack case with what you have as soon as you can because no one wants their case attacked. Remember that you have a lot of ground because they have to prove threat and what democratic ideals are. Overall this will be a fun topic to debate. Just remember that this is just a starter of ideas that I have on the topic and it is not by any way exhaustive and should be looked upon as a place to start and help you frame the debate before you go out and explore the literature yourself.

11PF4-Income Disparities www.victorybriefs.com

Page 32 of 141

Topic Analysis by Adam Torson


This months resolution implicates a variety of interesting issues that have been brought to center stage by the Occupy Wall Street movement. Below I will discuss some interpretive intricacies and describe core positions for both the Pro and the Con.

Interpretation A. In the United States Limiting the topic to the United States has two major implications. First, it excludes issues of international economic inequality. This means that arguments about how inequality affects emerging democracies, the dramatic nature of global poverty, and institutions of international democratic government are beyond the scope of the topic. While Occupy Wall Street has to some extent become an international movement, be careful of literature which deals with topics beyond domestic income inequality. Second, limiting the topic to the United States means that particular historical, social, and governmental contexts are relevant. For example, negatives will likely propound the theoretical advantages of laissez faire economic policies. In response affirmatives can suggest that even if in the abstract income inequality can come about through legitimate mechanisms (like voluntary exchange), the history of the United States suggests that many social and economic advantages were procured through unjust practices (e.g. slavery, discrimination in the workplace, etc.). Similarly, affirmatives might argue that in principle wealth redistribution best meets citizens fair expectations. Negatives can respond that the particular wealth redistribution programs employed in the United States have had and continue to have major problems (e.g. government healthcare spending is a major driver of the federal deficit and debt problems). B. Current The most obvious import of this phrase is to limit the resolution to the contemporary phenomenon of income inequality. The question is not whether income inequality in the abstract or at some point in history threatened democratic ideals, but rather whether it is doing so right now. This means that both sides must acknowledge the unique nature and almost unprecedented magnitude of current income inequality, as well as the myriad of social institutions and programs that attempts to remedy the most troubling social effects of such inequality.

11PF4-Income Disparities www.victorybriefs.com

Page 33 of 141

C. Income Disparities The first thing to note about this phrase is that there is a difference between wealth disparity and income disparity. Wealth would generally denote a persons net worth the total value of her assets minus her liabilities (debts). Income, on the other hand, generally denotes how much revenue a person is bringing in in a given time period. For example, if I have $1 Million, and next year it will earn 10% interest (and I have no other income), my income would be $100,000, and my net worth at the end of the year would be $1.1 Million (ignoring taxes). A person who makes $100,000 a year but only spends $50,000 will have a net worth that goes up over time. A person who makes $100,000 a year but spends all $100,000 will have a net worth that stays the same, even though both individuals have the same income. What is the practical effect of all of this? A few things to note: First, as the last example indicates, income is more of a snapshot than net worth. It tells you how a given individual did in a given period of time, but not necessarily his economic health overall. Obviously a person making $10,890 per year (the current HHS figure for poverty for a single individual) by working two jobs who is unable to save anything at all is not as well off in the long term as a person who earns the same amount of money from the interest on his $150,000 investment portfolio. Second, it should be noted that there are different statistical metrics for different kinds of inequality. You will no doubt be doing lots of statistical comparison in your rounds make sure that you and your opponents are talking about the same thing. Obviously these two measures are related people with more wealth tend to have more income but remembering the difference will often be helpful. The second thing to note about this phrase is that it defines the conflict area of the resolution. The most obvious interpretation of income disparity is that it refers to the gap between rich people and poor people generally. You will find it defined as a term of art to mean just that. However, there are more particular income disparities in the United States that you should also take note of. For example, it continues to be the case that women and people of color are on average paid less for the same work than white men. This has obvious implications for social justice, although there are a variety of causal factors at work. Similarly, there are income inequalities between professions that some consider disconcerting. Teachers, for example, are generally paid poorly relative to their training and the importance of the social function they serve. Cosmetic surgeons make more than family practice doctors. Finally, there are geographic income disparities. In some regions of the country the average income is substantially higher than others. More locally, it is often the case that the average income of suburban areas is higher than that of urban areas (resulting in

11PF4-Income Disparities www.victorybriefs.com

Page 34 of 141

tax differences that affect school funding, etc.). Again, both occupational and geographic income disparities have a variety of causal factors, and there are proponents on both sides of the argument about whether these disparities are appropriate, but you should definitely be prepared to debate about different types of income disparity. D. Threaten Democratic Ideals 1. What does democracy mean? The most important thing that you can do to improve your argument quality and clarity on this topic is to do meaningful analysis of what democratic ideals are. Democracy is a moniker claimed by people of many different kinds of theoretical and practical political persuasions. For a variety of reasons, democracy is associated with political legitimacy; some international organizations even classify governments on the assumption that democracy is definitionally on the opposite end of a spectrum from authoritarianism. It pays to have a quick refresher course on the basic underlying concepts associated with democracy. After discussing those concepts, I will talk about some competing conceptions of democracy which you might employ in your case positions. Democracy is most directly associated with the concept of popular sovereignty, which suggests that a government or particular government policies are only legitimate if they are based on the approval or consent of the population at large. In other words, the people are the source of legitimacy, not some other religious or doctrinal principle. In practice, democracy usually means some degree of popular participation in government. Going back to ancient Athens, Democracy is associated with the rule of the many in which citizens gather to make decisions directly affecting their collective lives. From here is it useful to divide democracy in to two competing conceptions: institutional and discursive. These categories are broad generalizations, and many different particular theories of democracy fit under each, but this division will give you a useful way of thinking through some of the core questions of what it means to be a democracy or to exhibit democratic ideals. The institutional conception of democracy emphasizes institutional procedures designed to respect and implement popular sovereignty. This would include, for example, assurance of fair voting rights and procedures, a balance of power among branches of government to prevent any from becoming too powerful, and federalism to give citizens multiple avenues to express their preferences. This particular set of institutions is associated with liberal democracy, which drawing on the enlightenment sees democracy as a way to protect the fundamental rights of

11PF4-Income Disparities www.victorybriefs.com

Page 35 of 141

individuals. For this reason it is (somewhat paradoxically) considered democratic to put certain rights beyond the reach of majority decisions, e.g. the right to freedom of speech, the right to due process when accused of a crime, etc. The institutional conception of democracy would also include illiberal democracies e.g. single-party states with a government that acts in the name of the people but which does not necessarily protect political participation. While most citizens of liberal democracies would regard the use of democracy to describe such states as a guise for authoritarian government, there are some theoreticians who see this as a justified extension of democratic theory. The point is to suggest that even where one agrees that democracy is a set of institutional arrangements, there is broad disagreement about what democratic ideals might be. The discursive conception of democracy is less concerned about a set of rigid institutional arrangements. Instead it focuses on the idea that collective decisions should represent the perspectives of all of those who will be affected by the decision. Failing to include people in decisions that affect their lives fails to respect their moral status and thus renders such decisions illegitimate. Furthermore, the idea of discursive democracy suggests that individuals sense of their identities and interests are formed in the democratic process. Institutional conceptions of democracy generally operate on the assumption that people come into political participation with their minds made up; they have certain interests that they want to promote, and they use the mechanisms of government to do so. Discursive democracy, in contrast, adopts the view that the process of debate and discussion can inform our sense of collective identity, shape what we value, and so in an important way determine what interests we will want to advance. When we adopt an enlarged perspective to see the world from the other persons point of view, we are obliged to advance justifications for our choices that are amenable to those perspectives (rather than narrow self-interest). This can mean that policies which systematically exclude people from discussion (or fail to empower them to participate) can be undemocratic even if citizens have their formal rights protected (e.g. the right to vote). 2. Politics, Economics, and Political Economy Another interpretive issue is whether the resolution only cares about the impact income inequality has on the political process, as opposed to economic consequences like GDP growth or decline. If the affirmative says income inequality increases unemployment, can negative say that doesnt matter because the resolution is asking about its effect on democratic ideals,? In the United States we tend to divide politics and economics into separate disciplines. In other places the concept of political economy treats the two as largely inseparable. On the one hand, it seems silly to ignore the economic implications of income disparity. At the same time, if any impact is relevant why would the resolution use the specific term democratic ideals? In practice

11PF4-Income Disparities www.victorybriefs.com

Page 36 of 141

most of your impacts can probably be framed in terms of their political implications, even if they are economic in nature e.g. recession is politically destabilizing, undermines the ability of citizens to meaningfully participate, etc. This is a good idea if you want to avoid this interpretive issue altogether. A related issue is whether the affirmative can take the position that democracy is less important than some other value. For instance, the affirmative team might argue that income inequality is undemocratic because it permits social inequality, but that this inequality is a good thing because it allows for material prosperity through economic growth. In other words, does the affirmative need to defend a change in the status quo to mitigate income inequality? I doubt this approach will occur to many affirmatives, but as a negative debater you should be prepared to force the affirmative to defend particular policies. Some of the best negative ground is criticizing social programs designed to remedy inequality rather than saying that inequality is a good thing in itself. If the affirmative tries to pull this trick, tell the judge that it is unreasonable for the affirmative weasel out of the obvious normative implications of their arguments, which include the types of programs just mentioned.

Affirmative Positions A. Disproportionate Political Influence One of the most straightforward affirmative cases will argue that income disparity gives wealthy citizens disproportionate political influence. This is undemocratic, because democracy ideally values each citizens voice equally. There are several ways this disproportionate influence can come about. First, wealthy citizens can spend money in a variety of ways that influences the outcome of the democratic process. They can make contributions directly to candidates and political parties who are in turn obliged to represent their interests. Given that the ability to spend lots of money in political advertising has an enormous effect on the outcomes of elections, politicians are especially motivated to keep their wealthy contributors happy. Less directly, wealthy individuals can promote particular political positions through nominally non-partisan free speech. For example, they can contribute to advocacy organizations like the NRA or AARP, fund politically driven research organizations like the Hoover Institute and the Center for American Progress, or advertise positions like opposition to abortion or ending the wars in Iraq and Afghanistan. Second, it is increasingly the case that members of elected bodies, in particular the President and members of Congress, are themselves very wealthy. This is the case for a variety of reasons.

11PF4-Income Disparities www.victorybriefs.com

Page 37 of 141

Sometimes personal wealth is used to partially or fully finance campaigns. Individuals who have some degree of economic security can afford to run for office, which while generally high paying is not competitive with the private sector. This gives the wealthy disproportionate influence because elected politicians do not have ready access to the perspectives of marginalized groups. It is hard to appreciate the urgency of being poor and without health insurance if youve never had to face those difficulties yourself. Third, wealthy people often are able to budget time for political participation. A person making minimum wage in most places has to work much more than 40 hours a week. Combine that with caring for a family and otherwise attending to lifes necessities, and there is very little time and energy left over to participate thoroughly in the political process (especially when you believe your participation is futile). Wealth creates the luxury of time and flexibility, which enables more thoroughgoing political participation. B. Social Cohesion Another set of arguments suggest that income disparities undermines the public spiritedness and social cohesion necessary to sustain a democracy, because democracy is dependent on broad based political participation. There are two major ways this can happen. First, many communitarian authors argue that class stratification undermines the institutions that constitute the public sphere. Instead of participating in clubs, local advocacy groups, school board and PTA meetings, etc., people withdraw into their private circle of family and friends. Institutions like public schools, public parks, and institutions of mutual support come under fire, and we lose the ability to cultivate a sense of shared purpose and public spirit. This excess of individualism (as Alexis de Tocqueville described it in the 1830s) threatens the most basic virtue of democracy on the discursive model, which is that our self-identity is constructed around processes of collective deliberation about the common good. Second, many argue that income stratification causes people to advance parochial interests rather than working toward a common good. Instead of the enlarged perspective described above, citizens use government as a means to pursue their narrow self-interest as defined by class, race, sex, religion, or other salient features of their identities. Government comes to be viewed as a vehicle for advancing economic interests or the only means to materially support the poor, and thus becomes merely another cite of political contest rather than a mechanism for accomplishing shared projects and aspirations. This is particularly true when wealthy, powerful

11PF4-Income Disparities www.victorybriefs.com

Page 38 of 141

individuals have inordinate influence, because they obviously have an interest in maintaining the status quo to the detriment of those on the bottom of the social ladder. C. Market Distortions There are reasons to believe that severe income inequality is bad for the economy. First, income inequality may be a symptom of deeper market distortions. For example, economists of the Austrian School argue that government intervention in the economy such as setting artificially low interest rates, bailing out failing firms, and subsidizing politically popular businesses picks certain people as winners and leaves everyone else on the outside. Institutional reforms like a fair tax structure, they argue, would reduce unfair inequality produced by government meddling. On the flip side, some argue that better government regulation to prevent massive institutional failures (like those that precipitated the most recent recession) is the appropriate way to reduce income inequality, because when the economy is depressed it is Main Street that is hurt while Wall Street pulls down huge bonuses and severance packages. In other words, the concentration of wealth and power in the hands of the few will make them incautious even when making choices that endanger the financial lives of literally hundreds of millions of people. Moreover, income disparities may be indicators of market failures of a different type. Above I gave the example of teacher pay, where it appears that the market fails to appropriately value the training and social importance of a particular profession. In many places other public servants like police and firefighters are underpaid relative to the importance of their jobs, the training required, and the danger involved. This seems undemocratic for a variety of reasons, not the least of which is that we fail to value those people who most directly make the day to day workings of our society possible. D. Poverty is Dehumanizing While this topic seems for the most part to focus us on the top 1% of income earners, it is also important to think about people on the opposite end of the spectrum. It is simply unacceptable, this position argues, that in the most affluent society in the history of the world that so many people should live in abject poverty. Poverty is dehumanizing because it makes it difficult to maintain a satisfactory quality of life. The dramatically poor must work an enormous number of hours to stay afloat when jobs are difficult to come by. They have little meaningful opportunity for social advancement or the development of their talents. They are subject to poor working conditions, catastrophic financial crises if they are injured or get sick, and often become

11PF4-Income Disparities www.victorybriefs.com

Page 39 of 141

dependent on public aid to make ends meet. Many argue, for obvious reasons, that this is a degrading condition. If we value everyone equally, as democracies often claim to, we should not

Negative Positions A. Capitalism A major position defending income inequality will be the argument that it is a natural outgrowth of a free market economy. Such an economic system is desirable for a variety of reasons, but most importantly because it maximizes economic productivity which makes everyone better off. While some people are dramatically wealthier than others, some three-quarters of poor people in the United States have air conditioning and a VCR or DVD player, and 97% have a color television. All have access to at least emergency healthcare. All of this is dramatically more luxury than even monarchs have enjoyed through most of history.
24

The fundamentals of this argument go like this. People are paid different amounts of money because what they produce is valued differently in society. Because being a physician requires intelligence and extensive training, the supply of medical services from doctors is relatively low. Because everyone has healthcare needs, the demand for medical services is relatively high. Thus doctors tend to be paid more than the average worker because their services are valued more highly. If we mandated that doctors got the same pay as those guys that spin advertising placards outside of sandwich shops, then everyone would be a sign-twirler and nobody would invest the time and energy to be a doctor, and that would make everyone worse off. In the business context, free markets produce an incredible variety of goods and services. The individuals who invest capital into producing those goods and services take an enormous risk their product might be unpopular and their venture fail, in which case they would lose their investment. People are incented to participate in large scale social projects because of the prospect of large returns. Those who choose to take on less risk are paid less money. This competition for market success also makes everyone better off by making goods and services cheaper and creating greater variety and innovation. The gasoline which powers an enormous portion of our everyday lives has to be found deep in the ground all over the world, extracted at enormous risk and expense, shipped literally across the oceans, refined into gasoline, and delivered to your local pump, where you will pay only about $4.00 a gallon less than you pay for bottled water.

24

Peter Singer [Professor of Bioethics, Princeton University], The Life You Can Save, New York: Random House (2009), p. 8-9.

11PF4-Income Disparities www.victorybriefs.com

Page 40 of 141

Proponents of Capitalism argue not only that income inequality creates the incentives that drive the free market, but also that government attempts to level economic prosperity have disastrous consequences, both economically and politically. On the economic side, proponents of capitalism argue that government attempts to mitigate income inequality create distorted incentives in the economy and therefore decrease productivity, making everyone worse off. For instance, a minimum wage means that businesses cant hire otherwise employable workers to capture efficiency and productivity gains, which means that some people are unemployed rather than earing a lower wage, and everyone else has to pay more money for a smaller variety of products. Steeply progressive taxation makes it less worthwhile to put in extra effort and take on extra risk to earn more money, and therefore decreases productivity. Government rationing of valuable goods produces systemic shortages and surpluses because government cannot capture information as efficiently as the price mechanism of the marketplace. The examples could go on and on. For negatives, it will be important to force affirmatives to say not just that income inequality is bad, but that it is worse than the alternative. The negative can forward convincing arguments that income disparity is the lesser of two evils when compared to extensive government intervention in the economy. On the political side, libertarians argue that extensive efforts to level economic prosperity are inconsistent with individual freedom. Individuals, they argue, have the right to enjoy the fruits of their labor. When government transfers wealth from one person to another, it is the functional equivalent of forced labor. A certain portion of your day is spent working to benefit someone else, and you have no choice in the matter. At the extreme end of the spectrum, libertarians point to failed attempts to centrally plan the economy and the totalitarian aspirations of many governments who have made the attempt. Even without going to that extreme, strong control of income inequality presumptively requires extensive government regulation of individual choices, which libertarians argue negates individuality and the right to pursue ones own legitimate life projects. B. Interest Group Politics Politicians who advocate programs to mitigate income inequality are often accused of engaging in class warfare. While that term is used polemically, the underlying fear is that treating government like an instrument for social groups to struggle over makes politics about serving clients rather than pursuing the public good. Proponents of this position point to the Presidents support of a bailout of American auto manufacturers as an example, suggesting that his position was taken because the union members they employ are major political supporters. Not only does this seem to lose focus on the general good, but it also creates a conception of government which

11PF4-Income Disparities www.victorybriefs.com

Page 41 of 141

the poor cannot possibly win. If government is a competition for resources, the affluent will win witness favorable tax treatment for corporations and wealthy individuals and members of Congress advocating for government subsidies for firms in their districts. This argument says that government should not be about pitting social groups against each other but rather working together to achieve the common good. Further, many worry that the poor become dependent on government services. This can create a cycle of poverty that is difficult to break as economically depressed neighborhoods find capital increasingly difficult to come by and means of social advancement more and more scarce. While there are people who draw benefits to avoid having to work, many people have been unfairly maligned as welfare frauds. You dont need to take assistance under false pretenses to become dependent on government assistance. On the flip side of the economic spectrum, many criticize corporate welfare, favorable tax treatment, government contracts, and subsidies used to keep businesses afloat, as another form of unjust dependency on government aid. Many accuse politicians of functionally paying off supporters by promising continuing provision of government benefits in exchange for political support. C. No Wealth Criterion for Political Rights A third strategy will call into question the degree to which money matters in terms of political influence. There are a whole set of institutional protections in place to ensure that every citizen is entitled to full-throated political participation. The right to vote, the right to speak and write freely, the right to petition the government for a redress of grievances, are all designed to ensure that the workings of democracy are not tied to any arbitrary criterion like social class, race or sex. Income disparity may have some undesirable social consequences, this position might concede, but it cannot undermine the political architecture associated with democracy. D. Income Inequality is Not That Bad While there is a general consensus that income inequality is pretty severe in the United States, that consensus is not universal. There are a number of credible statistics to draw upon which suggest that income inequality is not as severe as generally thought, or that studies suggesting that it is are methodologically flawed. This type of argument is probably not a sufficient case position in itself, but I suspect that it will comprise a substantial portion of negative argumentation. You would do well to be familiar with both sides of this empirical debate.

11PF4-Income Disparities www.victorybriefs.com Conclusion

Page 42 of 141

This topic gives you the opportunity to explore issues and academic literature which you often arent exposed to in school or debate. I encourage you to take advantage of it. While some of the literature is a little dense, learning about the way the economy works is essential for an understanding of public policy and democratic citizenship. I wish everyone the best of luck!

11PF4-Income Disparities www.victorybriefs.com

Page 43 of 141

FRAMEWORK EVIDENCE
INCOME INEQUALITY IS INCREASING BOTH QUANTITATIVELY AND QUALITATIVELY Task Force on Inequality and American Democracy, American Democracy in an Age of Rising Inequality, Published by the American Political Science Association (2004), p. 2-3 But as U.S. society has become more integrated across the previous barriers of race, ethnicity, gender, and other longstanding forms of social exclusion, it has simultaneously experienced growing gaps of income and wealth. Gaps have grown not just between the poor and the rest of society, but also between privileged professionals, managers, and business owners on the one hand, and the middle strata of regular white-collar and blue-collar employees on the other hand. Many middle-class families are just barely staying afloat with two parents working.3 And many African-Americans, Latinos, and women who head families find themselves losing ground. There are signs of increased segregation by, for example, income and race in our public schools.4 Meanwhile, the rich and the super-rich have gotten much more so especially since the mid1970s.5 Indeed, the very richest one percent of Americans has pulled away from not only the poor but also the middle class. Disparities in wealth and income have recently grown more sharply in the United States than in Canada, France, Germany, Italy, and many other advanced industrial democracies. Many kinds of statistics could be cited to document this statement. Figure 1 presents information about income trends for American families compared with families in Britain and France. The proportion of income accruing to the top one-tenth of one percent of families ran along parallel tracks for much of the 20th century. All three countries reduced inequality from the end of World War I through World War II and until the 1960s. But from the mid-1970s on, the United States rapidly diverged from its two allies and became far more unequal. By 1998, the share of income held by the very rich was two or three times higher in the United States than in Britain and France.

11PF4-Income Disparities www.victorybriefs.com

Page 44 of 141

ECONOMIC INEQUALITY IS PARTICULARLY PRONOUNCED BETWEEN RACIAL AND ETHNIC GROUPS Task Force on Inequality and American Democracy, American Democracy in an Age of Rising Inequality, Published by the American Political Science Association (2004), p. 3-4 Disparities are particularly striking when it comes to comparisons across races. Even as the absolute economic circumstances of minorities have improved, the median white household earned 62 percent more income and possessed twelvefold more wealth than the median black household, with nearly two-thirds of black households (61 percent) and half of Hispanic households lacking any financial assets, as compared with only a quarter of their white counterparts. Even young, married, black couples in which both adults work the shining beacons of progress toward racial equality still earn 20 percent less income than their white counterparts and possess a staggering 80 percent less net worth. Living conditions remain fragile for the black middle class, and continue to lag far behind those experienced by their white counterparts. And, of course, the life circumstances of African-Americans who have not reached the middle class are even more precarious.7 The Civil Rights era helped lift the absolute levels of income and wealth enjoyed by African-Americans and Hispanics, but they remain unacceptably far behind white America.

11PF4-Income Disparities www.victorybriefs.com

Page 45 of 141

INCOME INEQUALITY IS NOT THE SAME AS GENERAL ECONOMIC INEQUALITY, AND THE FORMER HAS BECOME MARKEDLY MORE STRATIFIED Tyler Cohen [Prof. of Economics, George Mason University], The Inequality That Matters, The American Interest, January/February 2011, < http://www.the-americaninterest.com/article.cfm?piece=907> The numbers are clear: Income inequality has been rising in the United States, especially at the very top. The data show a big difference between two quite separate issues, namely income growth at the very top of the distribution and greater inequality throughout the distribution. The first trend is much more pronounced than the second, although the two are often confused. When it comes to the first trend, the share of pre-tax income earned by the richest 1 percent of earners has increased from about 8 percent in 1974 to more than 18 percent in 2007. Furthermore, the richest 0.01 percent (the 15,000 or so richest families) had a share of less than 1 percent in 1974 but more than 6 percent of national income in 2007. As noted, those figures are from pre-tax income, so dont look to the George W. Bush tax cuts to explain the pattern. Furthermore, these gains have been sustained and have evolved over many years, rather than coming in one or two small bursts between 1974 and today.1 These numbers have been challenged on the grounds that, since various tax reforms have kicked in, individuals now receive their incomes in different and harder to measure ways, namely through corporate forms, stock options and fringe benefits. Caution is in order, but the overall trend seems robust. Similar broad patterns are indicated by different sources, such as studies of executive compensation. Anecdotal observation suggests extreme and unprecedented returns earned by investment bankers, fired CEOs, J.K. Rowling and Tiger Woods. At the same time, wage growth for the median earner has slowed since 1973. But that slower wage growth has afflicted large numbers of Americans, and it is conceptually distinct from the higher relative share of top income earners. For instance, if you take the 19792005 period, the average incomes of the bottom fifth of households increased only 6 percent while the incomes of the middle quintile rose by 21 percent. Thats a widening of the spread of incomes, but its not so drastic compared to the explosive gains at the very top.

11PF4-Income Disparities www.victorybriefs.com

Page 46 of 141

THIRD WAVE DEMOCRATIC MOVEMENTS COINCIDED WITH THE ASCENDANCE OF NEOLIBERAL ECONOMICS AND SO FOCUSED LESS ON ECONOMIC INEQUALITY Francis Fukuyama [Olivier Nomellini Senior Fellow at the Freeman Spogli Institute for International Studies, Stanford University], Dealing with Inequality, Journal of Democracy Volume 22, Number 3 (July 2011), p. 79-80 These are some of the questions that the articles on Poverty, Inequality, and Democracy in this and earlier issues of the Journal of Democracy seek to address.1 The third wave of democratization that began with the Spanish and Portuguese transitions in the 1970s (and which now may be followed by a fourth wave in the Arab world) consisted largely of political revolutions that sought to put in place regimes whose first objective was to secure electoral democracy and political rights in countries coming out of authoritarian rule. But the third wave also happened to coincide with a conservative trend in economic thinking, during which redistribution and active economic intervention fell out of favor, to be replaced by an emphasis on more classically liberal economic policies. Indeed, the collapse of communism after 1989 led to a rethinking of the entire redistributionist agenda on the Left, since its negative impact on economic growth had become evident for everyone to see. While a number of new democracies put into place a variety of social protections designed to bolster equality, the broader trend in economic thinking from the 1970s through the first decade of the twenty-first century was, if anything, toward support for a reduction in the size of welfare states.

11PF4-Income Disparities www.victorybriefs.com

Page 47 of 141

NEOLIBERAL ECONOMICS HAS A GREATER TOLERANCE FOR INEQUALITY Francis Fukuyama [Olivier Nomellini Senior Fellow at the Freeman Spogli Institute for International Studies, Stanford University], Dealing with Inequality, Journal of Democracy Volume 22, Number 3 (July 2011), p. 80 Neoliberal economics implied a greater tolerance for economic inequality, since many economists argued market competition inevitably meant that value-creating entrepreneurs would be more richly rewarded than others. And indeed, in some advanced democracies such as the United States and Britain, the last three decades of the twentieth century saw increasing de facto inequality. Some have maintained that global inequality also increased during these years, though this assertion is highly contested and dependent on the way one defines and measures inequality. Incomes may have grown more unequal within individual East Asian countries, for example, though per capita incomes in the region closed the gap with Europe and North America. Levels of inequality began to decline somewhat in Latin America during the first decade of the twenty-first century, but overall they remained stubbornly high for the region.2

11PF4-Income Disparities www.victorybriefs.com

Page 48 of 141

FEW PEOPLE BELIEVE THAT EITHER TRUE LAISSEZ FAIRE OR THE PURSUIT OF TOTAL EQUALITY ARE DESIRABLE POLICY OBJECTIVES Francis Fukuyama [Olivier Nomellini Senior Fellow at the Freeman Spogli Institute for International Studies, Stanford University], Dealing with Inequality, Journal of Democracy Volume 22, Number 3 (July 2011), p. 83 The real terrain of debate over the relationship between liberal democracy and socioeconomic equality is less the theoretical one than a more practical or consequentialist argument about the real-world impact both of socioeconomic inequality and of the various types of policy interventions designed to correct it. Few people take a strict Hayekian position against all forms of redistribution or social policy; on the other hand, few today would argue that the single-minded pursuit of economic equality is an appropriate end of public policy. In most liberal democracies, there is a consensus that excessive inequality is a bad thing, but considerable disagreement as to the practicality and likely effects of policies that governments can implement to mitigate it.

11PF4-Income Disparities www.victorybriefs.com

Page 49 of 141

THERE ARE A WIDE VARIETY OF ECONOMIC DISTRIBUTION PATTERNS AMONG CAPITALIST NATIONS Frank Stilwell [Prof. of Political Economy at the University of Sydney], Why bother about economic inequality? Evatt Foundation, 8 July 2003. <http://www.evatt.org.au/papers/whybother-about-economic-inequality.html> Of course, the extent to which the capitalist economy has been tempered by any such egalitarian principles and policies has varied considerably from country to country. One useful comparative measure of economic inequalities is the ratio of the income of the top 10 per cent of households to the poorest 10 per cent of households. According to the latest United Nations figures, this ratio is 12.5:1 for Australia. In other words, the wealthiest 10 per cent of households enjoy incomes on average 12.5 times higher than the poorest 10 per cent of households. This is a below the US ratio of 17:1 but above the UK figure of 10:1. For all the Scandinavian nations the ratio is in the range from 5:1 to 6:1. At the other extreme, the ratio is 42:1 for South Africa and 49:1 for Brazil. So, even within the contours of capitalism, all sorts of distributional patterns exist.

11PF4-Income Disparities www.victorybriefs.com

Page 50 of 141

AFFIRMATIVE EVIDENCE
INCOME INEQUALITY GIVES A DISPROPORTIONATE POLITICAL VOICE TO THE RICH Bradford Plumer [Assistant Web Editor], One Dollar, One Vote, Mother Jones, 11 Apr. 2006, <http://motherjones.com/politics/2006/04/one-dollar-one-vote> How pronounced is inequality in America? Between 1979 and 2003, the income of the richest 1 percent of Americans more than doubled, the income of the middle 15 percent grew by only 15 percent, and the income of the poorest 20 percent barely budged, according to CBO data. By the late 1990s, the richest one percent of Americans households had a third of all wealth in the economy, and took in 60 percent of the country's incomea greater share than at any point since the Great Depression. Incomes in the United States are far more unequal than in other industrialized countries, while mobility, contrary to widespread myth, is hardly much betterif you are born poor in America, you are very likely to stay that way your entire life. In politics, this all matters very much, as the APSA findings reveal. Larry Bartels of Princeton has studied the voting record of the Senate between 1989 and 1994a time, note, when Democrats controlled Congress. He found that Senators were very responsive to the preferences of the upper third of the income spectrum, somewhat less attentive to the middle third, and completely ignored the policy preferences of the poorest third of Americans. In one striking example, Bartels discovered that Senators were only likely to vote for a minimum wage increase if and when their wealthier constituents favored itthe views of those directly affected by the hike had "no discernible impact." Nor is this pattern limited to domestic policy. Lawrence Jacobs of the University of Minnesota and Benjamin Page of Northwestern found that that the foreign policy views of the executive and legislative branches are primarily influenced by business leaders, policy expertswhose think tanks are often funded by businessesand, to a lesser extent, organized labor. Surprisingly, Jacobs and Page found, the views of the broader public have essentially zero impact on the government when it comes to tariffs, treaties, diplomacy, or military action. The political theorist Walter Russell Mead famously argued that "Jacksonian" nationalism in the heartland drove American foreign policy, but the data doesn't back him up. Business still runs the world.

11PF4-Income Disparities www.victorybriefs.com

Page 51 of 141

THE WEALTHY HAVE DISPROPORTIONATE POLITICAL POWER BECAUSE THEY VOTE AND DONATE SUBSTANTIAL SUMS OF MONEY, AND BECAUSE MANY ELECTED LEADERS ARE WEALTHY Bradford Plumer [Assistant Web Editor], One Dollar, One Vote, Mother Jones, 11 Apr. 2006, <http://motherjones.com/politics/2006/04/one-dollar-one-vote> To some extent, these findings are probably a result of the fact that elected officials tend to hail from the upper classes, and so tend to be the sort of people who worry more about the burden the estate tax imposes than, say, food insecurity or too-high heating bills. In 2003, financial records revealed that 40 senators and 123 representatives were millionaires. This shouldn't be surprising. Without publicly-financed elections, it takes a good deal of personal wealth to run for officethe average Senate campaign in 2006 will cost about $10 million, minimum, according to a recent University of Washington study. But that's only the most obvious way economic power begets political power. Consider the fact that wealthy Americans are far more likely to vote: 86 percent of those in families with incomes over $75,000 reported voting in 1990, compared to only 52 percent of those in families with incomes under $15,000. Whether that's because the well-off are more likely to believe that government will work for thema reasonable assumption, evidentlyor because they have more time and opportunity to inform themselves and do their civic duty remains unknown. Nevertheless, those numbers matter. And it's not just voting. People in the $75,000 bracket are much more likely to join a political advocacy group like the NRA or the NAACP (73 percent vs. 29 percent), and much more likely to make campaign contributions (56 percent vs. 6 percent). Indeed, in the 2000 election, 95 percent of those donors making substantial campaign contributions came from households making over $100,000. With some exceptions, political influence generally isnt a straightforward matter of slipping across a few hundreds in exchange for a vote; outright bribery is relatively rare. But those who contribute do have a better chance of ensuring that those candidates sympathetic to their concerns will make it into office. Megacontributors lucky enough to win an audience with a Senator or member of Congress, meanwhile, can't demand specific votes, but they can make sure their concerns are heardwhile, say, advocates for the homeless are left picketing outside the gates. That counts for a lot. George W. Bush once reportedly told the Rev. Jim Wallis, "I don't understand how poor people think." Presumably he doesn't hear much on the subject during face time with key donors.

11PF4-Income Disparities www.victorybriefs.com

Page 52 of 141

WITH THE DECLINE OF UNIONS AND THE INCREASINGLY EFFECTIVE INFLUENCE OF BUSINESS INTERESTS, POLITICAL POWER IS LIKELY TO CONCENTRATE IN THE HANDS OF THE RICH Bradford Plumer [Assistant Web Editor], One Dollar, One Vote, Mother Jones, 11 Apr. 2006, <http://motherjones.com/politics/2006/04/one-dollar-one-vote> Back in the postwar era, the working classes at least had unions to fight for their concerns, some of the time. But thanks to the decline of manufacturing and the assault on organized labor over the past three decades, union density has shriveledfrom 24 percent in 1973 to 12.5 percent today. Meanwhile, in the late 1970s, in response to inflation that was eating away at the wealth of the top 1 percent, "business refined its ability to act as a class, submerging competitive instincts in favor of joint, cooperative action in the legislative arena," as the journalist Thomas Edsall wrote in his 1986 book, The New Politics of Inequality. And it worked. The leveling forces of the 1970s were overturned by the Reagan revolution, and the upper classes did very well from there on out. That dynamic is only likely to intensify. The political scientist Theda Skocpol has noted that there are more well-educated Americans in rapidly-growing professional societies than there are lessereducated workers in trade unions. Absent a countervailing force, the political influence of the wealthiest Americans will continue to expand. On the other side of the Atlantic, European nations have managed to reduce inequality, partly by redistributing wealth through the welfare state. Unless the United States figures out how to do something similaror else find some way to strengthen civic participationthe ideal of "equal citizenship" will continue its long erosion.

11PF4-Income Disparities www.victorybriefs.com INCOME INEQUALITY UNDERMINES FORMAL POLITICAL EQUALITY

Page 53 of 141

Frank Stilwell [Prof. of Political Economy at the University of Sydney], Why bother about economic inequality? Evatt Foundation, 8 July 2003. <http://www.evatt.org.au/papers/whybother-about-economic-inequality.html> Politically, economic inequality is also of concern because it undermines the potential for the full realisation of the democratic ideal. The principle of 'one person one vote' in the political sphere sits uncomfortably with the principle of 'one dollar one vote' in the economic sphere. Moreover, as the liberal economist Arthur Okun has noted, money can buy services that produce, in effect, more or better rights. These are circumstances where the formal equality of rights before the law has an uneasy relationship, to say the least, with the existence of pervasive economic inequalities. The posited connection between capitalism and democracy, which ideologues on the political right assert, looks ever more shaky in circumstances of growing economic inequality.

11PF4-Income Disparities www.victorybriefs.com THE WEALTHY HAVE DISPROPORTIONATE POLITICAL POWER

Page 54 of 141

Task Force on Inequality and American Democracy, American Democracy in an Age of Rising Inequality, Published by the American Political Science Association (2004), p. 1-2 Generations of Americans have worked to equalize citizen voice across lines of income, race, and gender. Today, however, the voices of American citizens are raised and heard unequally. The privileged participate more than others and are increasingly well organized to press their demands on government. Public officials, in turn, are much more responsive to the privileged than to average citizens and the least affluent. Citizens with lower or moderate incomes speak with a whisper that is lost on the ears of inattentive government officials, while the advantaged roar with a clarity and consistency that policy-makers readily hear and routinely follow. The scourge of overt discrimination against African-Americans and women has been replaced by a more subtle but potent threat the growing concentration of the countrys wealth and income in the hands of the few. Progress toward realizing Americas democratic promise can surely be revived. But to counter current trends that undermine inclusion and fairness, we must work together for democratic renewal. Failure to take urgent and concerted steps to expand political participation and enhance democratic responsiveness and failure to use democratic means creatively to temper rising social disparities will surely endanger our longstanding democratic ideals at home and undermine our countrys efforts to spread the hope of equal citizenship abroad.

11PF4-Income Disparities www.victorybriefs.com

Page 55 of 141

LESS AFFLUENT AMERICANS TEND TO ENGAGE IN LESS POLITICAL PARTICIPATION Task Force on Inequality and American Democracy, American Democracy in an Age of Rising Inequality, Published by the American Political Science Association (2004), p. 6-7 Although electoral participation ticks up somewhat when contests are closely fought and parties make extra efforts to mobilize voters, a number of ongoing trends discourage voting and reinforce inequalities in voter turnout. Rising economic inequality may discourage less privileged voters.13 And part of the decline in voting since the early 1970s results from laws in many states that forbid former (as well as current) prisoners from voting, sometimes for their entire lives. Millions of Americans, especially minority men, have been excluded from basic participation in our democracy by such laws.14 Below, we also discuss strategies pursued by the Democratic and Republican parties that place more emphasis on raising money and persuading the already active, rather than mobilizing eligible citizens who do not vote. Less advantaged Americans vote less because they lack the skills, motivation, and networks that the better advantaged pick up through formal education and occupational advancement. In addition, political parties and campaigns focus their resources on citizens who are affluent and are already active politically.

11PF4-Income Disparities www.victorybriefs.com

Page 56 of 141

DONATIONS TO POLITICAL CANDIDATES AND PARTIES IS A FORM OF PARTICIPATION ALMOST EXCLUSIVELY PRACTICED BY THE WEALTHY Task Force on Inequality and American Democracy, American Democracy in an Age of Rising Inequality, Published by the American Political Science Association (2004), p. 7 Campaign contributors are the least representative group of citizens. Only 12 percent of American households had incomes over $100,000 in 2000, but a whopping 95 percent of the donors who made substantial contributions were in these wealthiest households.15 Figure 2 shows that 56 percent of those with incomes of $75,000 and more reported making some form of campaign contribution compared with a mere 6 percent among Americans with incomes under $15,000. Giving money to politicians is a form of citizen activity that is, in practical terms, reserved for a select group of Americans. As wealth and income have become more concentrated and the flow of money into elections has grown, campaign contributions give the affluent a means to express their voice that is unavailable to most citizens. This undoubtedly aggravates inequalities of political voice.

11PF4-Income Disparities www.victorybriefs.com

Page 57 of 141

PROGRAMS DESIGNED TO REDRESS ECONOMIC INEQUALITY HAVE HISTORICALLY IMPROVED POLITICAL PARTICIPATION Task Force on Inequality and American Democracy, American Democracy in an Age of Rising Inequality, Published by the American Political Science Association (2004), p. 15-16 What is particularly relevant for understanding political inequality in America today is that many these broadly inclusive government programs also encouraged ordinary citizens to become more active participants in our democracy they helped equalize the voice of citizens in the halls of government. The United States pioneered schooling for all, spending about as much or more than many advanced industrialized countries. Promotion of education has helped to open the door to opportunity for students who work hard, to propel the countrys economy, and to lower economic disparities. It has also boosted participation in volunteer organizations and democratic life. In higher education, the G.I. Bill extended generous assistance to attend universities, community colleges, and vocational schools for millions of veterans of World War II and the Korean War.36 Since the 1970s, federal programs like the Pell Grants and state initiatives have allowed millions of lower- and middle-income students to pursue post-secondary schooling. Similarly, Social Security, which provides protection against low income in retirement to employees who contribute to the system, has helped to foster an extraordinary level of participation by the elderly in the electoral process and civic life. Social Security has encouraged participation by low- and moderate-income seniors, which means that the elderly are less subject to the skew in favor of the affluent and better educated that generally characterizes political participation in the United States.37

11PF4-Income Disparities www.victorybriefs.com

Page 58 of 141

MANY CONTEMPORARY LIBERALS DEFEND SOCIAL INSTITUTIONS DESIGNED TO EMPOWER THE ECONOMICALLY DISADVANTAGED Francis Fukuyama [Olivier Nomellini Senior Fellow at the Freeman Spogli Institute for International Studies, Stanford University], Dealing with Inequality, Journal of Democracy Volume 22, Number 3 (July 2011), p. 82 A far more influential strand of thinking on the question of equality today comes from within the liberal tradition itself, a tradition that sees society as based upon a social contract meant to enhance the freedoms of its individual members. An example is John Rawls, a contemporary Kantian who seeks to derive abstract rules of justice without reference to substantive understandings of human nature or a human essence.6 Rawlss veil of ignorance is a heuristic device to uncover ethical rules that individuals would choose if they did not know what position they would actually occupy in society. He argues that people deliberating under these conditions would agree upon a standard of justice based on the way that society treats its least fortunate members. Rawls does not go into the practical details of how societies are to meet the needs of the least fortunate, but clearly it would entail substantial redistribution. The economist Amartya Sen offers a rather different set of arguments, also from a liberal perspective, that in the end lead to a principled case for a more substantive view of equality. Like Hayek, Sen believes that individuals aim at different ends and that societys role is to allow people to pursue their chosen life courses in the freest possible manner. Sen argues, however, that the actual freedom one can achieve is dependent on a host of material preconditions: If one is starving, or in poor health, or is a woman in a society that limits female opportunity, then a simple Hayekian freedom from government coercion will not be sufficient to allow one to make meaningful individual choices. A genuine liberal society, then, is one that provides the necessary preconditions for the exercise of individual freedom, something which again presupposes a degree of redistribution or at least an equitable provision of basic services such as education and health.7

11PF4-Income Disparities www.victorybriefs.com

Page 59 of 141

IT IS MORALLY ARBITRARY TO ALLOW SOCIAL GOODS TO BE DISTRIBUTED BY THOSE ARRANGEMENTS ARISING SPONTANEOUSLY THROUGH THE EXERCISE OF OUR NATURAL LIBERTIES John Rawls [James Bryant Conant University Professorship at Harvard University], A Theory of Justice (Revised Edition), Cambridge: The Belknap Press of the Harvard University Press (1971), p. 62-63 In the system of natural liberty the initial distribution is regulated by the arrangements implicit in the conception of careers open to talents (as easlier defined). These arrangements presuppose a background of equal liberty (as specified by the first principle) and a free market economy. They require a formal equality of opportunity in that all have at least the same legal rights of access to all advantaged social positions. But since there is no effort to preserve an equality, or similarity, of social conditions, except insofar as this is necessary to preserve the requisite background institutions, the initial distribution of assets for any period of time is strongly influenced by natural and social contingencies. The existing distribution of income and wealth, say, is the cumulative effect of prior distributions of natural assets that is, natural talents and abilities as these have been developed or left unrealized, and their use favored or disfavored over time by social circumstances and such chance contingencies as accident and good fortune. Intuitively, the most obvious injustice of the system of natural liberty is that it permits distributive shares to be improperly influenced by these factors so arbitrary from a moral point of view.

11PF4-Income Disparities www.victorybriefs.com

Page 60 of 141

LIBERAL ATTEMPTS TO SECURE EQUALITY OF OPPORTUNITY AND SO GIVE PEOPLE WHAT THEY DESERVE STILL FALL VICTIM TO MORAL ARBITRARINESS John Rawls [James Bryant Conant University Professorship at Harvard University], A Theory of Justice (Revised Edition), Cambridge: The Belknap Press of the Harvard University Press (1971), p. 63-64 The liberal interpretation of the two principles seeks, then, to mitigate the influence of social contingencies and natural fortune on distributive shares. To accomplish this end it is necessary to impose further basic structural conditions on the social system. Free market arrangements must be set within a framework of political and legal institutions which regulates the overall trends of economic events and preserves the social conditions necessary for fair equality of opportunity. The elements of this framework are familiar enough, though it may be worthwhile to recall the importance of preventing excessive accumulations of property and wealth and of maintaining equal opportunities of education for all. Chances to acquire cultural knowledge and skills should not depend upon ones class position, and so the school system, whether public or private, should be designed to even out class barriers. While the liberal conception seems clearly preferable to the system of natural liberty, intuitively it still appears defective. For one thing, even if it works to perfection in eliminating the influence of social contingencies, it still permits the distribution of wealth and income to be determined by the natural distribution of abilities and talents. Within the limits allowed by the background arrangements, distributive shares are decided by the outcome of the natural lottery; and this outcome is arbitrary from a moral perspective. There is no more reason to permit the distribution of income and wealth to be settled by the distribution of natural assets than by historical and social fortune. Furthermore, the principle of fair opportunity can only be imperfectly carried out, at least as long as some form of the family exists. The extent to which natural capacities develop and reach fruition is affected by all kinds of social conditions and class attitudes. Even the willingness to make an effort, to try, and so to be deserving in the ordinary sense is itself dependent upon happy family and social circumstances. It is impossible in practice to secure equal chances of achievement and culture for those similarly endowed, and therefore we may want to adopt a principle which recognizes this fact and also mitigates the arbitrary effects of the natural lottery itself. That the liberal conception fails to do this encourages one to look for another interpretation of the two principles of justice.

11PF4-Income Disparities www.victorybriefs.com JUSTICE REQUIRES REDRESSING ARBITRARY INEQUALITY

Page 61 of 141

John Rawls [James Bryant Conant University Professorship at Harvard University], A Theory of Justice (Revised Edition), Cambridge: The Belknap Press of the Harvard University Press (1971), p. 86-87 First we may observe that the difference principle gives some weigh to the considerations singled out by the principle of redress. This is the principle that undeserved inequlities call for redress; and since inequalities of birth and natural endowment are undeserved, these inequalities are to be somehow compensated for. 17 Thus the principle holds that in order to treat all persons equally, to provide genuine equality of opportunity, society must give more attention to those with fewer native assets and to those born into the less favorable social positions. The idea is to redress the bias of contingencies in the direction of equality. In pursuit of this principle greater resources might be spent on the education of the less rather than the more intelligent, at least over a certain time of life, say the earlier years of school. Now the principle of redress has not to my knowledge been proposed as the sole criterion of justice, as the single aim of the social order. It is plausible as most such principles are only as a prima facie principle, one tha tis to be weighed in balance with others. For example, we are to weigh it against the principle to improve the average standard of life, or to advance the common good. 18 But whatever other principles we hold, the claims of redress are to be taken into account. It is thought to represent one of the elements in our conception of justice. Now the difference principle is not of course the principle of redress. It does not require society to try to even out handicaps as if all were expected to compete on a fair basis in the same race. But the difference principle would allocate resources in education, say, so as to improve the long-term expectation the least favored. If this end is attained by giving more attention to the better endowed, it is permissible; otherwise not. And in making this decision, the value of education should not be assessed solely in terms of economic efficiency and social welfare. Equally if not more important is the role of education in enabling a person to enjoy the culture of his society and to take part in its affairs, and in this way to provide for each individual a secure sense of his own worth. Thus although the difference principle is not the same as that of redress, it does achieve some of the intent of the latter principle. It transforms the aims of the basic structure so that the total scheme of institutions no longer emphasizes social efficiency and technocratic values. The difference principle represents, in effect, an agreement to regard the distribution of natural talents as in some respects a common asset and to share in the greater social and economic benefits made possible by the complementarities of this distribution. Those who have been favored by nature, whoever they are, may gain from their good fortune only on terms that improve the situation of those who have lost out. The naturally advantaged are not to gain merely because they are more gifted, but only to cover the costs of training and education and for using their endowments in ways that help the less fortunate as well. No one deserves his greater natural capacity nor merits a more favorable starting place in society. But, of course, there is no reason to ignore, much less to eliminate these distinctions. Instead, the basic structure can be arranged so that these contingencies work for the good of the least fortunate. Thus we are led to the difference principle if we wish to set up the social system so that no one gains or loses from his arbitrary place in the distribution of natural assets or his initial position in society without giving or receiving compensating advantages in return.

11PF4-Income Disparities www.victorybriefs.com

Page 62 of 141

THE NATURAL DISTRIBUTION OF TALENT IS NEITHER JUST NOR UNJUST; HOW INSTITUTIONS DEAL WITH IT IS John Rawls [James Bryant Conant University Professorship at Harvard University], A Theory of Justice (Revised Edition), Cambridge: The Belknap Press of the Harvard University Press (1971), p. 87-88 In view of these remarks we may reject the contention that the ordering of institution s is always defective because the distribution of natural talents and the contingencies of social circumstance are unjust, and for ignoring injustice, as if the refusal this injustice must inevitably carry over to human arrangements. Occasionally this reflection is offered as an excuse for ignoring injustice, as if the refusal to acquiesce in injustice is on par with being unable to accept death. The natural distribution is neither just nor unjust; nor is it unjust that persons are born into society as some particular position. These are simply natural facts. What is just and unjust is the way that institutions deal with these facts. Aristocratic and caste societies are unjust because they make these contingencies the ascriptive basis for belonging to more or less enclosed and privileged social classes. The basic structure of these societies incorporates the arbitrariness found in nature. But there is no necessity for men to resign themselves to these contingencies. The social system is not an unchangeable order beyond human control but a pattern of human action. In justice as fairness men agree to avail themselves of the accidents of nature and social circumstance only when doing so is for the common benefit. The two principles are a fair way of meeting the arbitrariness of fortune; and while no doubt imperfect in other ways, the institutions which satisfy these principles are just.

11PF4-Income Disparities www.victorybriefs.com

Page 63 of 141

POSSESSING CHARACTER IS MORALLY ARBITRARY; IT CANNOT JUSTIFY SUPERIOR ENTITLEMENTS John Rawls [James Bryant Conant University Professorship at Harvard University], A Theory of Justice (Revised Edition), Cambridge: The Belknap Press of the Harvard University Press (1971), p. 89 Thus it is incorrect that individuals with greater natural endowments and the superior character that has made their development possible have a right to a cooperative scheme that enables them to obtain even further benefits in ways that do not contribute to the advantages of others. We do not deserve our place in the distribution of native endowments, any more than we deserve our initial starting place in society. That we deserve the superior character that enables us to make the effort to cultivate our abilities is also problematic; for such character depends in good part upon fortunate family and social circumstances in early life for which we can claim no credit. The notion of desert does not apply here. To be sure, the more advantaged have a right to their natural assets, as does everyone else; this right is covered by the first principle under the basic liberty protecting the integrity of the person. And so the more advantaged are entitled to whatever they can acquire in accordance with the rules of a fair system of social cooperation. Our problem is how this scheme, the basic structure of society, is to be designed. From a suitably general standpoint, the difference principle appears acceptable to both the more advantaged and the less advantaged individual. Of course, none of this is strictly speaking an argument for the principle, since in a contract theory arguments are made from the point of view of the original position. But these intuitive considerations help to clarify the principle and the sense in which it is egalitarian.

11PF4-Income Disparities www.victorybriefs.com

Page 64 of 141

THE EXERCISE OF VIRTUES LIKE EFFORT IS A MORALLY ARBITRARY ENDOWMENT AND SO CANNOT CREATE A JUST ENTITLMENT John Rawls [James Bryant Conant University Professorship at Harvard University], A Theory of Justice (Revised Edition), Cambridge: The Belknap Press of the Harvard University Press (1971), p. 274 Moreover, none of the precepts of justice aims at rewarding virtue. The premiums earned by scarce natural talents, for example, are to cover the costs of training and to encourage the efforts of learning, as well as to direct ability to where it best furthers the common interest. The distributive shares that result do not correlate with moral worth, since the intitial endowment of natural assets and the contingencies of their growth and nurture in early life ar arbitrary from a moral point of view. The precept which seems intuitively to come closest to rewarding moral desert is that of distribution according to effort. 39 Once again, however, it seems clear that the effort a person is willing to make is influenced by his natural abilities and skills and the alternatives open to him. The better endowed are more likely, other things equal, to strive conscientiously, and there seems to be no way to discount for their greater good fortune. The idea of rewarding desert is impracticable. And certainly to the extent that the precept of need is emphasized, moral worth is ignored. Nor does the basic structure tend to balance the precepts of justice so as to achieve the requisite correspondence behind the scenes. It is regulated by the two principles of justice which define other aims entirely.

11PF4-Income Disparities www.victorybriefs.com

Page 65 of 141

THE ABSENSE OF INDIVIDUAL DESERT CREATES A PRESUMPTION IN FAVOR OF REGARDING TALENTS AS A COMMON ASSET Michael Sandel [Prof. of Government, Harvard University], Liberalism and the Limits of Justice (Second Edition), Cambridge: Cambridge University Press (1998), p.98 For Rawls, on the other hand, the absence of individual desert creates a presumption in favor of regarding the distribution of talents as a common asset. The lack of desert or a pre-institutional concept of virtue means that institutions are unconstrained by antecedent moral claims in their pursuit of the primary virtue of social justice. In this sense, the analogy of manna from even is apt. The array of assets dealt by fortune is neither just nor unjust. These are simply natural facts. What is just and unjust is the way that institutions deal with these facts (102). There is no reason to let assets and the benefits that flow from them lie where they fall. This would be simply to incorporate and affirm the arbitrariness of nature. The discovery that virtue and entitlements await social institutions rather than constrain them is a reason to pursue justice all the more insistently, not a reason to freeze arbitrariness in place.

11PF4-Income Disparities www.victorybriefs.com

Page 66 of 141

INCOME INEQUALITY UNDERMINES THE BASIC POLITICAL VALUE OF FAIRNESS Joseph E. Stiglitz [Prof. of Economics, Columbia University, 2002 Winner of the Nobel Memorial Prize in Economic Sciences], Of the 1%, by the 1%, for the 1%, Vanity Fair, May 2011, <http://www.vanityfair.com/society/features/2011/05/top-one-percent-201105> Or, more accurately, they think they dont. Of all the costs imposed on our society by the top 1 percent, perhaps the greatest is this: the erosion of our sense of identity, in which fair play, equality of opportunity, and a sense of community are so important. America has long prided itself on being a fair society, where everyone has an equal chance of getting ahead, but the statistics suggest otherwise: the chances of a poor citizen, or even a middle-class citizen, making it to the top in America are smaller than in many countries of Europe. The cards are stacked against them. It is this sense of an unjust system without opportunity that has given rise to the conflagrations in the Middle East: rising food prices and growing and persistent youth unemployment simply served as kindling. With youth unemployment in America at around 20 percent (and in some locations, and among some socio-demographic groups, at twice that); with one out of six Americans desiring a full-time job not able to get one; with one out of seven Americans on food stamps (and about the same number suffering from food insecurity)given all this, there is ample evidence that something has blocked the vaunted trickling down from the top 1 percent to everyone else. All of this is having the predictable effect of creating alienation voter turnout among those in their 20s in the last election stood at 21 percent, comparable to the unemployment rate.

11PF4-Income Disparities www.victorybriefs.com ECONOMIC INEQUALITY UNDERMINES COMMUNITY SOLIDARITY

Page 67 of 141

Michael Sandel [Prof. of Government, Harvard University], Justice: Whats The Right Thing To Do?, New York: Farrar, Straus and Giroux, p. 266-267 But there is a third, more important reason to worry about the growing inequality of American life: Too great a gap between rich and poor undermines the solidarity that democratic citizenship requires. Heres how: As inequality deepens, rich and poor live increasingly separate lives. The affluent send their children to private schools (or to public schools in wealthy suburbs), leaving urban public schools to the children of families who have no alternative. A similar trend leads to the secession by the privileged from other public institutions and facilities. 43 Private health clubs replace municipal recreation centers and swimming pools. Upscale residential communities hire private security guards and rely less on public transportation. And so on. The affluent secede from public places and services, leaving them to those who cant afford anything else. This has two bad effects, one fiscal, the other civic. First, public services deteriorate, as those who no longer use those services become less willing to support them with their taxes. Second, public instuttions such as schools, parks, playgrounds, and community centers cease to be places where citizens from different walks of life encounter one another. Institutions that once gathered people together and served as informal schools of civic virtue become few and far between. The hollowing out of the public realm makes it difficult to cultivate the solidarity and sense of community on which democratic citizenship depends. So, quite apart from its effects on utility or consent, inequality can be corrosive to civic virtue. Conservatives enamored of markets and liberals concerned with redistribution overlook this loss.

11PF4-Income Disparities www.victorybriefs.com ECONOMIC STRATIFICATION CAN UNDERMINE SOCIAL COHESION

Page 68 of 141

Francis Fukuyama [Olivier Nomellini Senior Fellow at the Freeman Spogli Institute for International Studies, Stanford University], Dealing with Inequality, Journal of Democracy Volume 22, Number 3 (July 2011), p. 83-84 Nonetheless, it is more than likely that some of Pickett and Wilkinsons causal assertions would withstand more rigorous analysis. For example, social trust between rich and poor in highly stratified countries is for a number of obvious reasons likely to be low, and low social trust has a number of deleterious effects on the quality of democracy.11 Economic divisions often correspond to ethnic, racial, or religious ones. Paul Colliers work has suggested that much of what passes for ethnic conflict in poor countries is actually driven by conflicts over resources in highly unequal societies.12 Social conflict can be driven either by minorities who are poorer than the rest of society, as in the case of African Americans in the United States or Muslim immigrants in the Netherlands or France, or by minorities that are better off than average, like the Chinese in Southeast Asia or the Indians and Lebanese in East Africa.

11PF4-Income Disparities www.victorybriefs.com

Page 69 of 141

PUBLIC PROGRAMS TO HELP ALLEVIATE INEQUALITY SOLIDIFY COMMUNITY SOLIDARITY Michael Sandel [Prof. of Government, Harvard University], Justice: Whats The Right Thing To Do?, New York: Farrar, Straus and Giroux, p. 267-268 If the erosion of the public realm is the problem, what is the solution? A politics of the common good would take as one of its primary goals the reconstruction of the infrastructure of civic life. Rather than focus on redistribution for the sake of broadening access to private consumption, it would tax the affluent to rebuild public institutions and services so that rich and poor alike would want to take advantage of them. An earlier generation made a massive investment in the federal highway program, which gave Americans unprecedented individual mobility and freedom, but also contributed to the reliance on the private automobile, suburban sprawl, environmental degradation, and living patterns corrosive to community. This generation could commit itself to an equally consequential investment in an infrastructure for civic renewal: public schools to which rich and poor alike would want to send their children; public transportation systems reliable enough to attract upscale commuters; and public health clinics, playgrounds, parks, recreation centers, libraries, and museums that would, ideally at least, draw people out of their gated communities and into the common spaces of shared democratic citizenship. Focusing on the civic consequences of inequality, and ways of reversing them, might find political traction that arguments about income distribution as such do not. It would also help highlight the connection between distributive justice and the common good.

11PF4-Income Disparities www.victorybriefs.com

Page 70 of 141

GOVERNMENT ACTIONS SYMBOLICALLY AFFIRM THE VALUES OF SOLIDARITY AND HUMAN DIGNITY Robert Nozick [Prof., Harvard University], The Examined Life: Philosophical Mediations, New York: Simon & Schuster (1989), p. 286-287 WE WANT our individual lives to express our conceptions of reality (and of responsiveness to that); so too we want the institutions demarcating our lives together to express and saliently symbolize our desired mutual relations. Democratic institutions and the liberties coordinate with them are not simply effective means toward controlling the powers of government and directing these toward matters of joint concern; they themselves express and symbolize, in a pointed and official way, our equal human dignity, our autonomy and powers of self-direction. We vote, although we are cognizant of the minuscule probability that our own actual vote will have some decisive effect on the outcome, in part as an expression and symbolic affirmation of our status as autonomous and self-governing beings whose considered judgments or even opinions have to be given weight equal to those of others. That symbolism is important to us. Within the operation of democratic institutions, too, we want expressions of the values that concern us and bind us together. The libertarian position I once propounded now seems to me seriously inadequate, in part because it did not fully knit the humane considerations and joint cooperative activities it left room for more closely into its fabric. It neglected the symbolic importance of an official political concern with issues or problems, as a way of marking their importance or urgency, and hence of expressing, intensifying, channeling, encouraging, and validating our private actions and concerns toward them. Joint goals that the government ignores completely it is different with private or family goals tend to appear unworthy of our joint attention and hence to receive little. There are some things we choose to do together through government in solemn marking of our human solidarity, served by the fact that we do them together in this official fashion and often also by the content of the action itself.*

11PF4-Income Disparities www.victorybriefs.com

Page 71 of 141

VOLUNTARY CONTRIBUTIONS TO HELP ALLEVIATE INEQUALITY ARE INSUFFICIENT TO SYMBOLICALLY EXPRESS THE VALUE OF COMMUNITY SOLIDARITY Robert Nozick [Prof., Harvard University], The Examined Life: Philosophical Mediations, New York: Simon & Schuster (1989), p. 288-289 But dont people have a right not to feel ties of solidarity and concern, and if so, how can the political society take seriously its symbolic expression of what may not be there? By what right does it express for others what they themselves choose not to? These others should feel they would be better human beings if they felt ties of solidarity and concern for fellow citizens (and for fellow human beings, perhaps also for fellow living things), although they do have a right not to feel this. (People sometimes have a right not to do or feel something even though they should; they have the right to choose.) Their fellow citizens, though, may choose to speak for them to cover up that lack of concern and solidarity whether or not the people themselves realize they are lacking something. This covering up for them may be done out of politeness, or because of the importance to the others of a joint public affirmation of concern and solidarity, if only so they wont be forced to notice how uncaring and inhumane some of their compatriots are. To be sure, this joint public affirmation is not simply verbal; those spoken for may have to pay taxes to help support the programs it involves. (That a fig leaf was created to cover the shame of their unconcern does not mean they do not have to help pay for it.) The complete absence of any symbolic public expression and marking caring and solidarity would leave the rest of us bereft of a society validating human relatedness. Well, why dont those who want and need such a society voluntarily contribute to pay for its public programs rather than taxing the others, who dont care anything about it? But a program thus supported by many peoples voluntary contributions, worthy though it might be, would not constitute the societys solemn marking and symbolic validation of the importance and centrality of those ties of concern and solidarity. That can occur only through its official joint action, speaking in the name of the whole. The point is not simply to accomplish the particular purpose that might be done through private contributions alone or to get the others to pay too that could occur through stealing the necessary funds from them but also to speak solemnly in everyones name, in the name of the society, about what it holds dear. A particular individual might prefer to speak only for himself. But to live in a society and to identify with it necessarily lays you open to being ashamed of things for which you are not personally responsible wars of oppression or subverting of foreign governments and to being proud of things you yourself have not done. A society sometimes speaks in our names. We could satisfy the people who object to the joint public expression of caring and solidarity and their attendant programs by eliminating such expressions, but this would leave the rest of us ashamed of our society, whose public voice of concern is silent. That silence would then speak for us. Just stop identifying with the society, then! You then wont have to be ashamed of what it does or doesnt do or say. To accommodate the objector to the public program, then, not only must we thwart our desire and need to jointly mark what we hold to be most central about our interrelations a desire and need that are continuous with those for personal self-expression we must stop identifying with our society despite all this means for our emotional life and sense of ourselves. The cost is too great.

11PF4-Income Disparities www.victorybriefs.com

Page 72 of 141

THE SUPER-RICH ARE UNLIKELY TO SUPPORT SPENDING FOR THE COMMON GOOD Joseph E. Stiglitz [Prof. of Economics, Columbia University, 2002 Winner of the Nobel Memorial Prize in Economic Sciences], Of the 1%, by the 1%, for the 1%, Vanity Fair, May 2011, <http://www.vanityfair.com/society/features/2011/05/top-one-percent-201105> None of this should come as a surpriseit is simply what happens when a societys wealth distribution becomes lopsided. The more divided a society becomes in terms of wealth, the more reluctant the wealthy become to spend money on common needs. The rich dont need to rely on government for parks or education or medical care or personal securitythey can buy all these things for themselves. In the process, they become more distant from ordinary people, losing whatever empathy they may once have had. They also worry about strong governmentone that could use its powers to adjust the balance, take some of their wealth, and invest it for the common good. The top 1 percent may complain about the kind of government we have in America, but in truth they like it just fine: too gridlocked to re-distribute, too divided to do anything but lower taxes.

11PF4-Income Disparities www.victorybriefs.com

Page 73 of 141

CONCENTRATION OF WEALTH UNDERMINES COLLECTIVE ACTION FOR THE COMMON GOOD Joseph E. Stiglitz [Prof. of Economics, Columbia University, 2002 Winner of the Nobel Memorial Prize in Economic Sciences], Of the 1%, by the 1%, for the 1%, Vanity Fair, May 2011, <http://www.vanityfair.com/society/features/2011/05/top-one-percent-201105> Americas inequality distorts our society in every conceivable way. There is, for one thing, a welldocumented lifestyle effectpeople outside the top 1 percent increasingly live beyond their means. Trickle-down economics may be a chimera, but trickle-down behaviorism is very real. Inequality massively distorts our foreign policy. The top 1 percent rarely serve in the militarythe reality is that the all-volunteer army does not pay enough to attract their sons and daughters, and patriotism goes only so far. Plus, the wealthiest class feels no pinch from higher taxes when the nation goes to war: borrowed money will pay for all that. Foreign policy, by definition, is about the balancing of national interests and national resources. With the top 1 percent in charge, and paying no price, the notion of balance and restraint goes out the window. There is no limit to the adventures we can undertake; corporations and contractors stand only to gain. The rules of economic globalization are likewise designed to benefit the rich: they encourage competition among countries for business, which drives down taxes on corporations, weakens health and environmental protections, and undermines what used to be viewed as the core labor rights, which include the right to collective bargaining. Imagine what the world might look like if the rules were designed instead to encourage competition among countries for workers. Governments would compete in providing economic security, low taxes on ordinary wage earners, good education, and a clean environmentthings workers care about. But the top 1 percent dont need to care.

11PF4-Income Disparities www.victorybriefs.com ECONOMIC INEQUALITY HAS SUBSTANTIAL SOCIAL COSTS

Page 74 of 141

Frank Stilwell [Prof. of Political Economy at the University of Sydney], Why bother about economic inequality? Evatt Foundation, 8 July 2003. <http://www.evatt.org.au/papers/whybother-about-economic-inequality.html> Socially, growing economic inequality has strong connections with an increased incidence of crime and ill-health. As the latest UN Human Development report notes, differences in income inequality across countries are closely associated with differences in rates of crime and violence. On the health issue, some particularly interesting epidemiological evidence compiled by R. G. Wilkinson shows that ill-health is not just correlated with poverty, but also with international differences in economic inequality. Relative deprivation matters, and it shows up in ill-health as a social problem and economic cost. More generally, of course, economic inequality undermines social cohesion. The situation of US cities, where the rich take refuge from the 'underclass' by living in security-patrolled gated communities, is already being reproduced down under. It is not a scenario conducive to society as a whole feeling relaxed and comfortable.

11PF4-Income Disparities www.victorybriefs.com

Page 75 of 141

ECONOMIC INEQUALITY PREVENTS SOCIAL COOPERATION TO PROTECT THE ENVIRONMENT Frank Stilwell [Prof. of Political Economy at the University of Sydney], Why bother about economic inequality? Evatt Foundation, 8 July 2003. <http://www.evatt.org.au/papers/whybother-about-economic-inequality.html> From an environmental perspective, economic inequality is one of the many impediments to the achievement of ecological sustainability. It is only when people perceive that they are 'all in the same boat' and have to incur reasonable equality of sacrifice for the common good that they will be willing to accept the policies needed to rein in the excessive levels of production and consumption that lie behind this latent crisis. Otherwise, access to 'environmental goods' ultimately to life itself - becomes increasingly a matter of ability to pay.

11PF4-Income Disparities www.victorybriefs.com

Page 76 of 141

THE CHOICES PEOPLE MAKE IN THE FREE MARKET ARE NOT ALWAYS FREE Michael Sandel [Prof. of Government, Harvard University], Justice: Whats The Right Thing To Do?, New York: Farrar, Straus and Giroux, p. 81-82 The first objection holds that, for those with limited alternatives, the free market is not all that free. Consider an extreme case: A homeless person sleeping under a bridge may have chosen, in some sense, to do so; but we would not necessarily consider his choice to be a free one. Nor would we be justified in assuming that he must prefer sleeping under a bridge to sleeping in an apartment. In order to know whether his choice reflects a preference for sleeping out of doors or an inability to afford an apartment, we need to know something about his circumstances. Is he doing this freely or out of necessity? The same question can be asked about market choices generally including the choices people make when they take on various jobs. How does this apply to military service? We cant determine the justice or injustice of the volunteer army without knowing more about the background conditions that prevail in the society: Is there a reasonable degree of equal opportunity, or do some people have very few options in life? Does everyone have a chance to get a college education, or is it the case that, for some people, the only way to afford college is to enlist in the military? From the standpoint of market reasoning, the volunteer army is attractive because it avoids the coercion of conscription. It makes military service a matter of consent. But some people who wind up serving in the all-volunteer army may be as averse to military service as those who stay away. If poverty and economic disadvantage are widespread, the choice to enlist may simply reflect the lack of alternatives. According to this objection, the volunteer army may not be as voluntary as it seems. In fact, it may involve an element of coercion. If some in the society have no other good options, those who choose to enlist may be conscripted, in effect, by economic necessity. In that case, the difference between conscription and the volunteer army is not that one is compulsory while the other is free; its rather that each employs a different form of compulsion the force of law in the fist case and pressure of economic necessity on in the second. Only if people have a reasonable range of decent job options can it be said that the choice to serve for pay reflects their preferences rather than their limited alternatives.

11PF4-Income Disparities www.victorybriefs.com

Page 77 of 141

THE BELIEF THAT THE WEALTHY ARE MORE PRODUCTIVE FOR SOCIETY IS UNTRUE Joseph E. Stiglitz [Prof. of Economics, Columbia University, 2002 Winner of the Nobel Memorial Prize in Economic Sciences], Of the 1%, by the 1%, for the 1%, Vanity Fair, May 2011, <http://www.vanityfair.com/society/features/2011/05/top-one-percent-201105> Economists long ago tried to justify the vast inequalities that seemed so troubling in the mid-19th centuryinequalities that are but a pale shadow of what we are seeing in America today. The justification they came up with was called marginal-productivity theory. In a nutshell, this theory associated higher incomes with higher productivity and a greater contribution to society. It is a theory that has always been cherished by the rich. Evidence for its validity, however, remains thin. The corporate executives who helped bring on the recession of the past three yearswhose contribution to our society, and to their own companies, has been massively negativewent on to receive large bonuses. In some cases, companies were so embarrassed about calling such rewards performance bonuses that they felt compelled to change the name to retention bonuses (even if the only thing being retained was bad performance). Those who have contributed great positive innovations to our society, from the pioneers of genetic understanding to the pioneers of the Information Age, have received a pittance compared with those responsible for the financial innovations that brought our global economy to the brink of ruin.

11PF4-Income Disparities www.victorybriefs.com

Page 78 of 141

IF ALL WE CARE ABOUT IS THE SIZE OF THE ECONOMY RATHER THAN INCOME DISTRIBUTION, WE CREATE ECONOMIC AND POLITICAL DISTORTIONS THAT ARE DAMAGING IN THE LONG RUN Joseph E. Stiglitz [Prof. of Economics, Columbia University, 2002 Winner of the Nobel Memorial Prize in Economic Sciences], Of the 1%, by the 1%, for the 1%, Vanity Fair, May 2011, <http://www.vanityfair.com/society/features/2011/05/top-one-percent-201105> Some people look at income inequality and shrug their shoulders. So what if this person gains and that person loses? What matters, they argue, is not how the pie is divided but the size of the pie. That argument is fundamentally wrong. An economy in which most citizens are doing worse year after yearan economy like Americasis not likely to do well over the long haul. There are several reasons for this. First, growing inequality is the flip side of something else: shrinking opportunity. Whenever we diminish equality of opportunity, it means that we are not using some of our most valuable assetsour peoplein the most productive way possible. Second, many of the distortions that lead to inequalitysuch as those associated with monopoly power and preferential tax treatment for special interestsundermine the efficiency of the economy. This new inequality goes on to create new distortions, undermining efficiency even further. To give just one example, far too many of our most talented young people, seeing the astronomical rewards, have gone into finance rather than into fields that would lead to a more productive and healthy economy. Third, and perhaps most important, a modern economy requires collective actionit needs government to invest in infrastructure, education, and technology. The United States and the world have benefited greatly from government-sponsored research that led to the Internet, to advances in public health, and so on. But America has long suffered from an under-investment in infrastructure (look at the condition of our highways and bridges, our railroads and airports), in basic research, and in education at all levels. Further cutbacks in these areas lie ahead.

11PF4-Income Disparities www.victorybriefs.com CONCENTRATION OF WEALTH AMONG THE TOP 1% IS LARGELY DUE TO GOVERNMENT MANIPULATION

Page 79 of 141

Joseph E. Stiglitz [Prof. of Economics, Columbia University, 2002 Winner of the Nobel Memorial Prize in Economic Sciences], Of the 1%, by the 1%, for the 1%, Vanity Fair, May 2011, <http://www.vanityfair.com/society/features/2011/05/top-one-percent-201105> But one big part of the reason we have so much inequality is that the top 1 percent want it that way. The most obvious example involves tax policy. Lowering tax rates on capital gains, which is how the rich receive a large portion of their income, has given the wealthiest Americans close to a free ride. Monopolies and near monopolies have always been a source of economic powerfrom John D. Rockefeller at the beginning of the last century to Bill Gates at the end. Lax enforcement of anti-trust laws, especially during Republican administrations, has been a godsend to the top 1 percent. Much of todays inequality is due to manipulation of the financial system, enabled by changes in the rules that have been bought and paid for by the financial industry itselfone of its best investments ever. The government lent money to financial institutions at close to 0 percent interest and provided generous bailouts on favorable terms when all else failed. Regulators turned a blind eye to a lack of transparency and to conflicts of interest. When you look at the sheer volume of wealth controlled by the top 1 percent in this country, its tempting to see our growing inequality as a quintessentially American achievementwe started way behind the pack, but now were doing inequality on a world-class level. And it looks as if well be building on this achievement for years to come, because what made it possible is selfreinforcing. Wealth begets power, which begets more wealth. During the savings-and-loan scandal of the 1980sa scandal whose dimensions, by todays standards, seem almost quaint the banker Charles Keating was asked by a congressional committee whether the $1.5 million he had spread among a few key elected officials could actually buy influence. I certainly hope so, he replied. The Supreme Court, in its recent Citizens United case, has enshrined the right of corporations to buy government, by removing limitations on campaign spending. The personal and the political are today in perfect alignment. Virtually all U.S. senators, and most of the representatives in the House, are members of the top 1 percent when they arrive, are kept in office by money from the top 1 percent, and know that if they serve the top 1 percent well they will be rewarded by the top 1 percent when they leave office. By and large, the key executive-branch policymakers on trade and economic policy also come from the top 1 percent. When pharmaceutical companies receive a trillion-dollar giftthrough legislation prohibiting the government, the largest buyer of drugs, from bargaining over priceit should not come as cause for wonder. It should not make jaws drop that a tax bill cannot emerge from Congress unless big tax cuts are put in place for the wealthy. Given the power of the top 1 percent, this is the way you would expect the system to work.

11PF4-Income Disparities www.victorybriefs.com

Page 80 of 141

INEQUALITY IS BAD FOR ECONOMIC EFFICIENCY BECAUSE IT UNDERMINES COOPERATION BETWEEN CAPITAL AND LABOR Frank Stilwell [Prof. of Political Economy at the University of Sydney], Why bother about economic inequality? Evatt Foundation, 8 July 2003. <http://www.evatt.org.au/papers/whybother-about-economic-inequality.html> This tendency to increased economic inequality has diverse consequences - economic, social, environmental, and political. Economically, the danger is that growing inequality undermines the potential for economic co-operation between capital and labour. More generally, it vitiates cooperation among all whose contribution to the creation of social wealth depends on the perception of 'fair shares' in the outcome. If so, that undermines the necessary conditions for economic efficiency within the private or public sectors. In effect, violating equity endangers efficiency. The much-touted 'efficiency-equity trade off' is a myth, as American political scientist Robert Kuttner's research on analyses of cross-country comparisons shows. At the macroeconomic scale, 'incentivation' does not work.

11PF4-Income Disparities www.victorybriefs.com

Page 81 of 141

AMERICANS FAIL TO ADVOCATE FOR MORE ECONOMIC EQUALITY BECAUSE THEY MISPERCEIVE EXISTING WEALTH DISPARITIES AND OVERESTIMATE SOCIAL MOBILITY Michael I. Norton [Associate Prof. of Business Administration, Harvard Business School] and Dan Ariely [Prof. of Psychology and Behavioral Economics, Duke University], Building a Better AmericaOne Wealth Quintile at a Time, Perspectives on Psychological Science 6(1): 9-12 (2011), pp. 10-12 Given the consensus among disparate groups on the gap between an ideal distribution of wealth and the actual level of wealth inequality, why are more Americans, especially those with low income, not advocating for greater redistribution of wealth? First, our results demonstrate that Americans appear to drastically underestimate the current level of wealth inequality, suggesting they may simply be unaware of the gap. Second, just as people have erroneous beliefs about the actual level of wealth inequality, they may also hold overly optimistic beliefs about opportunities for social mobility in the United States (Benabou & Ok, 2001; Charles & Hurst, 2003; Keister, 2005), beliefs which in turn may drive support for unequal distributions of wealth. Third, despite the fact that conservatives and liberals in our sample agree that the current level of inequality is far from ideal, public disagreements about the causes of that inequality may drown out this consensus (Alesina & Angeletos, 2005; Piketty, 1995). Finally, and more broadly, Americans exhibit a general disconnect between their attitudes toward economic inequality and their selfinterest and public policy preferences (Bartels, 2005; Fong, 2001), suggesting that even given increased awareness of the gap between ideal and actual wealth distributions, Americans may remain unlikely to advocate for policies that would narrow this gap.

11PF4-Income Disparities www.victorybriefs.com

Page 82 of 141

SOCIAL MOBILITY WOULD MITIGATE CONCERNS ABOUT INCOME INEQUALITY, BUT WE FAIL TO TAKE ADEQUATE MEASURES TO ASSURE SUCH MOBILITY Mark Thoma [Macroeconomist and Time-Series Econometrician, University of Oregon], Income Inequality is Hobbling the Middle Class, Fiscal Times, 25 Oct. 2011, <http://www.thefiscaltimes.com/Columns/2011/10/25/Income-Inequality-Is-Hobbling-the-MiddleClass.aspx#page1> If economic mobility increased along with the increase in inequality, then this would at least partially offset the worries associated with the rising concentration of income. To see how, suppose there are two types of jobs in society. One type is desirable and well paying; the other is hard, miserable work with little compensation. Suppose also that one group in society always gets the good jobs while the other gets the bad even though most people within each group are equally qualified to do both types of work. Thus, this is a highly unequal outcome. However if there is mobility if we prevent one group from keeping the good jobs through political power or other means and instead use a lottery at the beginning of each week to see which of the qualified workers does which job then the random allocation of jobs over time helps to solve the inequality problem. What has happened to mobility in the U.S.? Is it any easier to move up and down the ladder than it used to be? Unfortunately, the answer is no. As Lane Kenworthy, one of the top experts on this topic notes: Rising income and earnings inequality in the United States does not appear to have been offset by increased mobility. He points to several ways we might increase mobility such as universal preschool, affordable college, improved K-12 schooling, and policies to strengthen the social safety net so that households do not lose ground when they experience problems such as the need for expensive health care or job loss in a recession.

11PF4-Income Disparities www.victorybriefs.com

Page 83 of 141

SOCIAL MOBILITY IS HAMPERED BY THE CONCENTRATION OF WEALTH AND POWER, FOR EXAMPLE BY THE REDUCTION OR ELIMINATION OF THE ESTATE TAX Mark Thoma [Macroeconomist and Time-Series Econometrician, University of Oregon], Income Inequality is Hobbling the Middle Class, Fiscal Times, 25 Oct. 2011, <http://www.thefiscaltimes.com/Columns/2011/10/25/Income-Inequality-Is-Hobbling-the-MiddleClass.aspx#page1> Both types of mobility can be hampered by the concentration of wealth and power. For example, if wealth and power can be passed from generation to generation, and if the power that comes with wealth allows individuals to erect barriers that protect their businesses from competition or regulation, or to protect their position in society in other ways for example access to better education then the wealth that is passed from generation to generation serves as a barrier to mobility both within and across generations. Recently, the American Family Business Foundation began pushing for a cut in estate taxes as a job-creation policy. The job creation angle is, of course, just an excuse to advocate for a cut in these taxes, and this is far from the first or the last time well hear an argument from Republicans that these taxes should be reduced or eliminated entirely. But in addition to the fact that reducing estate taxes would cost us revenue at a time when we are looking to shore up our long-term finances, we should also consider the extent to which this would perpetuate existing social and power structures and erect barriers to social and economic mobility. If anything, these taxes should be increased.

11PF4-Income Disparities www.victorybriefs.com

Page 84 of 141

A VARIETY OF INCOME DISTRIBUTION PROGRAMS HELP TO BOLSTER THE MIDDLE CLASS AND THUS IMPROVE SOCIAL MOBILITY Mark Thoma [Macroeconomist and Time-Series Econometrician, University of Oregon], Income Inequality is Hobbling the Middle Class, Fiscal Times, 25 Oct. 2011, <http://www.thefiscaltimes.com/Columns/2011/10/25/Income-Inequality-Is-Hobbling-the-MiddleClass.aspx#page1> There is one final factor to consider. A vibrant middle class is essential to mobility. For many people, the middle class serves as a steppingstone to the upper classes both within and across generations. If we continue along the path we are on to an increasingly two-tiered society, and if the middle class continues to experience problems, then there will be less room at the middle tiers of society for those who are trying to move up the ladder. Mobility between the classes will be reduced. The increase in inequality in recent decades needs to be addressed. Increasing mobility can help, but this alone wont overcome the inequality problem. However, we should still do our best to ease the frictions than inhibit mobility between social classes through better education, universal health care, social insurance programs that prevent large income losses, protection for the middle class, and other measures including the redistribution of income to prevent the accumulation of wealth and power in society that help to ensure that opportunity is more equitably distributed.

11PF4-Income Disparities www.victorybriefs.com

Page 85 of 141

LACK OF SOCIAL MOBILITY DRIVES THE PUSH FOR SOCIAL PROGRAMS TO ENSURE EQUALITY OF OPPORTUNITY Francis Fukuyama [Olivier Nomellini Senior Fellow at the Freeman Spogli Institute for International Studies, Stanford University], Dealing with Inequality, Journal of Democracy Volume 22, Number 3 (July 2011), p. 86-87 In many respects, ones attitude toward inequality and the measures that one believes a democratic government should take to reduce it depends on ones belief about the type of society one is living in. The closer a society comes to having a level playing field in which differences in outcomes reflect individual-level differences in talents and character, the more likely one is to favor a Lockean liberal political system in which government protects only the equal ability to compete. But if a societys social structures or institutions obstruct individual access to opportunity, whether on the basis of class, race, gender, or some other ascriptive characteristic, then there is more likely to be a constituency in favor of government action to level up the playing field through various kinds of redistributive interventions. The belief on the part of many Americans that they live in a society with abundant social mobility, and that Europeans do not, provides one of the classic explanations for why Americans have been leery of European-style welfare states.22 Very few liberal democracies actually succeed in coming close to the ideal of a level playing field, however, so the demand for using social policies to rectify existing social inequalities is sure to persist.23

11PF4-Income Disparities www.victorybriefs.com

Page 86 of 141

THERE ARE A VARIETY OF POLICY OPTIONS AVAILABLE TO REDRESS INCOME INEQUAILTY Frank Stilwell [Prof. of Political Economy at the University of Sydney], Why bother about economic inequality? Evatt Foundation, 8 July 2003. <http://www.evatt.org.au/papers/whybother-about-economic-inequality.html> Redress of economic inequalities, nationally and internationally, is clearly imperative in these circumstances. There are numerous policy instruments available to nation states if there is the political will to use them - more progressive taxes on income and wealth, taxes on property and land ownership which capture the unearned incomes arising from inflationary processes, and inheritance taxes which capture some part of these unearned incomes which perpetuate inequality intergenerationally. Incomes policies can also seek to tackle the distribution of incomes at source.

11PF4-Income Disparities www.victorybriefs.com

Page 87 of 141

ECONOMIC INEQUALITY IS AT AN ALL TIME HIGH, THOUGH PEOPLE DISAGREE ON WHAT THE IDEAL WEALTH DISTRIBUTION IS Michael I. Norton [Associate Prof. of Business Administration, Harvard Business School] and Dan Ariely [Prof. of Psychology and Behavioral Economics, Duke University], Building a Better AmericaOne Wealth Quintile at a Time, Perspectives on Psychological Science 6(1): 9-12 (2011), p. 9 Most scholars agree that wealth inequality in the United States is at historic highs, with some estimates suggesting that the top 1% of Americans hold nearly 50% of the wealth, topping even the levels seen just before the Great Depression in the 1920s (Davies, Sandstrom, Shorrocks, & Wolff, 2009; Keister, 2000; Wolff, 2002). Although it is clear that wealth inequality is high, determining the ideal distribution of wealth in a society has proven to be an intractable question, in part because differing beliefs about the ideal distribution of wealth are the source of friction between policymakers who shape that distribution: Proponents of the estate tax, for example, argue that the wealth that parents bequeath to their children should be taxed more heavily than those who refer to this policy as a burdensome death tax.

11PF4-Income Disparities www.victorybriefs.com

Page 88 of 141

WITHOUT KNOWING THEIR PLACE IN SOCIETY, PEOPLE TEND TO PREFER A WEALTH DISTRIBUTION THAT IS MUCH MORE EGALITARIAN THAN THE UNITED STATES Michael I. Norton [Associate Prof. of Business Administration, Harvard Business School] and Dan Ariely [Prof. of Psychology and Behavioral Economics, Duke University], Building a Better AmericaOne Wealth Quintile at a Time, Perspectives on Psychological Science 6(1): 9-12 (2011), p. 10 For the first task, we created three unlabeled pie charts of wealth distributions, one of which depicted a perfectly equal distribution of wealth. Unbeknownst to respondents, a second distribution reflected the wealth distribution in the United States; in order to create a distribution with a level of inequality that clearly fell in between these two charts, we constructed a third pie chart from the income distribution of Sweden (Fig. 1).2 We presented respondents with the three pairwise combinations of these pie charts (in random order) and asked them to choose which nation they would rather join given a Rawls constraint for determining a just society (Rawls, 1971): In considering this question, imagine that if you joined this nation, you would be randomly assigned to a place in the distribution, so you could end up anywhere in this distribution, from the very richest to the very poorest. As can be seen in Figure 1, the (unlabeled) United States distribution was far less desirable than both the (unlabeled) Sweden distribution and the equal distribution, with some 92% of Americans preferring the Sweden distribution to the United States. In addition, this overwhelming preference for the Sweden distribution over the United States distribution was robust across gender (females: 92.7%, males: 90.6%), preferred candidate in the 2004 election (Bush voters: 90.2%; Kerry voters: 93.5%) and income (less than $50,000: 92.1%; $50,001$100,000: 91.7%; more than $100,000: 89.1%). In addition, there was a slight preference for the distribution that resembled Sweden relative to the equal distribution, suggesting that Americans prefer some inequality to perfect equality, but not to the degree currently present in the United States.

11PF4-Income Disparities www.victorybriefs.com

Page 89 of 141

PEOPLE UNDERESTIMATE THE WEALTH GAP IN THE UNITED STATES AND WOULD DESIGN A MORE EGALITARIAN SOCIETY Michael I. Norton [Associate Prof. of Business Administration, Harvard Business School] and Dan Ariely [Prof. of Psychology and Behavioral Economics, Duke University], Building a Better AmericaOne Wealth Quintile at a Time, Perspectives on Psychological Science 6(1): 9-12 (2011), p. 10 Figure 2 shows the actual wealth distribution in the United States at the time of the survey, respondents overall estimate of that distribution, and respondents ideal distribution. These results demonstrate two clear messages. First, respondents vastly underestimated the actual level of wealth inequality in the United States, believing that the wealthiest quintile held about 59% of the wealth when the actual number is closer to 84%. More interesting, respondents constructed ideal wealth distributions that were far more equitable than even their erroneously low estimates of the actual distribution, reporting a desire for the top quintile to own just 32% of the wealth. These desires for more equal distributions of wealth took the form of moving money from the top quintile to the bottom three quintiles, while leaving the second quintile unchanged, evincing a greater concern for the less fortunate than the more fortunate (Charness & Rabin, 2002).

11PF4-Income Disparities www.victorybriefs.com

Page 90 of 141

DESPITE SOME DIFFERENCES IN DEMOGRAPHIC GROUPS, THERE IS WIDE CONSENSUS THAT SOCIETY SHOULD BE MORE EGALITARIAN BUT THAT SOME INEQUALITY IS APPROPRIATE Michael I. Norton [Associate Prof. of Business Administration, Harvard Business School] and Dan Ariely [Prof. of Psychology and Behavioral Economics, Duke University], Building a Better AmericaOne Wealth Quintile at a Time, Perspectives on Psychological Science 6(1): 9-12 (2011), pp. 10-12 We next explored how demographic characteristics of our respondents affected these estimates. Figure 3 shows these estimates broken down by three levels of income, by whether respondents voted for George W. Bush (Republican) or John Kerry (Democrat) for United States president in 2004, and by gender. Males, Kerry voters, and wealthier individuals estimated that the distribution of wealth was relatively more unequal than did women, Bush voters, and poorer individuals. For estimates of the ideal distribution, women, Kerry voters, and the poor desired relatively more equal distributions than did their counterparts. Despite these (somewhat predictable) differences, what is most striking about Figure 3 is its demonstration of much more consensus than disagreement among these different demographic groups. All groupseven the wealthiest respondentsdesired a more equal distribution of wealth than what they estimated the current United States level to be, and all groups also desired some inequalityeven the poorest respondents. In addition, all groups agreed that such redistribution should take the form of moving wealth from the top quintile to the bottom three quintiles. In short, although Americans tend to be relatively more favorable toward economic inequality than members of other countries (Osberg & Smeeding, 2006), Americans consensus about the ideal distribution of wealth within the United States appears to dwarf their disagreements across gender, political orientation, and income. Overall, these results demonstrate two primary messages. First, a large nationally representative sample of Americans seems to prefer to live in a country more like Sweden than like the United States. Americans also construct ideal distributions that are far more equal than they estimated the United States to beestimates which themselves were far more equal than the actual level of inequality. Second, there was much more consensus than disagreement across groups from different sides of the political spectrum about this desire for a more equal distribution of wealth, suggesting that Americans may possess a commonly held normative standard for the distribution of wealth despite the many disagreements about policies that affect that distribution, such as taxation and welfare (Kluegel & Smith, 1986). We hasten to add, however, that our use of normative is in a descriptive sense reflecting the fact that Americans agree on the ideal distribution but not necessarily in a prescriptive sense. Although some evidence suggests that economic inequality is associated with decreased well-being and health (Napier & Jost, 2008; Wilkinson & Pickett, 2009), creating a society with the precise level of inequality that our respondents report as ideal may not be optimal from an economic or public policy perspective (Krueger, 2004).

11PF4-Income Disparities www.victorybriefs.com

Page 91 of 141

BECAUSE WEALTH HAS DECREASING MARGINAL UTILITY, MORE EGALITARIAN DISTRIBUTIONS WILL INCREASE THE AMOUNT OF AGGREGATE HAPPINESS Frank Stilwell [Prof. of Political Economy at the University of Sydney], Why bother about economic inequality? Evatt Foundation, 8 July 2003. <http://www.evatt.org.au/papers/whybother-about-economic-inequality.html> Ultimately we have to question what is the point of further affluence for the already affluent. There is growing evidence that the relationship between income and happiness is generally non-linear. Rising out of poverty reliably enhances personal happiness. However, in an affluent society, further increases in incomes produce no similarly dependable improvements in subjective wellbeing. Indeed, it is pertinent to recall the argument of the conservative economist A. C. Pigou, who noted that the principle of diminishing marginal utility implies that taking a dollar from a rich person and giving it to a poor person will normally reduce the former's welfare by less than it adds to the latter's. Ergo, a more egalitarian distribution of a given total income enhances economic welfare.

11PF4-Income Disparities www.victorybriefs.com

Page 92 of 141

THERE IS DRAMATIC AND PROGRESSIVE INCOME INEQUALITY IN THE UNITED STATES Joseph E. Stiglitz [Prof. of Economics, Columbia University, 2002 Winner of the Nobel Memorial Prize in Economic Sciences], Of the 1%, by the 1%, for the 1%, Vanity Fair, May 2011, <http://www.vanityfair.com/society/features/2011/05/top-one-percent-201105> Its no use pretending that what has obviously happened has not in fact happened. The upper 1 percent of Americans are now taking in nearly a quarter of the nations income every year. In terms of wealth rather than income, the top 1 percent control 40 percent. Their lot in life has improved considerably. Twenty-five years ago, the corresponding figures were 12 percent and 33 percent. One response might be to celebrate the ingenuity and drive that brought good fortune to these people, and to contend that a rising tide lifts all boats. That response would be misguided. While the top 1 percent have seen their incomes rise 18 percent over the past decade, those in the middle have actually seen their incomes fall. For men with only high-school degrees, the decline has been precipitous12 percent in the last quarter-century alone. All the growth in recent decadesand morehas gone to those at the top. In terms of income equality, America lags behind any country in the old, ossified Europe that President George W. Bush used to deride. Among our closest counterparts are Russia with its oligarchs and Iran. While many of the old centers of inequality in Latin America, such as Brazil, have been striving in recent years, rather successfully, to improve the plight of the poor and reduce gaps in income, America has allowed inequality to grow.

11PF4-Income Disparities www.victorybriefs.com

Page 93 of 141

NEGATIVE EVIDENCE
WE NATURALLY BENEFIT FROM THE UNEQUAL ENDOWMENTS OF NATURE; IT IS NOT GOVERNMENTS JOB TO TRY TO FIX THEM Milton Friedman [Prof. at University of Chicago; Nobel Prize Winner in Economics] and Rose Friedman [Prof. University of Chicago Law School], Free to Choose: A Personal Statement, New York: Harcourt Brace & Company (1980), p. 136-137 Much of the moral fervor behind the drive for equality of outcome comes from the widespread belief that it is not fair that some children should have a great advantage over others simply because they happen to have wealthy parents. Of course it is not fair. However, unfairness can take many forms. It can take the form of inheritance of property bonds and stocks, houses, factories; it can also take the form of the inheritance of talent musical ability, strength, mathematical genius. The inheritance of property can be interfered with more readily than the inheritance of talent. But from an ethical point of view, is there any difference between the two? Yet many people resent the inheritance of property but not the inheritance of talent. Look at the issue from the point of view of the parent. If you want to assure your child a higher income in life, you can do so in various ways. You can buy him (or her) an education that will equip him to pursue an occupation yielding a higher income; or you can set him up in a business that will yield a higher income than he could earn as a salaried employee; or you can leave him property, the income from which will enable him to live better. Is there any ethical difference among these three ways of using your property? Or again, if the state leaves you any money to spend over and above taxes, should the state permit you to spend it on riotous living but not to leave it to your children? The ethical issues involved are subtle and complex. They are not to be resolved by such simplistic formulas as fair shares for all. Indeed, if we took that seriously, youngsters with less musical skill should be given the greatest amount of musical training in order to compensate for their inherited disadvantage, and those with greater musical aptitude should be prevented from having access to good musical training; and similarly with all other categories of inherited personal qualitieis. That might be fair to the youngsters lacking in talent, but would it be fair to the talented, let alone to those who had to work to pay for training the youngsters lacking talent, or to the persons deprived of the benefits that might have come from the cultivation of the talents of the gifted? Life is not fair. It is tempting to believe that government can rectify what nature has spawned. But it is also important to recognize how much we benefit from the very unfairness we deplore. Theres nothing fair about Marlene Dietrichs having been born with beautiful legs that we all want to look at; or about Muhammad Alis having been born with the skill tthat made him a great fighter. But on the other side, millions of people who had enjoyed looking at Merlene Dietrichs legs or watching one of Muhammad Alis fights have benefited from natures unfairness in producing a Marlene Dietrich and a Muhammad Ali. What kind of a world would it be if everyone were a duplicate of everyone else?

11PF4-Income Disparities www.victorybriefs.com

Page 94 of 141

A FREE MARKET SYSTEM RESPECTS THE FACT THAT PEOPLE ARE RESPONSIBLE FOR THEIR CHOICES Milton Friedman [Prof. at University of Chicago; Nobel Prize Winner in Economics] and Rose Friedman [Prof. University of Chicago Law School], Free to Choose: A Personal Statement, New York: Harcourt Brace & Company (1980), p. 136-137 Still another facet of this complex issue of fairness can be illustrated by considering a game of chance, for example, an evening at baccarat. The people who choose to play may start the evening with equal piles of chips, but as the play progresses, those piles will become unequal. By the end of the evening, some will be big winners, others big losers. In the name of the ideal of equality, should the winners be required to repay the losers? That would take all the fun out of the game. Not even the losers would like that. They might like it for the one evening, but would they come back again to play if they knew that whatever happened, theyd end up exactly where they started? This example has a great deal more to do with the real world than one might at first suppose. Every day each of us makes decisions that involve taking a chance. Occasionally its a big chance as when we decide what occupation to pursue, whom to marry, whether to buy a house or make a major investment. More often its a small chance, as when we decide what movie to go to, whether to cross the street against the traffic, whether to buy one security rather than another. Each time the question is, who is to decide what chances we take? That in turn depends on who bears the consequences of the decision. If we bear the consequences, we can make the decision. But if someone else bears the consequences, should we or will we be permitted to make the decision? If you play baccarat as an agent for someone else with his money, will he, or should he, permit you unlimited scope for decision-making? Is he not almost certain to set some limit to your discretion? Will he not lay down some rules for you to observe? To take a very different example, if the government (i.e., your fellow taxpayers) assumes the cost of flood damage to your house, can you be permitted to decide freely whether to build you house on a floodplain? It is no accident that increasing government intervention into personal decisions has gone hand in hand with the drive for fair shares for all. The system under which people make their own choices and bear most of the consequences of their decisions is the system that has prevailed for most of our history. It is the system that gave the Henry Fords, the Thomas Alva Edisons, the George Eastmans, the John D. Rockefellers, the James Cash Penneys the incentive to transform our society over the past two centuries. It is the system that gave other people an incentive to furnish venture capital to finance the risky enterprises that these ambitious inventors and captains of industry undertook. Of course, there were many losers along the way probably more losers than winners. We dont remember their names. But for the most part they went in with their eyes open. They knew they were taking chances. And win or lose, society as a whole benefited from their willingness to take a chance.

11PF4-Income Disparities www.victorybriefs.com EXTENDED SOCIETY CANNOT BE CENTRALLY PLANNED

Page 95 of 141

F. A. Hayek [Nobel Prize Winner in Economics], The Fatal Conceit: The Errors of Socialism, Chicago: The University of Chicago Press (1988), p. 75-76 First, there is the question of how our knowledge really does arise. Most knowledge and I confes it took me some time to recognise this is obtained not from immediate experience or observation, but in the continuous process of sifting a learnt tradition, which requires individual recognition and following of moral traditions that are not justifiable in terms of the canons of traditional theories of rationality. The tradition is the product of a process of selection from among irrational, or, rather, unjustified beliefs which, without anyones knowing or intending it, assisted the proliferation of those who followed them (with no necessary relationship to the reasons as for example religious reasons for which they were followed). The process of selection that shaped customs and morality could take account of more factual circumstances than individuals could perceive, and in consequence tradition is in some respects superior to, or wiser than, human reason (see chapter one above). This decisive insight in one that only a very critical rationalist could recognise. Second, and closely related to this, there is the question raised earlier of what, in the evolutionary selection of rules of conduct, is really decisive. The immediately perceived effects of actions that humans tend to concentrate on are fairly unimportant to this selection; rather, selection is made according to the consequences of the decisions guided by the rules of conduct in the long run the same long run sneered at by Keynes (1971), C.W.: IV, 65). These consequences depend as argued above and discussed again below chiefly on rules of property and contract securing the personal domain of the individual. Hume had already noticed this, writing that these rules are not derived from any utility or advantage which either the particular person or the public may reap from his enjoyment of any particular good (1739/1886:II, 273). Men did not foresee the benefits of rules before adopting them, though some people gradually have become aware of what they owe to the whole system. Our earlier claim, that acquired traditions serve as adaptations to the unknown, must then be taken literally. Adaptation to the unknown is the key in all evolution, and the totality of events to which the modern market order constantly adapts itself is indeed unknown to anybody. The information that individuals or organizations can use to adapt to the unknown is necessarily partial, and is conveyed by signals (e.g., prices) through long chains of individuals, each person passing on in modified form a combination of streams of abstract market signals. Nonetheless, the whole structure of activities tends to adapt, through these partial and fragmentary signals, to conditions foreseen by and known to no individual, even if this adaptation is never perfect. That is why this structure survives, and why those who use it also survive and prosper. There can be no deliberately planned substitutes for such a self-ordering process of adaptation to the unknown. Neither his reason nor his innate natural goodness leads man this way, only the bitter necessity of submitting to rules he does not like in order to maintain himself against competing groups that had already begun to expand because they stumbled upon such rules earlier. If we had deliberately built, or were consciously shaping, the structure of human action, we would merely have to ask individuals why they had interacted with any particular structure. Whereas, in fact, specialised students, even after generations of effort, find it exceedingly difficult to explain such matters, and cannot agree on what are the causes or what will be the effects of particular events. The curious task of economics is to demonstrate to men how little they really know about what they imagine they can design.

11PF4-Income Disparities www.victorybriefs.com

Page 96 of 141

PRIVATE PROPERTY FACILITATES COORDINATION IN A SOCIETY WHERE EACH OF US ONLY HAS PARTIAL KNOWLEDGE, AND THUS FACILITATES SERVING THE NEEDS OF DISTANT OTHERS F. A. Hayek [Nobel Prize Winner in Economics], The Fatal Conceit: The Errors of Socialism, Chicago: The University of Chicago Press (1988), p. 77-78 Much of the particular information which any individual possesses can be used only to the extent to which he himself can use it in his own decisions. Nobody can communicate to another all that he knows, because much of the information he can make use of he himself will elicit only in the process of making plans for action. Such information will be evoked as he works upon the particular task he has undertaken in the conditions in which he finds himself, such as the relative scarcity of various materials to which he has access. Only thus can the individual find out what to look for, and what helps him to do this in the market is the responses others make to what they find in their own environments. The overall problem is not merely to make use the given knowledge, but to discover as much information as is worth searching for in prevailing conditions. It is often objected that the institution of property is selfish in that it benefits only those who own some, and that is was indeed invested by some persons who, having acquired some individual possessions, wished for their exclusive benefit to protect these from others. Such notions, which of course underlie Rousseaus resentment, and his allegation that our shackles have been imposed by selfish and exploitative interests, fail to take into account that the size of our overall product is so large only because we can, through market exchange of severally owned property, use widely dispersed knowledge of particular facts to allocate severally owned resources. The market is the only known method of providing information enabling individuals to judge comparative advantages of different uses of resources of which they have immediate knowledge and through whose use, whether they so intend or not, they serve the needs of distant unknown individuals. This dispersed knowledge is essentially dispersed, and cannot possibly be gathered together and conveyed to an authority charged with the task of deliberately creating order. Thus the institution of several property is not selfish, nor was it, nor could it have been, invented to impose the will of property owners upon the rest. Rather, it is generally beneficial in that it transfers the guidance of production from the hands of a few individuals who, whatever they may pretend, have limited knowledge, to a processes, the extended order, that makes maximum use of the knowledge of all, thereby benefiting those who do not own property nearly as much as those who do.

11PF4-Income Disparities www.victorybriefs.com OUR INSTINCTUAL ALTRUISM ACTUALLY HARMS THOSE IN NEED

Page 97 of 141

F. A. Hayek [Nobel Prize Winner in Economics], The Fatal Conceit: The Errors of Socialism, Chicago: The University of Chicago Press (1988), p. 80-81 Fifth, there is the question whence then, in the presence of all these difficulties and objections, the demand to restrict ones action to the deliberate pursuit of known and observable beneficial ends arises. It is in part a remnant of the instinctual, and cautious, micro-ethic of the small band, wherein jointly perceived purposes were directed to the visible needs of personally known comrades (i.e., solidarity and altruism). Earlier I claimed that, within an extended order, solidarity and altruism are possible only in a limited way within some sub-groups, and that to restrict the behaviour of the group at large to such action would work against coordinating the efforts of its members. Once most of the productive activities members of a cooperating group transcend the range of the individuals perception, the old impulse to follow inborn altruistic instincts actually hinders the formation of more extensive orders. In the sense of inculcating conduct that benefits others, all systems of morality of course commend altruistic action; but the question is how to accomplish this. Good intentions will not suffice we all know what road they pave. Guidance strictly by perceivable favourable effects on particular other persons is insufficient for, and even irreconcilable with, the extended order. The morals of the market do lead us to benefit others, not by our intending to do so, but by making us act in a manner which, nonetheless, will have just that effect. The extended order circumvents individual ignorance (and thus also adapts us to the unknown, as discussed above) in a way that good intentions alone cannot do and thereby does make our efforts altruistic in their effects. In an order taking advantage of the higher productivity of extensive division of labour, the individual can no longer know whose needs his efforts do or ought to serve, or what will be the effects of his actions on those unknown persons who do consume his products or products to which he has contributed. Directing his productive efforts altruistically thus becomes literally impossible for him. In so far as we can still call his motives altruistic in that they eventually redound to the benefit of others, they will do this not because he aims at or intends to serve the concrete needs of others, but because he observes abstract rules. Our altruism, in this new sense, is very different form instinctual altruism. No longer the end pursued but the rules observed make the action good or bad. Observing these rules, while bending most of our efforts towards earning a living, enables us to confer benefits beyond the range of our concrete knowledge (yet at the same time hardly prevents us from using whatever extra we earn also to gratify our instinctive longing to do visible good). All this is obscured by the systematic abuse of the term altruistic by sociobiologists.

11PF4-Income Disparities www.victorybriefs.com EFFICIENT MARKETS MAKE EVERYONE BETTER OFF

Page 98 of 141

F. A. Hayek [Nobel Prize Winner in Economics], The Fatal Conceit: The Errors of Socialism, Chicago: The University of Chicago Press (1988), p. 84-85 Some persons are so troubled by some effects of the market order that they overlook how unlikely and even wonderful it is to find such an order prevailing in the greater part of the modern world, a world in which we find thousands of millions of people working in a constantly changing environment, providing means of subsistence for others who are mostly unknown to them, and at the same time finding satisfied their own expectations that they themselves will receive goods and services produced by equally unknown people. Even in the worst of times something like nine out of ten of them will find these expectations confirmed. Such an order, although far from perfect and often inefficient, can extend farther than any order men could create by deliberately putting countless elements into selected appropriate places. Most defects and inefficiencies of such spontaneous orders result from attempting to interfere with or to prevent their mechanisms from operating, or to improve the details of their results. Such attempts to intervene in spontaneous order rarely result in anything closely corresponding to mens wishes, since these orders are determined by more particular facts than any such intervening agency can know. Yet, while deliberate intervention to, say, flatten out inequalities in the interest of a random member of the order risks damaging the working of the whole, the selfordering process will secure for any random member of such a group a better chance over a wider range of opportunities available to all than any rival system could offer.

11PF4-Income Disparities www.victorybriefs.com CENTRAL PLANNING IS BOUND TO FAIL

Page 99 of 141

F. A. Hayek [Nobel Prize Winner in Economics], The Fatal Conceit: The Errors of Socialism, Chicago: The University of Chicago Press (1988), p. 86 Meanwhile, among those who, in the tradition of Mandeville, Hume, and Smith, did study economics, there gradually emerged not only an understanding of market processes, but a powerful critique of the possibility of substituting socialism for them. The advantages of these market procedures were so contrary to expectation that they would be explained only retrospectively, through analyzing this spontaneous formation itself. When this was done, ti was found that decentralised control over resources, control through several property, leads to the generation and use of more information than is possible under central direction. Order and control extending beyond the immediate purview of any central authority could be attained by central direction only if, contrary to fact, those local managers who could gauge visible and potential resources were also currently informed of the constantly changing relative importance of such resources, and could then communicate full and accurate details about this to some central planning authority in time for it to tell them what to do in light of all the other, different, concrete information it had received from other regional or local managers who of course, in turn, found themselves in similar difficulties in obtaining and delivering any such information. Once we realise what the task of such a central planning authority would be, it becomes clear that the commands it would have to issue could not be derived from the information the local managers hd recognised as important, but could only be determined through direct dealings among individuals or groups controlling clearly delimited aggregates of means. The hypothetical assumption, customarily employed in theoretical descriptions of the market process (descriptions made by people who usually have no intention of supporting socialism), to the effect that all such facts (or parameters) can be assumed to be known to the explaining theorist, obscures all this, and consequently produces the curious deceptions that help to sustain various forms of socialist thinking.

11PF4-Income Disparities www.victorybriefs.com

Page 100 of 141

SOME UTILITARIANS OBJECT TO REDISTRIBUTION OF WEALTH ON THE GROUNDS THAT IT DISINCENTIVES PRODUCTIVITY Michael Sandel [Prof. of Government, Harvard University], Justice: Whats The Right Thing To Do?, New York: Farrar, Straus and Giroux, p. 59 This Robin Hood scenario is open to at least two objections one from within utilitarian thinking, the other from outside it. The first objection worries that high tax rates, especially on income, reduce the incentive to work and invest, leading to a decline in productivity. If the economic pie shrinks, leaving less to redistribute, the overall level of utility might go down. So before taxing Bill Gates and Oprah Winfrey too heavily, the utilitarian would have to ask whether doing so would lead them to work less and so to earn less, eventually reducing the amount of money available for redistribution to the needy.

11PF4-Income Disparities www.victorybriefs.com

Page 101 of 141

DEVELOPED DEMOCRACIES ARE FACING A CRISIS IN THE LONG-TERM FISCAL SUSTAINABILITY OF SOCIAL PROGRAMS Francis Fukuyama [Olivier Nomellini Senior Fellow at the Freeman Spogli Institute for International Studies, Stanford University], Dealing with Inequality, Journal of Democracy Volume 22, Number 3 (July 2011), p. 87 On the other hand, rich developed democracies with far more extensive welfare states are all facing a long-term crisis of fiscal sustainability. Existing social contracts regarding welfare-state benefits were written in an age in which birth rates were far higher and life expectancies much lower than they are today. Even in countries that have a less generous level of public benefits, such as the United States and Japan, underlying demographic realities and the spiraling costs of health care have put national finances on a fiscally unsustainable path, with high debt-to-GDP ratios that will steadily worsen in the absence of remedial action.24 All modern democracies are plagued with the problem of interest groups whose stake in protecting a certain part of the budget is much stronger than the diffuse general interest in long-term fiscal sustainability. The difficulties that the French government recently had in raising the retirement age from 60 to 62, as well as the deadlock that currently exists in the U.S. Congress with regard to entitlement spending, are indicative of the problem.

11PF4-Income Disparities www.victorybriefs.com

Page 102 of 141

INCREASING ECONOMIC EFFICIENCY IN CONSUMER GOODS MARKETS HAS BENEFITED LESS AFFLUENT AMERICANS MORE THAN THE RICH James Pethokoukis [Money & Politics columnist and blogger for Reuters], 5 reasons why income inequality is a myth and Occupy Wall Street is wrong, The American, 18 Oct. 2011 < http://blog.american.com/2011/10/5-reasons-why-income-inequality-is-a-myth-and-occupy-wallstreet-is-wrong/> 3. A 2008 paper by Christian Broda and John Romalis from the University of Chicago documents how traditional measures of inequality ignore how inflation affects the rich and poor differently: Inflation of the richest 10 percent of American households has been 6 percentage points higher than that of the poorest 10 percent over the period 19942005. This means that real inequality in America, if you measure it correctly, has been roughly unchanged. And why is that? China and Wal-Mart. Lower-income families spend a larger share of income than wealthier families on goods whose prices are more directly affected by trade. Higher income folks, by contrast, spend more on services which are less subject to foreign competition.

11PF4-Income Disparities www.victorybriefs.com

Page 103 of 141

ECONOMIC INEQUALITY HAS BEEN DRIVEN BY GROWTH IN THE FINANCIAL SECTOR, NOT MARKET FAILURE Tyler Cohen [Prof. of Economics, George Mason University], The Inequality That Matters, The American Interest, January/February 2011, < http://www.the-americaninterest.com/article.cfm?piece=907> The use of micro-data now makes it possible to trace some high earners by income and thus construct a partial picture of what is going on among the upper echelons of the distribution. Steven N. Kaplan and Joshua Rauh have recently provided a detailed estimation of particular American incomes.6 Their data do not comprise the entire U.S. population, but from partial financial records they find a very strong role for the financial sector in driving the trend toward income concentration at the top. For instance, for 2004, nonfinancial executives of publicly traded companies accounted for less than 6 percent of the top 0.01 percent income bracket. In that same year, the top 25 hedge fund managers combined appear to have earned more than all of the CEOs from the entire S&P 500. The number of Wall Street investors earning more than $100 million a year was nine times higher than the public company executives earning that amount. The authors also relate that they shared their estimates with a former U.S. Secretary of the Treasury, one who also has a Wall Street background. He thought their estimates of earnings in the financial sector were, if anything, understated. Many of the other high earners are also connected to finance. After Wall Street, Kaplan and Rauh identify the legal sector as a contributor to the growing spread in earnings at the top. Yet many high-earning lawyers are doing financial deals, so a lot of the income generated through legal activity is rooted in finance. Other lawyers are defending corporations against lawsuits, filing lawsuits or helping corporations deal with complex regulations. The returns to these activities are an artifact of the growing complexity of the law and government growth rather than a tale of markets per se. Finance aside, there isnt much of a story of market failure here, even if we dont find the results aesthetically appealing.

11PF4-Income Disparities www.victorybriefs.com TRANSFERING WEALTH HAS SUBSTANTIAL TRANSACTION COSTS

Page 104 of 141

Richard A. Epstein [Prof. of Law, NYU Law School], Three Cheers for Income Inequality, Defining Ideas, 8 Nov. 2011, <http://www.hoover.org/publications/defining-ideas/article/99241> First, the use of state coercion to remedy inequalities of wealth is not easily done. The most obvious method for doing so is by creating subsidies for people at the bottom, which are offset by high rates of taxation for people at the top. The hope is that high taxes will do little to blunt economic activity at the high end, while the payments will do little to dull initiative at the low end. But this program is much more difficult to implement than is commonly supposed. The process of income redistribution opens up opportunities for powerful groups to secure transfers of wealth to themselves. This does nothing to redress inequalities of wealth. Even if these political players are constrained, there is still no costless way to transfer wealth up and down the income scale.

11PF4-Income Disparities www.victorybriefs.com

Page 105 of 141

A PROGRESSIVE INCOME TAX CREATES COMPLEXITY BECAUSE OF PRIVATE TAX EVASION STRATEGIES Richard A. Epstein [Prof. of Law, NYU Law School], Three Cheers for Income Inequality, Defining Ideas, 8 Nov. 2011, <http://www.hoover.org/publications/defining-ideas/article/99241> The administrative costs of running a progressive income tax system are legion. Unfortunately, that point was missed in a recent op-ed. Writing in the New York Times, Cornell economist Robert H. Frank plumped hard for steeper progressive income tax rates as a way to amend income inequality. Yet matters are not nearly as simple as he supposes. In his view, the source of complexity in the current income tax code lies in the plethora of special interest provisions that make it difficult to calculate income by recognized standard economic measures. Thus, he thinks that it is flatly wrong to think that the flat tax will result in tax simplification. After all, it is just as easy to read a tax schedule that has progressive rates as one that has a uniform flat rate. But more than reading tax schedules is at stake. First, one reason why the internal revenue code contains such complexity is its desire to combat the private strategies that people, especially those in the top one percent, use to avoid high levels of taxation. Anyone who has spent time in dealing with family trusts and partnerships, with income averaging, with the use of real estate shelters, and with foreign investments, knows just how hard it is to protect the progressive rate schedule against manipulation. Second, the creation of these large tax loopholes is not some act of nature. Frank, like so many defenders of progressive taxation, fails to realize that progressive rates generate huge pressures to create new tax shelters. Lower the overall tax rates and the pressure to create tax gimmicks with real economic costs diminishes. Overall social output is higher with a flat tax than it is with a progressive one.

11PF4-Income Disparities www.victorybriefs.com

Page 106 of 141

FLAT TAXES REMOVE THE IMPRECISION IN DECIDING HOW STEEPLY PROGRESSIVE THE INCOME TAX SHOULD BE Richard A. Epstein [Prof. of Law, NYU Law School], Three Cheers for Income Inequality, Defining Ideas, 8 Nov. 2011, <http://www.hoover.org/publications/defining-ideas/article/99241> Third, the dangers posed by the use of progressive taxation are not confined to these serious administrative issues. There are also larger questions of political economy at stake. The initial question is just how steep the progressive tax ought to be. Keep it too shallow, and it does little to generate additional public revenues to justify the added cost of administration. Make it too steep, and it will reduce the incentives to create wealth that are always unambiguously stronger under a flat tax system. But since no one knows the optimal level of progressivity, vast quantities of wealth are dissipated in fighting over these levels. The flat tax removes that dimension of political intrigue.

11PF4-Income Disparities www.victorybriefs.com HIGH TAX RATES KILL ECONOMIC GROWTH

Page 107 of 141

Richard A. Epstein [Prof. of Law, NYU Law School], Three Cheers for Income Inequality, Defining Ideas, 8 Nov. 2011, <http://www.hoover.org/publications/defining-ideas/article/99241> Fourth, sooner or laterand probably soonerhigh tax rates will kill growth. Progressives like Frank operate on the assumption that high taxation rates have little effect on investment by asking whether anyone would quit a cushy job just to save a few tax dollars. But the situation is in reality far more complex. One key to success in the United States lies in its ability to attract foreign labor and foreign capital to our shores. In this we are in competition with other nations whose tax policies are far more favorable to new investment than ours. The loss of foreign people and foreign capital is not easy to observe because we cannot identify with certainty most of the individuals who decide to go elsewhere. But we should at the very least note that there is the risk of a brain drain as the best and brightest foreign workers who came to the United States in search of economic opportunity ultimately may return home. They will likely not want to brave the hostile business climate that they see in the United States.

11PF4-Income Disparities www.victorybriefs.com

Page 108 of 141

IN A COMPLICATED TAX SYSTEM DEISGNED TO REDISTRIBUTE WEALTH, THE AFFLUENT ARE ABLE TO EMPLOY SOPHISTICATED TAX AVOIDANCE STRATEGIES Richard A. Epstein [Prof. of Law, NYU Law School], Three Cheers for Income Inequality, Defining Ideas, 8 Nov. 2011, <http://www.hoover.org/publications/defining-ideas/article/99241> Fifth, sophisticated forms of tax avoidance are not limited to foreign laborers. Rich people have a choice of tax-free and taxable investments. They can increase transfers to family members in order to reduce the incidence of high progressive taxation. They can retire a year sooner, or go part-time to reduce their tax burdens. And of course, they can fight the incidence of higher taxation by using their not inconsiderable influence in the tax arenas.

11PF4-Income Disparities www.victorybriefs.com

Page 109 of 141

REDISTRIBUTIVE TAX SYSTEMS CAN BE UNDERMINED WHEN SYSTEMIC INEFFICIENCIES COMPROMISE THE TAX BASE ON WHICH THE SYSTEM DEPENDS Richard A. Epstein [Prof. of Law, NYU Law School], Three Cheers for Income Inequality, Defining Ideas, 8 Nov. 2011, <http://www.hoover.org/publications/defining-ideas/article/99241> Sixth, the inefficiencies created by a wide range of tax and business initiatives reduces the wealth earned by people in that top one percent, and thus the tax base on which the entire redistributive state depends. Defenders of progressive taxation, like Frank, cite the recent report of the Congressional Budget Office, which shows huge increases of wealth in the top one percent from 1979 to 2007. The top one percent increased its wealth by 275 percent in those years. The rest of the income distribution lagged far behind. Unfortunately, the CBO report was out of date the day it was published. We now have tax data available that runs through 2009, which shows the folly of seeking to rely on heavier rates of taxation on the top one percent. The Tax Foundations October 24, 2011 report, contains this solemn reminder of the risks of soaking the rich in bad times: In 2009, the top 1 percent of tax returns paid 36.7 percent of all federal individual income taxes and earned 16.9 percent of adjusted gross income (AGI), compared to 2008 when those figures were 38.0 percent and 20.0 percent, respectively. Both of those figures share of income and share of taxes paidwere their lowest since 2003 when the top 1 percent earned 16.7 percent of adjusted gross income and paid 34.3 percent of federal individual income taxes. It is worth adding that the income of the top one percent also dropped 20 percent between 2007 and 2008, with a concomitant loss in tax revenues. There are several disturbing implications that flow from this report. The first is that these figures explain the vulnerability in bad times of our strong dependence on high-income people to fund the transfer system. The current contraction in wealth at the top took place with only few new taxes. The decline in taxable income at the top will only shrink further if tax rates are raised. A mistake, therefore, in setting tax rate increases could easily wreck the entire system. Indeed, the worst possible outcome would be for high taxation to lower top incomes drastically. Right now, for better or worse, the entire transfer system of the United States is dependent on the continued success of high-income earners whom the egalitarians would like to punish. Put otherwise, if a person at the middle of the income distribution loses a dollar in income, the federal government loses nothing in income tax revenues. Let a rich person suffer that decline and the revenue loss at the federal level is close to 40 percent, with more losses at the state level. The slow growth policies of the last three years have cost far more in revenue from the top one percent than any increase in progressive taxation could possibly hope to achieve. The more we move toward an equal income policy, the more we shall need tax increases on the middle class to offset the huge revenue losses at the top. Our current political economy makes the bottom 99 percent hostage to the continued success of the rich.

11PF4-Income Disparities www.victorybriefs.com

Page 110 of 141

THE THEORETICAL BASIS OF LIBERAL DEMOCRACY REQUIRES PROTECTION OF THE RIGHT TO PROPERTY Francis Fukuyama [Olivier Nomellini Senior Fellow at the Freeman Spogli Institute for International Studies, Stanford University], Dealing with Inequality, Journal of Democracy Volume 22, Number 3 (July 2011), p. 80-81 The early theorists of liberal democracy were concerned with political equality, but either paid little attention to economic equality or else assumed that it would continue to exist. Thomas Hobbes in Leviathan (1651) asserts that human beings are equal because any given human being can kill any other human being, and that all humans are equally driven by the fear of violent death. This leads him to derive a right to life as a right of nature that humans seek to preserve by entry into civil society (and is the progenitor of the rights to life, liberty, and the pursuit of happiness proclaimed in the U.S. Declaration of Independence). John Locke, in his Second Treatise on Government (1689), views human nature as both gentler and more focused on a universal desire to acquire property. Like Hobbes, Locke emphasizes the equality of political rights, but goes on to acknowledge that there are important differences between the industrious and rational and the quarrelsome and contentious in their ability to mix their labor with the mere things of Nature and turn them into private property. Thus, while Lockes liberal state protects equally the right to acquire property, it does not assume that there will be anything like equality of actual outcomes with regard to wealth. Indeed, his stress on the importance of property implies that the industrious and rational will need political protection of their wealth, both from the state and from others who might unjustly take it away from them. U.S. founding father James Madison revisits the latter theme in Federalist 10 (1787), in which he argues that the first object of government is the protection of the unequal ability to acquire property. More recent liberal theorists such as Friedrich A. Hayek are similarly insistent that the preservation of a free society entails acceptance of potentially high degrees of economic inequality. In The Constitution of Liberty (1960), Hayek defines freedom in very narrow terms as an absence of compulsionin particular, compulsion by a centralized state.3 He rejects metaphysical definitions of freedom as freedom of the will, and assumes that the absence of compulsion will be sufficient to provide space for the exercise of individual choice. While Hayek is not rigidly opposed to government provision of poor relief or health insurance, he is strongly critical of formal schemes of redistribution such as the progressive income tax. His opposition does not stem from utilitarian concerns about the effects of progressive taxation on economic growth; rather, he argues that such a tax represents an illegitimate exercise of government power over individuals because it violates the principle of equal legal protection. Yet this seeming stand on principle is in fact based on a certain kind of consequentialism on Hayeks part, since he also believes that once government gets into the business of coercive redistribution, it is more likely than not to increase its role over time.4

11PF4-Income Disparities www.victorybriefs.com

Page 111 of 141

LIBERTARIANS OBJECT TO REDISTRIBUTION OF WEALTH ON THE GROUNDS THAT DOING SO VIOLATES A FUNDAMENTAL RIGHT TO LIBERTY Michael Sandel [Prof. of Government, Harvard University], Justice: Whats The Right Thing To Do?, New York: Farrar, Straus and Giroux, p. 59-60 The second objection regards these calculations as beside the point. It argues that taxing the rich to help the poor is unjust because it violates a fundamental right. According to this objection, taking money from Gates and Winfrey without their consent, even for a good cause, is coercive. It violates their liberty to do with their money whatever they please. Those who object to redistribution on these grounds are often called libertarians. Libertarians favor unfettered markets and oppose government regulation, not in the name of economic efficiency but in the name of human freedom. Their central claim is that each of us has a fundamental right to liberty the right to do whatever we want with the things we own, provided we respect other peoples rights to do the same.

11PF4-Income Disparities www.victorybriefs.com

Page 112 of 141

LIBERTARIANS OPPOSE REDISTRIBUTION OF WEALTH OR INCOME IN ANY FORM Michael Sandel [Prof. of Government, Harvard University], Justice: Whats The Right Thing To Do?, New York: Farrar, Straus and Giroux, p. 60-61 3. No Redistribution of Income or Wealth. The libertarian theory of rights ruels out any law that requires some people to help others, including taxation for redistribution of wealth. Desirable though it may be for the affluent to support the less fortunate by subsidizing their health care or housing or education such help should be left up to the individual to undertake, not mandated by the government. According to the libertarian, redistributive taxes are a form of coercion, even theft. The state has no more right to force affluent taxpayers to support social programs for the poor than a benevolent thief has the right to steal money from a rich persona nd give it to the homeless. The libertarian philosophy does not map neatly onto the political spectrum. Conservatives who favor laissez-faire economic policies often part company with libertarians on cultural issues such as school prayer, abortion, and restrictions on pornography. And many proponents of the welfare state hold libertarian views on issues such as gay rights, reproductive rights, freedom of speech, and the separation of church and state. During the 1980s, libertarian ideas found prominent expression in the pro-market, antigovernment rhetoric of Ronald Reagan and Margaret Thatcher. As an intellectual doctrine, libertarianism emerged earlier, in opposition to the welfare state. In The Constitution of Liberty (1960), the Austrian-born economist-philospher Friedrich A. Hayek (1899-1992) argued that any attempt to bring about greater economic equality was bound to be coercive and destructive of a free society. 3 In Capitalism and Freedom (1962), the American economist Milton Friedman (1912-2006) argued that many widely accepted state activities are illegitimate infringements on individual freedom. Social Security, or any mandatory, government run retirement program, is one of his prime examples: If a man knowingly prefers to live for today, to use his resources for current enjoyment, deliberately choosing a penurious old age, by what right do we prevent him from doing so? Friedman asks. We might urge such a person to save for his retirement, but are we entitled to use coercion to prevent him from doing what he chooses to do? 4 Friedman objects to minimum wage laws on similar grounds. Government has no right to prevent employers fom paying any wage, however low, that workers are prepared to accept. The government also violates individual freedom when it makes laws against employment discrimination. If employers want to discriminate on the basis of race, religion, or any other factor, the state has no right to prevent them from doing so. In Friedmans view, such legislation clearly involves interference with the freedom of individuals to enter into voluntary contracts with one another. 5

11PF4-Income Disparities www.victorybriefs.com

Page 113 of 141

LIBERTARIANS ARGUE THAT THERE IS NOTHING INHERENTLY WRONG WITH ECONOMIC INEQUALITY SO LONG AS WEALTH IS PROCURED LEGITIMATELY Michael Sandel [Prof. of Government, Harvard University], Justice: Whats The Right Thing To Do?, New York: Farrar, Straus and Giroux, p. 62-63 According to Nozick, there is nothing wrong with economic inequality as such. Simply knowing that the Forbes 400 have billions while others are penniless doesnt enable you to conclude anything about the justice or injustice of the arrangement. Nozick rejects the idea that a just distribution consists of a certain pattern such as equal income, or equal utility, or equal provision of basic needs. What matters Is how the distribution came about. Nozick rejects patterned theories of justice in favor of those that honor the choices peoples make in free markets. He argues that distributive justice depends on two requirements justice in initial holdings and justice in transfer. 3 The first asks if the resources you used to make your money were legitimately yours in the first place. (If you made a fortune selling stolen goods, you would not be entitled to the proceeds.) The second aks if you made your money either through free exchanges in the marketplace or from gifts voluntarily bestowed upon you by others. If the answer to both questions is yes, you are entitled to what you have, and the state may not take it without your consent. Provided no one starts out with ill-gotten gains, any distribution that results from a free market is just, however equal or unequal it turns out to be.

11PF4-Income Disparities www.victorybriefs.com

Page 114 of 141

ONE IS ENTITLED TO PROPERTY IF IT WAS JUSTLY OBTAINED AND JUSTLY TRANSFERRED Robert Nozick [Prof., Harvard University], Anarchy, State, and Utopia, New York: Basic Books, Inc., Publishers (1974), p. 151-152 If the world were wholly just, the following inductive definition would exhaustively cover the subject of justice in holdings. 1. A person who acquires a holding in accordance with the principle of justice in acquisition is entitled to that holding. 2. A person who acquires a holding in accordance with the principles of justice in transfer, from someone else entitled to the holding, is entitled to the holding. 3. No one is entitled to a holding except by (repeated) applications of 1 and 2. The complete principle of distributive justice would say simply that a distribution is just if everyone is entitled to the holding they possess under the distribution. A distribution is just if it arises from another just distribution by legitimate means. The legitimate means of moving from one distribution to another are specified by the principle of justice in transfer. The legitimate first moves are specified by the principle of justice in acquisition.* Whatever arises from a just situation by just steps is itself just. The means of change specified by the principle of justice in transfer preserve justice. As correct rules of inference are truthpreserving, and any conclusion deduced via repeated application of such rules from only true premises is itself true, so the means of transition from one situation to another specified by the principle of justice in transfer are justice-preserving, and any situation actually arising from repeated transitions in accordance with the principle from a just situation is itself just. The parallel between justice-preserving transformations and truth-preserving transformations illuminates where it fails as well as where it holds. That a conclusion could have been deduced by truthpreserving transformations illuminates where it fails as well as where it holds. That a conclusion could have been deduced by truth-preserving means from premises that are true suffices to show its truth. That from a just situation a situation could have arisen via justice preserving means does not suffice to show its justice. The fact that a thiefs victims voluntarily could have presented him with gifts does not entitle the thief to his ill-gotten gains. Justice in holdings is historical; it depends upon what actually has happened.

11PF4-Income Disparities www.victorybriefs.com

Page 115 of 141

TRYING TO CONSTRUCT A PARTICULAR PATTERN OF THE DISTRIBUTION OF GOODS FAILES TO REALIZE THE HISTORICAL CHARACTER OF ENTITLEMENTS Robert Nozick [Prof., Harvard University], Anarchy, State, and Utopia, New York: Basic Books, Inc., Publishers (1974), p. 151-152 To think that the task of a theory of distributive justice is to fill in the blank in to each according to his _____ is to be predisposed to search for a pattern; and the separate treatment of from each according to his _____ treats production and distribution as two separate and independent issues. On an entitlement view these are not two separate questions. Whoever makes something, having bought or contracted for all other held resourced used in the process (transferring some of his holdings for these cooperating factors), is entitled to it. The situation is not one of somethings getting made, and there being an open question of who is to get it. Things come into the world already attached to people having entitlements over them. From the point of view of the historical entitlement conception of justice in holdings, those who start afresh to complete to each according to his _____ treat objects as if they appeared from nowhere, out of nothing. A complete theory of justice might cover this limit case as well; perhaps here is a use for the usual conceptions of distributive justice. 5

11PF4-Income Disparities www.victorybriefs.com

Page 116 of 141

OUR INTUITIONS TELL US THAT PEOPLE ARE ENTITLED TO WHAT THEY EARN THROUGH FAIR TRANSFERS Robert Nozick [Prof., Harvard University], Anarchy, State, and Utopia, New York: Basic Books, Inc., Publishers (1974), p. 160-162 It is not clear how those holding alternative conceptions of distributive justice can reject the entitlement conception of justice in holdings. For suppose a distribution favored by one of these non-entitlement conceptions is realized. Let us suppose it is your favorite one and let us call this distribution D1; perhaps everyone has an equal share, perhaps shares vary in accordance with some dimension you treasure. Now suppose that Wilt Chamberlain is greatly in demand by basketball teams, being a great gate attraction. (Also suppose contracts run only for a year, with players being free agents.) He signs the following sort of contract with a team: In each home game, twenty-five cents from the price of each ticket of admission goes to him. (We ignore the question of whether he is gouging the owners, letting them look out for themselves). The season starts, and people cheerfully attend his teams games; they buy their tickets, each time dropping a separate twenty-five cents of their admission price into a special box with Chamberlains name on it. They are excited about seeing him play; it is worth the total admission price to them. Let us suppose that in one season one million persons attend his home games, and Wilt Chamberlain winds up with $250,000, a much larger sum than the average income and larger even than anyone else has. Is he entitled to this icome? Is this new distribution D2, unjust? If so, why? There is no question about whether each of the people was entitled to the control over the resources held in D1; because that was the distribution (your favorite) that (for the purposes of argument) we assumed was acceptable. Each of these persons chose to give twenty-five cents of their money to Chamberlain. They could have spent it on going to the movies, or on candy bars, or on copies of Dissent magazine, or of Monthly Review. But they all, at least one million of them, converged on giving it to Wilt Chamberlain in exchange for watching him play basketball. If D1 was a just distribution, and people voluntarily moved from it to D2, transferring parts of their shares they were given under D1 (what was it for if not to do something with?), isnt D2 also just? If the people were entitled to dispose of the resources to which they were entitled (under D1), didnt this include their being entitled to give it to, or exchange it with, Wilt Chamberlain? Can anyone else complain on grounds of justice? Each other person already has his legitimate share under D1. Under D1, there is nothing that anyone has that anyone else has a claim of justice against. After someone transfers something to Wilt Chamberlain, third parties still have their legitimate shares; their shares are not changed. By what process could such a transfer among two persons give rise to a legitimate claim of distributive justice on a portion of what was transferred, by a third party who had no claim of justice on any holding of the others before the transfer?* To cut off objections irrelevant here, we might imagine the exchanges occurring in a socialist society, after hours. After playing whatever basketball he does in his daily work, or doing whatever other daily work he does, Wilt Chamberlain decides to put in overtime to earn additional money. (First his work quota is set; he works time over that.) Or imagine it is a skilled juggler people like to see, who puts on shows after hours.

11PF4-Income Disparities www.victorybriefs.com

Page 117 of 141

DISTRIBUTING RESOURCES TO ATTEMPT TO ACHIEVE SOME DESIRABLE PATTERN REQUIRES CONSTANT REGULATION OF PEOPLES CHOICES Robert Nozick [Prof., Harvard University], Anarchy, State, and Utopia, New York: Basic Books, Inc., Publishers (1974), p. 160-162 The general point illustrated by the Wilt Chamberlain example and the example of the entrepreneur in a socialist society is that no end-state principle or distributional patterned principle of justice can be continuously realized without continuous interference with peoples lives. Any favored pattern would be transformed into one unflavored by principle, by people choosing to act in various ways; for example, by people exchanging goods and services with other people, or giving things to other people, things the transferrers are entitled to under the favored distributional pattern. To maintain a pattern one must either continually interfere to stop people from transferring resources as they wish to, or continually (or periodically) interfere to take from some persons resources that others for some reason chose to transfer to them. (But if some time limit is to be set on how long people may keep resources others voluntarily transfer to them, why let them keep these resources for any period of time? Why not have immediate confiscation?) It might be objected that all persons voluntarily will to refrain from actions which would upset the pattern. This presupposes unrealistically (1) that all will most want to maintain the pattern (are those who dont, to be reeducated or forced to undergo self-criticism?), (2) that each can gather enough information about his own actions and the ongoing activities of others to discover which of his actions will upset the pattern, and (3) that diverse and far-glung persons can coordinate their actions to dovetail into the pattern. Compare the manner in which the market is neutral among persons desires, as it reflects and transmits widely scattered information via prices, and coordinates persons activities.

11PF4-Income Disparities www.victorybriefs.com TAXATION IS ON PAR WITH FORCED LABOR

Page 118 of 141

Robert Nozick [Prof., Harvard University], Anarchy, State, and Utopia, New York: Basic Books, Inc., Publishers (1974), p. 169-171 Taxation of earnings from labor is on par with forced labor.* Some persons find this claim obviously true: taking the earnings of n hours labor is like taking n hours from the person; it is like forcing the person to work n hours for anothers purpose. Others find the claim absurd. But even these, if they object to forced labor, would oppose forcing unemployed hippies to work for the benefit of the needy. And they would also object to forcing each person to work five extra hours each week for the benefit of the needy. But a system that takes five hours wages in taxes does not seem to them like one that forces someone to work five hours, since it offers the person forced a wider range of choice in activities than does taxation in kind with the particular labor specified. (But we can imagine a gradation of systems of forced labor, from one that specifies a particular activity, to one that gives a choice among two activities, to ; and so on up.) Furthermore, people envisage a system with something like a proportional tax on everything above the amount necessary for basic needs. Some think this does not force someone to work extra hours, since there is no fixed number of extra hours he is forced to work, and since he can avoid the tax entirely by earning only enough to cover his basic needs. This is a very uncharacteristic view of forcing for those who also think people are forced to do something whenever the alternatives they face are considerably worse. However, neither view is correct. The fact that others intentionally intervene, in violation of a side constraint against aggression, to threaten force to limit the alternatives, in this case to paying taxes or (presumably the worse alternative) bare subsistence, makes the taxation system one of force labor and distinguishes it from other cases of limited choices which are not forcings. 10

11PF4-Income Disparities www.victorybriefs.com

Page 119 of 141

IT IS ILLEGITIMATE TO SEIZE A PERSONS PROPERTY TO HELP THE NEEDY Robert Nozick [Prof., Harvard University], Anarchy, State, and Utopia, New York: Basic Books, Inc., Publishers (1974), p. 170-171 The man who chooses to work longer to gain an income more than sufficient for his basic needs prefers some extra goods or services to the leisure and activities he could perform during the possible nonworking hours; whereas the man who chooses not to work the extra time prefers the leisure activities to the extra goods or services he could acquire by working more. Given this, if it would be illegitimate for a tax system to seize some of a mans leisure (forced labor) for the purpose of serving the needy, how can it be legitimate for a tax system to seize some of a mans goods for that purpose? Why should we treat the man whose happiness requires certain material goods or services differently from the man whose preferences and desires make such goods unnecessary for his happiness? Why should the man who prefers seeing a movie (and who has to earn money for a ticket) be open to the required call to aid the needy, while the person who prefers looking at a sunset (and hence need earn no extra money) is not? Indeed, isnt it surprising that redistributionists choose to ignore the man whose pleasures are so easily attainable without extra labor, while adding yet another burden to the poor unfortunate who must work for his pleasures? If anything, one would have expected the reverse. Why is the person with the nonmaterial or nonconsumption desire allowed to proceed unimpeded to his most favored feasible alternative, whereas the man whose pleasures or desires involve material things and who must work for extra money (thereby serving whomever considers his activities valuable enough to pay him) is constrained in what he can realize? Perhaps there is no difference in principle. And perhaps some think the answer concerns merely administrative convenience. (These questions and issues will not disturb those who think that forced labor to serve the needy or to realize some favored end-state pattern is acceptable.) In a fuller discussion we would have (and want) to extend our argument to include interests, entrepreneurial profits, and so on. Those who doubt that this extension can be carried through, and who draw the ilne here at taxation of income from labor, will have to state rather complicated patterned historical principles of distributive justice, since end-state principles would not distinguish sources of income in any way. It is enough for now to get away from end-state principles and to make clear how various patterned principles are dependent upon particular views about the sources or the illegitimacy or the lesser legitimacy of profits, interest, and so on; which particular views may well be mistaken.

11PF4-Income Disparities www.victorybriefs.com

Page 120 of 141

REDISTRIBUTION OF WEALTH IS THE EQUIVALENT OF GIVING PEOPLE A PROPERTY RIGHT IN EACH OTHERS LABOR Robert Nozick [Prof., Harvard University], Anarchy, State, and Utopia, New York: Basic Books, Inc., Publishers (1974), p. 171-172 When end-result principles of distributive justice are built into the legal structure of a society, they (as do most patterned principles) give each citizen an enforceable claim to some portion of the total social product; that is, to some portion of the sum total of the individually and jointly made products. This total product is produced by individuals laboring, suing means of production others have saved to bring into existence, by people organizing production or creating means to produce new things or things in a new way. It is on this batch of individual activities that patterned distributional principles give each individual an enforceable claim. Each person has a claim to the activities and the products of other persons, independently of whether the other persons enter into particular relationships that give rise to these claims, and independently of whether they voluntarily take these claims upon themselves, in charity or in exchange for something. Whether it is done through taxation on wages or on wages over a certain amount, or through seizure of profits, or through there being a big social pot so that its not clear whats coming from where and whats going where, patterned principles of distributive justice involve appropriating the actions of other persons. Seizing the results of someones labor is equivalent to seizing hours form him and directing him to carry on vairuos activities. If people force you to do certain work, or unrewarded work, for a certain period of time, they decide what you are to do and what purposes your work is to serve apart from your decisions. This process whereby they take this decision from you makes them a part-owner of you; it gives them a property right in you. Just as having such partial control and power of decisions, by right, over an animal or inanimate object would be to have a property right in it. End-state and most patterned principles of distributive justice institute (partial) ownership by others of people nad their actions and labor. These principles involve a shift from the classical liberals notion of self-ownership to a notion of (partial) property rights in other people.

11PF4-Income Disparities www.victorybriefs.com

Page 121 of 141

TO CLAIM THAT ALL ENTITLEMENTS ARE MORALLY ARBITRARY UNDERMINES HUMAN WORTH Robert Nozick [Prof., Harvard University], Anarchy, State, and Utopia, New York: Basic Books, Inc., Publishers (1974), p. 213 Here we have Rawls reason for rejecting a system of natural liberty: it permits distributive shares to be improperly influenced by factors that are so arbitrary from a moral point of view. These factors are: prior distribution of natural talents and abilities as these have been developed over time by social circumstances and such chance contingencies as accident and good fortune. Notice that there is no mention at all of how persons have chosen to develop their own natural assets. Why is that simply left out? Perhaps because such choices are also viewed as being the products of factors outside the persons control, and hence as arbitrary from a moral point of view. The assertion that a man deserves the superior character that enables him to make the effort to cultivate his abilities is equally problematic; for his character depends in large part upon fortunate family and social circumstances for which he can claim no credit. 34 (What view is presupposed here of character and its relation to action?) The initial endowment of natural assets and the contingencies of their growth and nurture in early life ar arbitrary from a moral point of view the effort a person is willing to make is influenced by his natural abilities and skills and the alternatives open to him. The better endowed are more likely, other things equal, to strive conscientiously. 35 This line of argument can succeed in blocking the introduction of a persons autonomous choices and actions (and their results) only by attributing everything noteworthy about the person completely to certain sorts of external factors. So denigrating a persons autonomy and prime responsibility for his actions is a risky like to take for a theory that otherwise wishes to buttress the dignity and self-respect of autonomous beings; especially for a theory that founds so much (including a theory of the good) upon persons choices. One doubts that the unexalted picture of human beings Rawls theory presupposes and rests upon can be made to fit together with the view of human dignity it is designed to lead to and embody.

11PF4-Income Disparities www.victorybriefs.com

Page 122 of 141

REDISTRIBUTION OF WEALTH THINKS OF PEOPLES NATURAL ABILITIES AS THE COMMON ASSETS OF THE COMMUNITY Robert Nozick [Prof., Harvard University], Anarchy, State, and Utopia, New York: Basic Books, Inc., Publishers (1974), p. 228 Rawls view seems to be that everyone has some entitlement or claim on the totality of natural assets (viewed as a pool), with no one having differential claims. The distribution of natural abilities is viewed as a collective asset. 43 We see then that the difference principle represents, in effect, an agreement to regard the distribution of natural talents as a common asset and to share in the benefits of this distribution whatever it turns out to be. Those who have been favored by nature, whoever they are, may gain from their good fortune only on terms that improve the situation of those who have lost out. No one deserves his greater natural capacity nor merits a more favorable starting place in society. But it does not follow that one should eliminate these basic distinctions. There is another way to deal with them. The basic structure can be arranged so that these contingencies work for the good of the least fortunate. 44 People will differ in how they view regarding natural talents as a common asset. Some will complain, echoing Rawls against utilitarianism, 45 that this does not take seriously the distinction between persons; and they will wonder whether any reconstruction of Kant that treats peoples abilities and talents as resources for others can be adequate. The two principles of justice rule out even the tendency to regard men as means to one anothers welfare. 46 Only if one presses very hard on the distinction between men and their talents, assets, abilities, and special traits. Whether any coherent conception of a person remains when the distinction is so pressed is an open question. Why we, think with particular traits, should be cheered that (only) the thus purified men within us are not regarded as means is also unclear.

11PF4-Income Disparities www.victorybriefs.com

Page 123 of 141

THE ARBITRARINESS OF NATURAL ENDOWMENTS DOES NOT LOGICALLY IMPLY THAT THESE ENDOWMENTS ARE COMMUNAL PROPERTY Michael Sandel [Prof. of Government, Harvard University], Liberalism and the Limits of Justice (Second Edition), Cambridge: Cambridge University Press (1998), p.98 But Nozick has a further objection, independent of the first, that goes something like this: even if R awls is right that arbitrariness undermines individual possession and hence individual desert, the difference principle is not the inevitable result; something like the principle of natural liberty and here Nozick prefers to speak in terms of his own version, which he calls the entitlement theory could still be true. For even if no one deserved any of his natural assets, it might still be that people were entitled to them, and to what flows from them. At best, this argument maintains, Rawls case for the difference principle is underdetermined by the argument from arbitrariness. To show that individuals, as individuals, do not deserve or possess their assets is not necessarily to show that society as a whole does deserve or possess them. Simply because the attributes accidentally located in me are not my assets, why must it follow, as Rawls seems to think, that they are common assets, rather than nobodys assets? If they cannot properly be said to belong to me, why assume automatically that they belong to the community? Is their location in the communitys province any less accidental, any less arbitrary from a moral point of view? And if not, why not regard them as free-floating assets, unattached in advance to any subject of possession, whether individual or social?

11PF4-Income Disparities www.victorybriefs.com

Page 124 of 141

SOCIAL PROGRAMES THAT REDISTRIBUTE WEALTH ARE OFTEN OPPOSED ON GROUNDS THAT THEY FOSTER DEPENDENCE ON THE STATE Francis Fukuyama [Olivier Nomellini Senior Fellow at the Freeman Spogli Institute for International Studies, Stanford University], Dealing with Inequality, Journal of Democracy Volume 22, Number 3 (July 2011), p. 86 Perhaps the most powerful arguments against excessive concern with the problem of inequality have to do with the unintended consequences of social programs, income transfers, and other policies designed to redistribute income or shift the targeting of public services more decisively to the poor. The first and most straightforward of these is the moral hazard argumentnamely, that subsidies or income transfers to the poor reduce incentives to work and increase dependence on the state. In the American context, this was linked to the particular pathologies said to be produced by the New Deals premier welfare program, Aid to Families with Dependent Children (AFDC), which provided subsidies to poor women but only if they were unmarried. The perception that this was fueling the explosion of single-parent families in poor neighborhoods was one of the reasons why AFDC was finally abolished in 1996.21

11PF4-Income Disparities www.victorybriefs.com

Page 125 of 141

ENTITLEMENT PROGRAMS CAUSE UNSUSTAINABLE POLITICAL PATRONAGE Francis Fukuyama [Olivier Nomellini Senior Fellow at the Freeman Spogli Institute for International Studies, Stanford University], Dealing with Inequality, Journal of Democracy Volume 22, Number 3 (July 2011), p. 86 A second argument against social-policy programs is more political: Any subsidy or cash transfer given out by the state, however well-intentioned, becomes a source of rents and therefore of political contestation. The poor who are the beneficiaries of subsidies come to see them as entitlements and try to hang on to them even when they are no longer poor; moreover, subsidies are sought by middle-class voters as well, and therefore tend to expand over time. Politicians catering to voters see social programs as a form of patronage that they can use to build political machines. This produces an unhealthy interest-group politics at best, and clientelism and corruption at worst. The very popularity of social programs thus spurs their growth over time and the fiscal problems that this engenders. It is much easier for a politician to initiate a new program than to take away an old one. Programs that were begun in flush economic times often persist in downturns or periods of low growth and become drags on national budgets. Fiscally unsustainable programs have all sorts of negative implications for macroeconomic stability and therefore long-term economic growth.

11PF4-Income Disparities www.victorybriefs.com

Page 126 of 141

PICKETT AND WILKINSONS STUDY OF ECONOMIC INEQUALITY HAS SERIOUS METHODOLOGICAL PROBLEMS Francis Fukuyama [Olivier Nomellini Senior Fellow at the Freeman Spogli Institute for International Studies, Stanford University], Dealing with Inequality, Journal of Democracy Volume 22, Number 3 (July 2011), p. 83 Let us begin with arguments that inequality undermines or is dangerous to democracy. Kate Pickett and Richard Wilkinson present data, mostly based on cross-country or cross-state analysis, purporting to show that high levels of inequality are correlated with a host of social ills, including drug use, criminality, obesity and other forms of poor health, poor educational outcomes, low social trust, and the like.9 While none of these results necessarily undermines democracy, democratic societies are clearly better off when their citizens are healthy, welleducated, secure, and so on. The statistical analysis in their book is relatively unsophisticated; they fail to test for endogeneity or excluded variables and assume that a correlation implies that the causal direction inevitably runs from inequality to the social ill in question. Moreover, they fail to ask whether poverty reduction brought about by rapid economic growth might cure certain of the social problems that they describe, even in the presence of substantial levels of inequality. Indeed, many years ago Simon Kuznets argued that rapidly growing countries went through a period of increasing economic inequality as certain sectors of the economy pulled ahead of others, but that levels of equality would then increase again as development became more broadbased.10 By this account inequality would, at certain stages of development, be a symptom of economic health.

11PF4-Income Disparities www.victorybriefs.com

Page 127 of 141

SINGLE TIME-POINT INDICATORS OF INEQUALITY FAIL TO CAPTURE SOCIAL MOBILITY ACROSS GENERATIONS Francis Fukuyama [Olivier Nomellini Senior Fellow at the Freeman Spogli Institute for International Studies, Stanford University], Dealing with Inequality, Journal of Democracy Volume 22, Number 3 (July 2011), p. 85 On the other side of the ledger are a number of arguments as to why liberal democracies should not worry very much about the impact of inequality on democracy and why efforts to remedy inequality will be more damaging than the problem itself. One argument made by many Americans in defense of their own countrys relatively high degree of inequality compared to other developed democracies is that single timepoint measurements of income inequality such as the Gini coefficient are less important than the question of intergenerational social mobility. A great deal of inequality is due to life-course effectsover time, people accumulate skills and therefore wealth. What is more important is whether their society permits them to reap the rewards of their acquired skills and effort. Americans have historically tolerated higher degrees of inequality than have Europeans because of Americans perception that their society offers opportunity to rise, if not for them, then certainly for their children and grandchildren.

11PF4-Income Disparities www.victorybriefs.com

Page 128 of 141

INCOME GAINS HAVE ACTUALLY BEEN SHARED MUCH MORE EQUALLY THAN MANY BELIEVE James Pethokoukis [Money & Politics columnist and blogger for Reuters], 5 reasons why income inequality is a myth and Occupy Wall Street is wrong, The American, 18 Oct. 2011 < http://blog.american.com/2011/10/5-reasons-why-income-inequality-is-a-myth-and-occupy-wallstreet-is-wrong/> 1. In a 2009 paper, Northwestern University economist Robert Gordon found the supposed sharp rise in American inequality to be exaggerated both in magnitude and timing. Here is the conundrum: Family income is supposed to rise right along with productivity. But median real household incomeas reported by the Census Bureaugrew just 0.49 percent per year between 1979 and 2007 even as worker productivity grew four times faster at 1.95 percent per year. The wide gap between the two measures, if accurate, would suggest wealthy households rather than middle-class families grabbed most of the income gains from faster productivity. But Gordon explained that this compares apples with oranges, and then oranges with bananas. When various statistical quirks are harmonized between the two economic measures, Gordon found middle-class income growth to be much faster and the conceptually consistent gap between income and productivity growth is only 0.16 percent per year. Thats barely onetenth of the original gap of 1.46 percent. In other words, income gains were shared fairly equally.

11PF4-Income Disparities www.victorybriefs.com

Page 129 of 141

AFTER APPROPRIATE STATISTICAL ADJUSTMENT, STUDIES SHOW MEDIAN HOUSEHOLD INCOME HAS GONE UP SUBSTANTIALLY James Pethokoukis [Money & Politics columnist and blogger for Reuters], 5 reasons why income inequality is a myth and Occupy Wall Street is wrong, The American, 18 Oct. 2011 < http://blog.american.com/2011/10/5-reasons-why-income-inequality-is-a-myth-and-occupy-wallstreet-is-wrong/> 2. A pair of studies from 2007 and 2008 conducted by the Federal Reserve Bank of Minneapolis supports Gordon. Researchers examined why the Census Bureau reported median household income stagnated from 1976 to 2006, growing by only 18 percent. In contrast, data from the Bureau of Economic Analysis showed income per person was up 80 percent. Like Gordon, they found apples-to-oranges issues such as different ways of measuring prices and household size. But in the end, they concluded that after adjusting the Census data for these three issues, inflation-adjusted median household income for most household types is seen to have increased by 44 percent to 62 percent from 1976 to 2006. In addition, research shows that median hourly wages (including fringe benefits) rose by 28 percent from 1975 to 2005.

11PF4-Income Disparities www.victorybriefs.com

Page 130 of 141

MEASURES OF INCOME INEQUALITY FAIL TO CAPTURE THE INCREASING VALUE OF GOVERNMENT SERVICES AND BENEFICIAL TAX TREATMENT James Pethokoukis [Money & Politics columnist and blogger for Reuters], 5 reasons why income inequality is a myth and Occupy Wall Street is wrong, The American, 18 Oct. 2011 < http://blog.american.com/2011/10/5-reasons-why-income-inequality-is-a-myth-and-occupy-wallstreet-is-wrong/> 4. A 2010 study by the University of Chicagos Bruce Meyer and Notre Dames James Sullivan notes that official income inequality statistics indicate a sharp rise in inequality over the past four decades: The ratio of the 90th to the 10th percentile of income, for example, grew by 23 percent between 1970 and 2008. But Meyer and Sullivan point out that income statistics miss a lot, such as the value of government programs and the impact of taxes. The latter, especially, is a biggie. The researchers find that accounting for taxes considerably reduces the rise in income inequality over the past 45 years. In addition, consumption inequality is less pronounced than income inequality.

11PF4-Income Disparities www.victorybriefs.com

Page 131 of 141

IN THE LAST FEW DECADES QUALITY OF LIFE HAS IMPROVED SUBSTANTIALLY FOR ALL AMERICANS James Pethokoukis [Money & Politics columnist and blogger for Reuters], 5 reasons why income inequality is a myth and Occupy Wall Street is wrong, The American, 18 Oct. 2011 < http://blog.american.com/2011/10/5-reasons-why-income-inequality-is-a-myth-and-occupy-wallstreet-is-wrong/> 5. Set all the numbers aside for a moment. If youve lived through the past four decades, does it really seem like America is no better off today? It doesnt to Jason Furman, the deputy director of Obamas National Economic Council. Here is Furman back in 2006: Remember when even upper-middle class families worried about staying on a long distance call for too long? When flying was an expensive luxury? When only a minority of the population had central air conditioning, dishwashers, and color televisions? When no one had DVD players, iPods, or digital cameras? And when most Americans owned a car that broke down frequently, guzzled fuel, spewed foul smelling pollution, and didnt have any of the now virtually standard items like air conditioning or tape/CD players? No doubt the past few years have been terrible. But the past few decades have been pretty goodfor everybody.

11PF4-Income Disparities www.victorybriefs.com

Page 132 of 141

INEQUALITY IN WELLBEING IS SHARPLY DOWN IN THE LAST CENTURY, EVEN IF ECONOMIC INEQUALITY IS GOING UP Tyler Cohen [Prof. of Economics, George Mason University], The Inequality That Matters, The American Interest, January/February 2011, < http://www.the-americaninterest.com/article.cfm?piece=907> In terms of immediate political stability, there is less to the income inequality issue than meets the eye. Most analyses of income inequality neglect two major points. First, the inequality of personal well-being is sharply down over the past hundred years and perhaps over the past twenty years as well. Bill Gates is much, much richer than I am, yet it is not obvious that he is much happier if, indeed, he is happier at all. I have access to penicillin, air travel, good cheap food, the Internet and virtually all of the technical innovations that Gates does. Like the vast majority of Americans, I have access to some important new pharmaceuticals, such as statins to protect against heart disease. To be sure, Gates receives the very best care from the worlds top doctors, but our health outcomes are in the same ballpark. I dont have a private jet or take luxury vacations, andI think it is fair to saymy house is much smaller than his. I cant meet with the worlds elite on demand. Still, by broad historical standards, what I share with Bill Gates is far more significant than what I dont share with him. Compare these circumstances to those of 1911, a century ago. Even in the wealthier countries, the average person had little formal education, worked six days a week or more, often at hard physical labor, never took vacations, and could not access most of the worlds culture. The living standards of Carnegie and Rockefeller towered above those of typical Americans, not just in terms of money but also in terms of comfort. Most people today may not articulate this truth to themselves in so many words, but they sense it keenly enough. So when average people read about or see income inequality, they dont feel the moral outrage that radiates from the more passionate egalitarian quarters of society. Instead, they think their lives are pretty good and that they either earned through hard work or lucked into a healthy share of the American dream. (The persistently unemployed, of course, are a different matter, and I will return to them later.) It is pretty easy to convince a lot of Americans that unemployment and poverty are social problems because discrete examples of both are visible on the evening news, or maybe even in or at the periphery of ones own life. Its much harder to get those same people worked up about generalized measures of inequality.

11PF4-Income Disparities www.victorybriefs.com

Page 133 of 141

DEMOGRAPHIC CHANGE ACCOUNTS FOR MORE ECONOMIC INEQUALITY Tyler Cohen [Prof. of Economics, George Mason University], The Inequality That Matters, The American Interest, January/February 2011, < http://www.the-americaninterest.com/article.cfm?piece=907> The broader change in income distribution, the one occurring beneath the very top earners, can be deconstructed in a manner that makes nearly all of it look harmless. For instance, there is usually greater inequality of income among both older people and the more highly educated, if only because there is more time and more room for fortunes to vary. Since America is becoming both older and more highly educated, our measured income inequality will increase pretty much by demographic fiat. Economist Thomas Lemieux at the University of British Columbia estimates that these demographic effects explain three-quarters of the observed rise in income inequality for men, and even more for women.2

11PF4-Income Disparities www.victorybriefs.com

Page 134 of 141

INCOME INEQUALITY OUTSIDE THE TOP 1% HAS NOT RISEN DRAMATICALLY, AND GROWTH IN THE TOP 1% IS MOSTLY TIED TO PERFORMANCE PAY Tyler Cohen [Prof. of Economics, George Mason University], The Inequality That Matters, The American Interest, January/February 2011, < http://www.the-americaninterest.com/article.cfm?piece=907> Attacking the problem from a different angle, other economists are challenging whether there is much growth in inequality at all below the super-rich. For instance, real incomes are measured using a common price index, yet poorer people are more likely to shop at discount outlets like Wal-Mart, which have seen big price drops over the past twenty years.3 Once we take this behavior into account, it is unclear whether the real income gaps between the poor and middle class have been widening much at all. Robert J. Gordon, an economist from Northwestern University who is hardly known as a right-wing apologist, wrote in a recent paper that there was no increase of inequality after 1993 in the bottom 99 percent of the population, and that whatever overall change there was can be entirely explained by the behavior of income in the top 1 percent.4 And so we come again to the gains of the top earners, clearly the big story told by the data. Its worth noting that over this same period of time, inequality of work hours increased too. The top earners worked a lot more and most other Americans worked somewhat less. Thats another reason why high earners dont occasion more resentment: Many people understand how hard they have to work to get there. It also seems that most of the income gains of the top earners were related to performance paybonuses, in other wordsand not wildly out-of-whack yearly salaries.5

11PF4-Income Disparities www.victorybriefs.com

Page 135 of 141

INCOME INEQUALITY IS DRAMATICALLY INFLATED BY THE EVOLVING TAX TREATMENT OF CERTAIN KINDS OF WEALTH Alan Reynolds [Senior Fellow at The Cato Institute], Has U.S. Income Inequality Really th Increased? Policy Analysis, [Cato Institute], No. 586, January 8 , 2007, p. 1 There are frequent complaints that U.S. income inequality has increased in recent decades. Estimates of rising inequality that are widely cited in the media are often based on federal income tax return data. Those data appear to show that the share of U.S. income going to the top 1 percent (those people with the highest incomes) has increased substantially since the 1970s. However, there have been large changes in U.S. tax rules over time that have made a dramatic difference on what is reported as income on individual tax returns. Tax changes induced thousands of businesses to switch from filing under the corporate tax system to filing under the individual tax system. Corporate executives switched from accepting stock options taxed as capital gains to nonqualified stock options taxed as salaries. The huge growth in tax-favored savings plans, such as 401(k)s, has resulted in billions of dollars of investment income disappearing from tax returns. Meanwhile, studies of inequality that are based on tax return data usually exclude transfer payments, which results in exaggerating the shares of income received by those at the top by ignoring growing amounts of income at the bottom.

11PF4-Income Disparities www.victorybriefs.com

Page 136 of 141

STATISTICS SHOWING INCREASED INCOME INEQUALITY ARE DISTORTED BY THE FACT THAT DECREASED MARGINAL TAX RATES IN RECENT YEARS HAVE CAUSED THE WEALTHY TO REPORT MORE INCOME THAN THEY OTHERWISE WOULD Alan Reynolds [Senior Fellow at The Cato Institute], Has U.S. Income Inequality Really th Increased? Policy Analysis, [Cato Institute], No. 586, January 8 , 2007, p. 1 Measurements of inequality have also been affected by large reductions in income tax rates, particularly in 1986. Estimates by many economists indicate that the reported income of highincome taxpayers is very responsive to tax rates. When top tax rates on wages or capital gains fall, reported incomes rise, and a larger fraction of the incomes of those at the top show up on tax returns. International comparisons show that reported income shares of those at the top have risen the most where top tax rates have been cut the most (the United States, the United Kingdom, and India) and have risen the least where top tax rates have remained very high (France and Japan).

11PF4-Income Disparities www.victorybriefs.com

Page 137 of 141

EVIDENCE SUGGESTING INCREASING INCOME INEQUALITY IS INCONSISTENT WITH MOST DATA Alan Reynolds [Senior Fellow at The Cato Institute], Has U.S. Income Inequality Really th Increased? Policy Analysis, [Cato Institute], No. 586, January 8 , 2007, p. 18 The widespread impression that the United States has experienced a large and continuous increase in income inequality since the 1970s is almost entirely dependent on the disingenuous practice of using estimates based on income tax returns to compare the distribution of incomes before and after the dramatic tax changes of 1981 and 1986. If the Piketty and Saez estimates actually demonstrated a continuous and credible upward trend toward greater inequality since the late-1980s, all other estimates of income distribution would have to be wrong including those of the Census Bureau, the CBO, and the Federal Reserve Board.

11PF4-Income Disparities www.victorybriefs.com

Page 138 of 141

EVIDENCE FOR INCREASING INCOME INEQUALITY HAS BEEN SUBSTANTIALLY DISTORTED Alan Reynolds [Senior Fellow at The Cato Institute], Has U.S. Income Inequality Really th Increased? Policy Analysis, [Cato Institute], No. 586, January 8 , 2007, p. 18-19 In a recent column, Paul Krugman complained about the amount of time that inequalitys apologists spend attacking a claim nobody is making: that there has been a clear long-term decline in middle-class living standards. 51 Yet, in a 2004 column Krugman said, according to estimates by the economists Thomas Piketty and Emmanuel Saezconfirmed by data from the CBObetween 1973 and 2000 the average real income of the bottom 90 percent of American taxpayers actually fell by 7 percent. 52 For Krugman to assert that there was a 7 percent drop in real income for 90 percent of taxpayers over 27 years certainly sounds like a clear long-term decline in middle-class living standards. To question such claims requires no straw man. On the contrary, Krugmans astonishingly incorrect assertion provided an excellent example of how remarkably uncritical even professional economists have become toward the Piketty-Saez estimates. Piketty and Saez recently acknowledged that, our long-run series are generally confined to top income and wealth shares and contain little information about bottom segments of the distribution. 53 Yet one of their figures from their 2001 paper (Figure A-1) encouraged the exact opposite impression. The figure appeared to show 27 years of real income stagnation (not decline) for the bottom 99 percent of taxpayers rather than Krugmans bottom 90 percent. In a key footnote to that graph, however, they explain that from 1973 to 2000, the average income of the bottom 99 percent would have grown by about 40 percent in real terms instead of stagnating (as displayed in the figure above) if we had included all transfers (+7% effect), used the CPI-URS (+13% effect), and especially defined income per capita (20% effect). 54 That 40 percent increase in per capita real income for the bottom 99 percent makes it quite impossible that income of the bottom 90 percent of American taxpayers actually fell by 7 percent, as Krugman wrote. CBOs estimates began in 1979, not 1973, and they certainly do not confirm what Piketty and Saez deny (in the footnote). Between 1979 and 2000, the CBO estimates indicate that the average real income of the bottom 80 percent of American taxpayers rose by 12 percent before taxes and by 15 percent after taxes. 55 Real after-tax income of the middle quintile rose from $38,900 in 1979 (in 2003 dollars) to $44,700 in 2000, according to the CBO. Census Bureau estimates of mean household income among the bottom four quintiles rose from $32,786 in 1973 (in 2005 dollars) to $40,640 in 2000. 56 That is, Census estimates show a 24 percent real increase for the bottom 80 percentwith all of that gain occurring since 1982. In fact, the Census estimates show an accelerating pace of gains among the bottom 80 percent, with real income in that group rising 7.6 percent from 1970 to 1980, 9.6 percent from 1980 to 1990, and 12.4 percent from 1990 to 2000. 57

11PF4-Income Disparities www.victorybriefs.com NEW POVERTY METRICS ARE HUGELY DISTORTED

Page 139 of 141

Robert Rector [Senior Research Fellow in Domestic Policy at the Heritage Foundation] and Rachel Sheffield [Research Assistant, Heritage Foundation], Worse than useless measure of poverty, New York Post, 12 Nov. 2011, <http://www.nypost.com/p/news/opinion/opedcolumnists/worse_than_useless_measure_of_pover ty_5P2VXYeXBkjQaAlOuiWmMK> On Monday, the Census Bureau released a new poverty measure created by the Obama administration. Using this new calculation, Census officials assert that more than 49 million Americans are poor -- roughly 3 million more than they reported in Septembers annual poverty report. The news was previewed last week in an Associated Press story, which cited the Census Bureau in reporting that the ranks of Americas poorest poor have climbed to a record high. It said the data also indicate poverty is spreading further into mainstream America. In fact, these reports rely on flawed and misleading poverty measures. The Census Bureaus old measure failed to fully take into account the nearly $1 trillion a year in federal welfare benefits received by low-income families. How many of these benefits were counted toward an individual or family income when calculating poverty? Almost none. Thats part of the reason that, despite decades of increased welfare spending, poverty rates remained nearly unchanged. The new measure, released Monday, does take more of these benefits into account when calculating poverty. But it further distorts the picture of poverty by placing income thresholds on an automatic elevator that climbs as overall income rises. Thus, by this measure, even if the income of all Americans doubled immediately, poverty wouldnt decrease, because the income thresholds would also double. Poverty would fall only if the incomes of the poor rise more quickly than those of the rest of the population. Inaccurate as the old measure was, the new measure is much worse. It will ensure that poverty cant be alleviated except by extreme income leveling.

11PF4-Income Disparities www.victorybriefs.com

Page 140 of 141

WE ARE LESS CONCERNED WITH ECONOMIC INEQALITY AS SUCH THAN WE ARE THAT THE SYSTEM ALLOWS SOME PEOPLE TO GET RICH UNDESERVEDLY Tyler Cohen [Prof. of Economics, George Mason University], The Inequality That Matters, The American Interest, January/February 2011, < http://www.the-americaninterest.com/article.cfm?piece=907> This is why, for example, large numbers of Americans oppose the idea of an estate tax even though the current form of the tax, slated to return in 2011, is very unlikely to affect them or their estates. In narrowly self-interested terms, that view may be irrational, but most Americans are unwilling to frame national issues in terms of rich versus poor. Theres a great deal of hostility toward various government bailouts, but the idea of undeserving recipients is the key factor in those feelings. Resentment against Wall Street gamesters hasnt spilled over much into resentment against the wealthy more generally. The bailout for General Motors labor unions wasnt so popular eitheragain, obviously not because of any bias against the wealthy but because a basic sense of fairness was violated. As of November 2010, congressional Democrats are of a mixed mind as to whether the Bush tax cuts should expire for those whose annual income exceeds $250,000; that is in large part because their constituents bear no animus toward rich people, only toward undeservedly rich people.

11PF4-Income Disparities www.victorybriefs.com

Page 141 of 141

EMBRACING FAMILY VALUES CAUSE ECONOMIC CHOICES WHICH EXACERBATE INEQUALITY BUT WHICH ARE NONETHELESS MORALLY PRAISEWORTHY Tyler Cohen [Prof. of Economics, George Mason University], The Inequality That Matters, The American Interest, January/February 2011, < http://www.the-americaninterest.com/article.cfm?piece=907> It is also the case that any society with a lot of threshold earners is likely to experience growing income inequality. A threshold earner is someone who seeks to earn a certain amount of money and no more. If wages go up, that person will respond by seeking less work or by working less hard or less often. That person simply wants to get by in terms of absolute earning power in order to experience other gains in the form of leisurewhether spending time with friends and family, walking in the woods and so on. Luck aside, that persons income will never rise much above the threshold. Its not obvious what causes the percentage of threshold earners to rise or fall, but it seems reasonable to suppose that the more single-occupancy households there are, the more threshold earners there will be, since a major incentive for earning money is to use it to take care of other people with whom one lives. For a variety of reasons, single-occupancy households in the United States are at an all-time high. There are also a growing number of late odyssey years graduate students who try to cover their own expenses but otherwise devote their time to study. If the percentage of threshold earners rises for whatever reasons, however, the aggregate gap between them and the more financially ambitious will widen. There is nothing morally or practically wrong with an increase in inequality from a source such as that. The funny thing is this: For years, many cultural critics in and of the United States have been telling us that Americans should behave more like threshold earners. We should be less harried, more interested in nurturing friendships, and more interested in the non-commercial sphere of life. That may well be good advice. Many studies suggest that above a certain level more money brings only marginal increments of happiness. What isnt so widely advertised is that those same critics have basically been telling us, without realizing it, that we should be acting in such a manner as to increase measured income inequality. Not only is high inequality an inevitable concomitant of human diversity, but growing income inequality may be, too, if lots of us take the kind of advice that will make us happier.

Das könnte Ihnen auch gefallen